You are on page 1of 147

SAT

Practice Tests
10

Contents
Practice Test 1 Section 1 Writing Essay Section 2 Critical Reading .... Section 5 Critical Reading Section 7 Writing .......... Section 8 Critical Reading Section 10 Writing ...... Practice Test 2 Section 1 Writing Essay .... Section 2 Critical Reading .... Section 5 Critical Reading Section 7 Writing .. Section 8 Critical Reading .... Section 10 Writing .. Practice Test 3 Section 1 Writing Essay Section 3 Critical Reading Section 6 Writing .. Section 7 Critical Reading .... Section 9 Critical Reading Section 10 Writing .. Practice Test 4 Section 1 Writing Essay Section 3 Writing .. Section 4 Critical Reading .... Section 6 Critical Reading Section 9 Critical Reading Section 10 Writing .. Practice Test 5 Section 1 Writing Essay Section 4 Critical Reading .... Section 5 Writing ..... Section 7 Critical Reading Section 8 Critical Reading Section 10 Writing .. Practice Test 6 Section 1 Writing Essay Section 2 Critical Reading Section 4 Critical Reading Section 6 Writing .. Section 8 Critical Reading .... Section 10 Writing ...... 124 125 130 136 142 145 99 100 105 111 117 121 74 75 81 87 92 96 49 50 56 62 67 71 25 26 31 36 42 46 2 3 7 13 19 22

Practice Test 1

15 SECTION 1 WRITING ESSAY

Time 25 minutes

20The essay gives you an opportunity to show how effectively you can develop and express ideas. You should, therefore, take care to develop your point of view, present your ideas logically and clearly, and use language precisely.

Your essay must be written on the lines provided on your answer sheet you will receive no other paper on which to write. 25You will have enough space if you write on every line, avoid wide margins, and keep your handwriting to a reasonable size. Remember that people who are not familiar with your handwriting will read what you write. Try to write or print so that what you are writing is legible to those readers.
30You have twenty-five minutes to write an essay on the topic assigned below. DO NOT WRITE ON ANOTHER TOPIC. AN OFF-TOPIC ESSAY WILL RECEIVE A SCORE OF ZERO.

Think carefully about the issue presented in the following quotations and the assignment below.
35

To change is to risk something, making us feel insecure. Not to change is a bigger risk, though we seldom feel that way. There is no choice but to change. People, however, cannot be motivated to change from the outside. All of our motivation comes from within. Adapted from Ward Sybouts, Planning in School Administration: A Handbook

40

45Assignment:

What motivates people to change? Plan and write an essay in which you develop your point of view on this issue. Support your position with reasoning and examples taken from your reading, studies, experience, or observations.

50

Practice Test 1 SECTION 2


55

CRITICAL READING
24 Questions

Time 25 minutes

Directions: For each question in this section, select the best answer from among the choices given and fill in the corresponding circle on the answer sheet.

60
Each sentence below has one or two blanks, each blank indicating that something has been omitted. Beneath the sentence are five words or sets of words labeled A through E. Choose the word or set of words that, when inserted in the sentence, best fits the meaning of the sentence as a whole. Example: Hoping to ------- the dispute, negotiators proposed a compromise that they felt would be ------- to both labor and management. (A) (B) (C) (D) (E) enforce .. useful end .. divisive overcome .. unattractive extend .. satisfactory resolve .. acceptable

904. More valuable and comprehensive than any previously proposed theory of the phenomenon, Salazars research has ------- the basis for all subsequent ------- in her field. 95

(A) (B) (C) (D) (E)

undermined .. advancements prepared .. debacles provided .. investigations dissolved .. experiments reinforced .. misconceptions

1005. Dangerously high winds ------- attempts to begin the space shuttle mission on schedule, delaying the launch by nearly a week.

(A) thwarted (B) forfeited (C) implemented (D) discharged (E) redoubled
105

1. The ------- of Maria Irene Fornes play Mud a realistic room perched on a dirt pile challenges conventional interpretations of stage scenery.
65

6. The guest speaker on Oprah Winfreys talk show offended the audience by first ------- them and then refusing to moderate these ------- remarks.
110

(A) appeal(B) plot (C) mood (D) setting (E) rehearsal

2. Ironically, an affluent society that purchases much more food than it actually needs suffers because of that 70 -------, since in conditions of affluence diseases related to overeating and poor nutrition seem to -------. (A) (B) (C) (D) (E) lavishness .. adapt overabundance .. thrive corpulence .. vex practicality .. awaken commonness .. abound

(A) (B) (C) (D) (E)

flattering .. commendable haranguing .. intemperate praising .. radical enraging .. conciliatory accommodating .. indulgent

1157. By the end of the long, arduous hike, Chris was walking with a ------- gait, limping slowly back to the campsite.

75

(A) halting (D) prompt


120

(B) robust (E) facile

(C) constant

3. Because of the ------- effects of the not springs, tourists suffering from various ailments flocked to the villages 80 thermal pools. (A) succulent (D) mandatory (B) redolent (C) cerebral (E) therapeutic

8. Actors in melodramas often emphasized tense moments by being -------, for example, raising their voices and pretending to swoon.
125

(A) imperious (D) histrionic

(B) inscrutable (E) solicitous

(C) convivial

85

The passages below are followed by questions based on their content; questions following a pair of related passages may also be based on the relationship between the paired passages. Answer the questions on the basis of what is stated or implied in the passages and in any introductory material that may be provided.

130
Questions 9-12 are based on the following passages. Passage 1 The intelligence of dolphins is well documented by science. Studies show that dolphins are able to understand sign language, solve puzzles, and use objects in their 135environment as tools. Scientist also believe that dolphins possess a sophisticated language: numerous instances have been recorded in which dolphins transmitted information from one individual to another. A recent experiment proved that dolphins can even recognize themselves in a mirror 140something achieved by very few animals. This behavior demonstrates that dolphins are aware of their own individuality, indicating a level of intelligence that may be very near our own. Passage 2 Are dolphins unusually intelligent? Dolphins have 145large brains, but we know that brain size alone does not determine either the nature or extent of intelligence. Some researchers have suggested that dolphins have big brains because they need them for sonar and sound processing and for social interactions. Others have argued 150that regardless of brain size, dolphins have an intelligence level somewhere between that of a dog and a chimpanzee. The fact is, we dont know, and comparisons may not be especially helpful. Just as human intelligence is appropriate for human needs, dolphin intelligence is right for the 155dolphins way of life. Until we know more, all we can say is that dolphin intelligence is different. 9. In lines 2-8, the author of Passage 1 mentions activities that suggest dolphins (A) (B) (C) (D) (E) are unusually sensitive to their environment do not generally thrive in captivity have a unique type of intelligence are uncommonly playful animals have skills usually associated with humans 10. The author of Passage 2 would most likely respond to the last sentence of Passage 1 by (A) suggesting that intelligence in animals is virtually impossible to measure (B) observing that intelligence does not mean the same thing for every species (C) questioning the objectivity of the studies already conducted (D) noting that dolphin activities do not require a high level of intelligence (E) arguing that little is actually known about dolphin social behavior 11. The two passages differ in their views of dolphin intelligence in that Passage 1 states that dolphins (A) share a sophisticated culture, while Passage 2 contends that dolphin intelligence is roughly equal to human intelligence (B) are as intelligent as humans, while Passage 2 notes that dolphins outperform other animals (C) are more intelligent than most other animals, while Passage 2 points out that dolphins are less intelligent than other mammals (D) are highly intelligent, while Passage 2 suggests that there is not enough evidence to understand dolphin intelligence fully (E) have large brains, while Passage 2 argues that brain size does not signify intelligence 12. Which generalization about dolphins is supported by both passages? (A) (B) (C) (D) (E) They display self-awareness. They are more emotional than other animals. They learn at a rapid rate. They have a certain degree of intelligence. They have shown the ability to use tools.

10

Questions 13-24 are based on the following passage. The following passage appeared in an essay written in 1987 in which the author, who is of Native American descent, examines the representation of Native Americans during the course of United States history. In many respects living Native Americans remain as mysterious, exotic, and unfathomable to their contemporaries at the end of the twentieth century as they were to the 160Pilgrim settlers over three hundred fifty years ago. Native rights, motives, customs, languages, and aspirations are misunderstood by Euro-Americans out of a culpable ignorance that is both self-serving and self-righteous. Part of the problem may well stem from the long-standing ten165dency of Europeans or Euro-American thinkers to regard Native Americans as fundamentally and profoundly different, motivated more often by mysticism than by ambition, charged more by unfathomable visions than by intelligence or introspection. 170 This idea is certainly not new. Rousseaus* noble savages wandered, pure of heart, through a pristine world. Since native people were simply assumed to be incomprehensible, they were seldom comprehended. Their societies were simply beheld, often through cloudy glasses, and 175rarely probed by the tools of logic and deductive analysis automatically reserved for cultures prejudged to be civilized. And on those occasions when Europeans did attempt to formulate an encompassing theory, it was not, ordinarily, on a human-being-to-human basis, 180but rather through an ancestor-descendant model. Native Americans, though obviously contemporary with their observers, were somehow regarded as ancient, examples of what Stone Age Europeans must have been like Its a great story, an international crowd pleaser, but 185there is a difficulty: Native Americans were, and are, Homo sapiens sapiens. Though often equipped with a shovel-shaped incisor tooth, eyes with epicanthic folds, or an extra molar cusp, Native American people have had to cope, for the last forty thousand years or so, just like 190everyone else. Their cultures have had to make internal sense, their medicines have had to work consistently and practically, their philosophical explanations have had to be reasonably satisfying and dependable, or else the ancestors of those now called Native Americans would truly have 195vanished long ago. The reluctance in accepting this obvious fact comes from the Eurocentric conviction that the West holds a monopoly on science, logic, and clear thinking. To admit that other, culturally divergent viewpoints are 200equally plausible is to cast doubt on the monolithic center of Judeo-Christian belief: that there is but one of everything God, right way, truth and Europeans alone knew what that was. If Native American cultures

were acknowledged as viable, then European societies 205were something less than an exclusive club. It is little wonder, therefore, that Native Americans were perceived not so much as they were but as they had to be, from a European viewpoint. They dealt in magic, not method. They were stuck in their past, not guided by its precedents. 210 Such expedient misconception argues strongly for the development and dissemination of a more accurate, more objective historical account of native peoples a goal easier stated than accomplished. Native American societies were nonliterate before and during much of the early period 215of their contact with Europe, making the task of piecing together a history particularly demanding. The familiar and reassuring kinds of written documentation found in European societies of equivalent chronological periods do not exist, and the forms of tribal record preservation available oral 220history, tales, mnemonic devices, and religious rituals strike university-trained academics as inexact, unreliable, and suspect. Western historians, culture-bound by their own approach to knowledge, are apt to declaim that next to nothing, save the evidence of archaeology, can be known 225of early Native American life. To them, an absolute void is more acceptable and rigorous than an educated guess. However, it is nave to assume that any cultures history is perceived without subjective prejudice. Every modern observer, whether he or she was schooled in the traditions 230of the South Pacific or Zaire, of Hanover, New Hampshire, or Vienna, Austria, was exposed at an early age to one or another form of folklore about Native Americans. For some, the very impressions about Native American tribes that initially attracted them to the field of American history 235are aspects most firmly rooted in popular myth and stereotype. Serious scholarship about Native American culture and history is unique in that it requires an initial, abrupt, and wrenching demythologizing. Most students do not start from point zero, but from minus zero, and in the process are 240often required to abandon cherished childhood fantasies of superheroes or larger0than-life villains.
* Rousseau was an eighteenth-century French philosopher.

13. The reference to the Pilgrim settlers (lines 3-4) is used to (A) (B) (C) (D) (E) invite reflection about a less complicated era suggest the lasting relevance of religious issues establish a contrast with todays reformers debunk a myth about early colonial life draw a parallel to a current condition

24514. In line 12, charged most nearly means

300

250

(A) (B) (C) (D) (E)

commanded indicated replenished inspired attacked

20. In lines 66-70, the author portrays Western historians as (A) (B) (C) (D) (E) oblivious to the value of archaeological research disadvantaged by an overly narrow methodology excessively impressed by prestigious credentials well meaning but apt to do more harm than good anxious to contradict the faulty conclusions of their predecessors

305

15. In line 14, the reference to Rousseau is used to emphasize the


255

(A) (B) (C) (D) (E)

philosophical origins of cultural bias longevity of certain types of misconceptions tendency to fear the unknown diversity among European intellectual traditions argument that even great thinkers are fallible

31021. The educated guess mentioned in line 70 would most likely be based on

26016. The phrase international crowd pleaser (line 28) refers to

315

265

(A) (B) (C) (D) (E)

an anthropological fallacy an entertaining novelty a harmless deception a beneficial error a cultural revolution

(A) compilations of government population statistics (B) sources such as oral histories and religious rituals (C) analyses of ancient building structures by archaeologists (D) measurements of fossils to determine things such as physical characteristics (E) studies of artifacts discovered in areas associated with particular tribes 22. The geographical references in lines 74-75 serve to underscore the (A) influence Native American culture has had outside the United States (B) argument that academic training is undergoing increasing homogenization (C) universality of certain notions about Native American peoples (D) idea that Native Americans have more in common with other peoples than is acknowledged (E) unlikelihood that scholars of Native American history will settle their differences 23. The passage suggests that "Most students" (line 82) need to undergo a process of (A) (B) (C) (D) (E) rebelliousness disillusionment hopelessness inertia self-denial

320

17. The difficulty referred to in line 29 most directly undermines


270

325

275

(A) the ancestor-descendant model used by European observers (B) the possibility for consensus in anthropological inquiry (C) efforts to rid popular culture of false stereotypes (D) theories based exclusively on logic and deductive reasoning (E) unfounded beliefs about early European communities

330

28018. Lines 34-37 (Their cultures ... dependable) describe

335

285

(A) customs that fuel myths about a society (B) contradictions that conventional logic cannot resolve (C) characteristics that are essential to the survival of any people (D) criteria that Western historians traditionally use to assess cultures (E) preconditions that must be met before a culture can influence others 19. The two sentences that begin with They in lines 52-53 serve to express the (A) (B) (C) (D) way one group perceived another results of the latest research theories of Native Americans about Europeans external criticisms that some Native Americans accepted (E) survival techniques adopted by early human societies

340

24. In line 83, minus zero refers to the (A) nature of the preconceptions held by most beginning scholars of Native American culture (B) quality of scholarship about Native American cultures as currently practiced at most universities (C) reception that progressive scholars of Native American history have received in academia (D) shortage of written sources available. to students of Native American history (E) challenges that face those seeking grants to conduct original research about Native American history

290

345

295

350

355

15 SECTION 5 CRITICAL READING


24 Questions
360

Time - 25 minutes

Directions: For each question in this section, select the best answer from among the choices given and fill in the corresponding circle on the answer sheet.

365
Each sentence below has one or two blanks, each blank indicating that something has been omitted. Beneath the sentence are five words or sets of words labeled A through E. Choose the word or set of words that, when inserted in the sentence, best fits the meaning of the sentence as a whole. Example: Hoping to ------- the dispute, negotiators proposed a compromise that they felt would be ------- to both labor and management. (A) (B) (C) (D) (E) enforce .. useful end .. divisive overcome .. unattractive extend .. satisfactory resolve .. acceptable
3952. Her dislike of ------- made her regard people who tried to win her approval through praise as -------.

400

(A) (B) (C) (D) (E)

autocrats .. dictators defiance .. toadies tyrants .. connoisseurs adulation .. superiors flattery .. sycophants

3. Some scientists speculate that a small pterosaur of the Jurassic period known as Sordes pilosus had ------ . 405 wings that were thin, pliable, and somewhat transparent. (A) callous (B) arable (C) inflexible (D) membranous (E) viscous
410

4. To reflect the ------- of that nations spoken languages, its writers often make use of a mixture of dialects. (A) articulation (D) profundity (B) intonation (C) spontaneity (E) heterogeneity

370

1. Soon after the first visitors arrived, increasing numbers of the residents of the remote island thought it possible that the outside world, instead of being -------, could be and worth exploring. (A) (B) (C) (D) (E) insular .. unlimited friendly .. wicked amiable .. cooperative threatening .. fascinating forbidding .. harmful

415

5. She apologized profusely, only to discover that her self-serving excuses failed to have a ----:-- effect. (A) reprehensible (B) palliative (C) depreciatory (D) litigious (E) compendious

375

420

380

385

390

Each passage below is followed by questions based on its content. Answer the questions on the basis of what is stated or implied in the passages and in any introductory material that may be provided.

425

Questions 6-7 are based on the following passage. Questions 8-9 are based on the following passage. Sometimes the meaning of old phrases is self-evident, as with to move like greased lightning and a close shave. But quite often we are left with language that seems to have sprung out of the blue and does not appear to signify 430anything in particular even steven, fit as a fiddle, or to paint the town red. Explanations are frequently posited but are too often unpersuasive. One popular dictionary, for example, suggests that to be joshing might be connected to the humorist Josh Billings, but in fact the term was current 435as early as 1845. Josh Billings was unknown outside his neighborhood until 1860. 6. Which of the following phrases would the author he most likely to add to the list in lines 5-6 ? (A) (B) (C) (D) (E) To take a chance To jump for joy To lend an ear To talk through your hat To flare up The following study is concerned with Western cities from the Middle Ages up to the twentieth century, in terms of who did what, why, where, and when. It aims to start 440with the functions that have drawn people to cities, and to work outward from them to the spaces and buildings that grew up to cater to them. Savoring cities in ignorance or drinking them in visually is not enough; I want to find out not just who designed the buildings and when they were 445built but why they were built. 8. Which of the following would most likely be found at the beginning of this study? (A) A statistical analysis of crime rates in several ancient Western cities (B) A discussion of the role of central marketplaces in the early Middle Ages (C) A series of portraits of famous people who have chosen city life (D) An account of the architectural challenges involved in building large cathedrals (E) An essay on ancient archaeological sites worth visiting today 9. The primary purpose of the passage is to (A) (B) (C) (D) (E) criticize a study justify an expense explain an approach depict an era defend a decision

7. The last sentence of the passage primarily serves to (A) (B) (C) (D) (E) cite a well-known fact invalidate a theory make a veiled accusation note a puzzling incident explain the origins of a phrase

Questions 10-18 are based on the following passage. In this passage, a British novelist and critic recalls a favorite painting. The first painting I ever bought was by Sheila Fell. I went to her studio in Redcliffe Square feeling uncom-. fortable and even embarrassed, thinking how awful to be an artist, having to put up with prospective buyers coming 450to gape, whereas writers never need to see anyone read their books. I kept wishing, all the way up the steep flights of stairs, that I could go and look without Sheila being there. I imagined she must be feeling the same. I was wrong. Sheila didnt care who looked at her 455paintings or what they thought of them or whether she sold them. She was perfectly at ease, seemed to me to enjoy showing her work. There was a confidence about how she propped up canvas after canvas that made me in turn relax. I dont know why Id been so apprehensive after all, 460we had Cumberland in common, there was no need for me to explain why I was drawn to her work. What I missed, exiled in London, she missed: the landscape of where we had both been born and brought up. The painting was of a haystack in a field. The haystack 465 had clearly just been made. it was golden and the field flooded with a red-gold light. the whole atmosphere mellow and rich. It was a large painting and I realized as soon as it arrived at my home that however much I loved it I had no wall and 470no room to do it justice. I put it on the largest wall we had in the biggest room and still I felt I was insulting it the power of the picture was too huge to be contained in our ordinary house. And the light was wrong. The painting couldnt glow as it wanted to it needed a vast, empty 475room and a great distance in front of it. One day, I hoped, Id take it back to Cumberland and find a house there where it could settle happily. But when, after thirty years, we found that house, the painting was failed again. The walls were no bigger and neither were the rooms. So I sold 480the painting and bought another, smaller Sheila Fell. It was a terrible mistake. The moment the painting had been taken away I realized how stupid Id been. So it had been overwhelming, too large, too dramatic to contain in either house but I shouldnt have let that matter, I should 485have found a way to keep it. I grieved for it and wished I could buy it back, marry it again after the folly of a divorce. But it was too late. And then, in 1990, I went to the Sheila Fell Exhibition at the Royal Academy and there, in pride of place, at the end of the longest room, the room it had 490always needed, was my painting. Its beauty was stunning. People stopped and stared and admired and I wanted to shout that what they were looking at was mine. I am not at all possessive by nature but suddenly I felt fiercely possessive. This glorious painting had been part of my life 495for so very long and I didnt seem to be able to grasp that I had wilfully let it go.

I went back to the exhibition day after day and on the last one became almost maudlin at saying my good-byes. I dont know who owns the painting now it merely said 500Private Collection in the catalog but I doubt if Ill ever see it again. In a way, thats better than being able to go and look at it hanging in a public gallery Id only go on torturing myself with wanting it back. I can see every detail of it in my minds eye anyway. It lives in my head. I can 505recite it like a poem, and so in a sense I can never lose it.

10. Which statement best summarizes the description of the hypothetical group of people in lines 4-5 compared to that of the actual group in line 46? (A) The first is uneducated; the second has professional training. (B) The first slights the artist; the second is overly respectful. (C) The first is somewhat intrusive; the second is apparently appreciative. (D) The first rejects the artists methodology; the second praises it. (E) The first is acquisitive; the second is generous and giving. 11. Line 8 (I imagined ... the same) suggests that the narrator (A) believes that most artists feel as she does in the presence of an audience (B) is as excited about Sheila Fells work as she is about her own (C) is insecure about promoting her books in front of prospective buyers (D) regards Sheila Fells attitude as eccentric (E) enjoys the company of artists and writers 12. The central contrast between the first paragraph (lines 1-8) and the second (lines 9-18) is best described in which terms? (A) (B) (C) (D) (E) Idealism versus practicality Expectation versus reality Speculation versus investigation Anticipation versus disappointment Generosity versus possessiveness

20

13. In line 25, the author assumes that justice would be


510

515

(A) recognizing the unique achievements of an artist (B) ensuring that a work of art reaches the widest possible audience (C) displaying a work of art to its best advantage (D) enhancing ones daily life with beautiful art (E) providing elegant surroundings for exceptional paintings 14. It was a terrible mistake (line 36) because the narrator

57016. The narrator says that for her the painting is like a poem (line 60) because it

575

(A) may be shared with others as a source of pleasure (B) is essential to the narrators sense of identity (C) represents the narrators longing for beautiful objects (D) makes a powerful first impression upon the narrator (E) is preserved vividly within the narrators mind

520

525

(A) had no other souvenirs of Cumberland (B) allowed pragmatic concerns to override her fondness for the painting (C) did not realize how valuable the painting would become to collectors (D) felt that she had betrayed Sheila Fells trust (E) was unable to appreciate the smaller Sheila Fell painting

58017. In the closing paragraphs, the narrator uses the language of human interaction in describing the painting in order to emphasize the

585

(A) (B) (C) (D) (E)

empathy she feels with its creator difficulty she encounters in maintaining it pressure she feels to divorce it extent to which she feels its loss quality of her nostalgia for what it depicts

18. The passage serves mainly to 15. In line 41, the metaphor describing folly suggests 530 that paintings can (A) lose their aura when seen too often in familiar surroundings (B) reinforce misleading recollections of childhood places (C) arouse strong emotions in their owners (D) provoke artists to make premature decisions (E) bring back painful memories of what they depict
590

595

535

(A) discuss the influence of environment on artistic achievement (B) defend the works of a controversial artist (C) explore the emotional context of a particular series of events (D) argue against placing undue emphasis on the economic value of art (E) stimulate interest in an overlooked artistic genre

540

545

550

555

560

565

Questions 19-24 are based on the following passage. The following passage is excerpted from a review of a book about aviation's early years. Aviation belonged to the new century in part because 600the engineering that went into flying machines was utterly different from that of the Industrial Revolution, Nineteenthcentury engineering revolved around the steam engine. It was about weight and brute power beautifully machined heavy steel, burnished bronze, polished copper pipes, 605ornamental cast iron everything built, with no expense spared, to withstand great pressures and last any number of lifetimes. Airplane construction was the opposite of all that; it was about lightness. The Wright brothers, who created one of the first 610airplanes, started out making bicycles, which were all the rage at the turn of the century. They knew about thin-wall steel tubes, wire-spoked wheels, chain drives, and whatever else it took to construct efficient machines that weighed as little as possible. In effect, they were practical engineers at 615the cheap end of the market, but they happened to be fascinated by flight. Says one writer, Wilbur [Wright] spent his time studying the flight of vultures, eagles, ospreys, and hawks, trying to discover the secret of their ability to maneuver with their wings in unstable air. To 620those who later asked him how he learned to fly, he loved to reply through his scarcely opened lips: Like a bird. This is the point at which engineering intersects with the imagination, with humanitys ancient dream of freeing itself from gravity. Until the first fliers got to work, the 625body was earthbound, but it enclosed a soul that flew in meditation, in poetry, and, as the seventeenth-century English poet Andrew Marvell showed, sometimes spectacularly in both:
630

19. The primary purpose of the passage is to (A) profile the unique personalities of aviation pioneers (B) examine the theme of flight in contemporary poetry (C) survey the effects of aviation on twentieth-century lifestyles (D) explain important principles of flight in nontechnical language (E) discuss how early aviation captured peoples imagination 20. In lines 3-9, the description of the steam engine is primarily intended to illustrate (A) how train engineers provided a model that aviation engineers could follow (B) how the Industrial Revolution accelerated societys interest in travel (C) a form of engineering that emphasized immense mass and strength (D) a twentieth-century preoccupation with style over practicality (E) an inefficient mode of transportation whose value was overrated 21. The author refers to the cheap end of the market (line 17) to make the point that (A) aviations progress was hindered by people who had little concern for quality (B) the public could afford to fly because airplanes used inexpensive materials (C) aviators were the target of unwarranted and petty criticism (D) the pioneers of aviation had modest technological beginnings (E) nineteenth-century engineering methods were too extravagant 22. In lines 31-36. the author quotes Marvells poetry primarily to illustrate (A) the contrast between imaginative and practical engineering (B) the solution to the mystery of flight (C) how the advantages of flight outweigh its dangers (D) how those who analyze the mechanics of flight overlook its beauty (E) humanitys deep longing to be able to fly

Casting the bodys vest aside My soul into the boughs does glide: There, like a Bird, it sits and sings. Then whets and combs its silver wings, And, till prepared for longer flight. Waves in its plumes the various light.

At the beginning of this century, the new light engineering that allowed people to fly seemed to the uninitiated a kind of poetry. In 1913, a writer in the Atlantic Monthly claimed that machinery is our new art form and praised the engineers whose poetry is too deep 640to look poetic and whose gifts have swung their souls free ... like gods. One of Wrights most eloquent admirers called him a poet and compared him to one of those monks of Asia Minor who live perched on the tops of inaccessible mountain peaks. The soul of Wilbur Wright 645is just as high and faraway. Wright was, in fact, deeply middle-class and unheroic, writes one biographer, but those obsessed with the glamour of flight pretended not to notice.
635

25
650
23. The quotation in lines 41-42 (the engineers ... poetic) serves to reinforce the point that (A) machines can be as inspiring as works of art (B) technology and poetry are both misunderstood (C) scientific practicality is more important than artistic creativity (D) the technical language of engineers has a lyrical quality (E) artistic pretensions are not suitable for engineers
715

24. In lines 47-48, the inclusion of the biographers remarks is intended to (A) (B) (C) (D) (E) criticize an instance of unimaginative thinking demystify the image of an individual reiterate a generally accepted view reassess the importance of an invention perpetuate the legacy of a scientific hero

655

720

660

665

670

675

680

685

690

695

700

705

710

SECTION 7

WRITING

Time - 25 minutes 35 Questions


725 Directions: For each question in this section, select the best answer from among the choices given and fill in the corresponding circle on the answer sheet.

The following sentences test correctness and effectiveness of expression. Part of each sentence or the entire sentence is underlined; beneath each sentence are five ways of phrasing the underlined material. Choice A repeats the original phrasing; the other four choices are different. If you think the original phrasing produces a better sentence than any of the alternatives, select choice A; if not, select one of the other choices. In making your selection, follow the requirements of standard written English; that is, pay attention to grammar, choice of words, sentence construction, and punctuation. Your selection should result in the most effective sentence clear and precise, without awkwardness or ambiguity. EXAMPLE: Laura Ingalls Wilder published her first book and she was sixty-five years old then. (A) and she was sixty-five years old then (B) when she was sixty-five (C) at age sixty-five years old (D) upon the reaching of sixty-five years (E) at the time when she was sixty-five

750

2. Burdened with three pieces of luggage and a pair of skis, Sarahs search for a baggage cart was desperate. (A) Sarahs search for a baggage cart was desperate (B) Sarahs desperate search was for a baggage cart (C) a baggage cart was what Sarah desperately searched for (D) a baggage cart for which Sarah desperately searched (E) Sarah searched desperately for a baggage cart 3. Karen, James, and Sam were hiking when, stumbling over a rock, he fell down a steep embankment. (A) when, stumbling over a rock, he fell down a steep embankment (B) and then he fell down a steep embankment after he stumbled over a rock (C) when Sam fell down a steep embankment after stumbling over a rock (D) when Sam fell down a steep embankment, since he stumbled over a: rock (E) and, since Sam has stumbled over a rock, he fell down a steep embankment 4. By attracting new industry when tile old factory closed, the council kept the economy of the town from collapsing, this was a disaster many workers had feared. (A) this was a disaster many workers had feared (B) because many workers had feared a disaster (C) the fear many workers had would be a disaster (D) a disaster that many workers had feared (E) it was feared by many workers as a disaster

755

760

765

770

775 7301. Roger had just walked into his office and that was when he was told that his plan had finally been approved.

735

(A) (B) (C) (D) (E)

and that was when he was told and then he learned when it was learned by him and then they told him when he learned

780

740

7855. A healthy economy can be measured not only by the growth of businesses but it has a psychological effect on people.

790 745

(A) (B) (C) (D) (E)

it has a as well in the also by the also the in the way of having a

855 795

6. Todays political candidates may reach wide audiences by appearing on television. but old-fashioned barnstorming still has value because it allows the electorate to meet candidates face to face. (A) television, but old-fashioned barnstorming still has value because it allows (B) television, but old-fashioned barnstorming still would have value because of allowing (C) television; however, there is still value in old-fashioned barnstorming by allowing (D) television, old-fashioned barnstorming still having value because it allows (E) television, when old-fashioned barnstorming still has value in allowing 7. Linguistic research often requires fieldwork where they can study and record the spoken dialects of a region. (A) (B) (C) (D) (E) where they through which they and the linguist during which the linguist which they

9. George Orwells term doublespeak referring to the intentional use of language to confuse or to mislead, as when one says revenue enhancement instead of 860 tax increase. (A) (B) (C) (D) (E) referring to the intentional use of language referring to language which is intentionally used which refers to intentionally using language refers to the intentional use of language is when it refers to language used intentionally

800

865

805

10. Scientists predict technological changes in the next century, they will be as dramatic as was the development of the transcontinental railroad in the last 870 century. (A) (B) (C) (D) (E) century, they will be as dramatic as was century, these will be as dramatic as century; being as dramatic as was century will be dramatic as is century as dramatic as

810

875

815

11. With billions of tons yet to be mined, some argue that coal conservation measures are unnecessary.
880

8. The primatologist has argued that sustained observation 820 of a few animals provides better behavioral data than does intermittent observation of many animals. (A) provides better behavioral data than does intermittent observation of many animals (B) provides better behavioral data than many animals are observed intermittently (C) providing better behavioral data than does intermittent observation of many animals (D) do provide better behavioral data than intermittent observation of many animals do (E) in contrast to intermittent observation of many animals, provides better behavioral data

(A) (B) (C) (D) (E)

With billions of tons Because billions of tons of coal are Because of coal in billions of tons By considering that there are billions of tons Aware of the coal in billions of tons

825

885

830

835

840

845

850

30

920

The following sentences test your ability to recognize grammar and usage errors. Each sentence contains either a single error or no error at all. No sentence contains more than one error. The error, if there is one, is underlined and lettered. If the sentence contains an error, select the one underlined part that must be changed to make the sentence correct. If the sentence is correct, select choice E. In choosing answers, follow the requirements of standard written English. EXAMPLE: The other delegates and him immediately A B C accepted the resolution drafted by the D neutral states. No error E

15. Cocoa was popular with Europeans before either tea A and coffee, its consumption gradually spreading from
925

Spain and Portugal to Italy, Austria, France. and then across the channel to the British Isles. D
930

No error E 16. To become a world figure-skating champion like A

935

Kristi Yamaguchi, one must be so dedicated that you B will practice six hours a day. No error C E

890
12. Beatrix Potter completely transformed the A traditional animal fable, and they had been B
895

94017. Each time Caroline turns on her computer, she

C has to enter a company code, then her initials, . A and then enters a password before she can
945

used by other writers simply to illustrate D moral lessons. No error E

B begin working. No error D E

90013. No matter where they came from or what their

A C
905

B 18. A talented and versatile artist, Twyla Tharp


950

previous lifestyle is, the refugees were grateful for D having been granted political asylum in the United States. No error E 14. Susan and Peter were inspired to become
910

A has been a dancer, choreographer, and B C D E collaborated on various productions. No error

955

A a professional writer after hearing a famous journalist B D C speak about the challenges of investigative reporting.

915

No error E

19. The scientific writings of Edward O. Wilson,

23. The governors aides are convinced that A

960
Stephen Jay Gould, and Richard Dawkins, which A has continued the discussion of genetic issues B

1005

the announcement of the investigation, coming just days before the filing deadline, B were calculated to discourage the governor

965

raised by Charles Darwin, are familiar to many C D E

1010
D

C from running for reelection. No error E

high school and college students. No error

97020. Conflicts between land developers and conserva1015


tionists have repeatedly arose, causing Congress A B to reconsider legislation that prohibits building

24. Although the new device was the most clever A B designed bird feeder that Ms. Rodriguez had ever owned, it could not keep squirrels from stealing C D E 25. Whatever price the company finally sets for A

975
D

C within habitats of endangered species. No error E

1020

the birdseed. No error

21. Surely one of the most far-reaching changes in the

980

A nineteenth century will be the change from working B at home to working in the factory. No error C D E No error 22. Howard Gardner, an observer of Chinese elementary A education, has questioned the view that requiring

1025

the fuel will probably be determined as much by B D C politics as by a realistic appraisal of the market.

985 1030

990

young children to copy models prevents them from B D C E becoming a creative artist later in life. No error

995

1000

35
103526. Air pollution caused by industrial fumes 1080 Directions: The following passage is an early draft of an essay. Some parts of the passage need to be rewritten.

has been studied for years, but only recently A


1040

has the harmful effects of noise pollution C become known. No error D E

Read the passage and select the best answers for the questions that follow. Some questions are about particular sentences or parts of sentences and ask you to improve sentence structure or word choice. Other questions ask you to consider organization and development. In choosing answers, follow the requirements of standard written English. Questions 30-35 are based on the following passage. (1) Many people complain about the negative state1085ments made by candidates that are arising during political campaigns. (2) But really, what candidate is ever going to say something nice about an opponent? (3) Their goal, after all, is if you elect them. (4) Clearly, there are times when negatives must be mentioned. (5) For example, it is 1090only fair for a challenger to point out that an incumbent has in fact done a poor job in office. (6) Now that I am almost old enough to vote, I pay more attention to the character of candidates. (7) But there is another kind of negative campaigning, 1095the kind known as sleaze. (8) Instead of proposing new policies, a sleazy candidate will run a campaign aimed at smearing the opponent. (9) It has become so common that it is almost taken for granted. (10) The dirtiest kinds of campaigns use tactics such 1100as character assassination and outright lying about an opponent. (11) The actual work of planting the lies is often done by campaign staff. (12) Then the accusing candidate denies knowing about it. (13) Meanwhile, someones reputation is ruined because people who hear 1105the lies believe them without checking the facts first. (14) The media report the lies, they say it is because they are newsworthy. (15) Thus the media contribute to a vicious circle.
111030. In context, which of the following revisions is necessary in sentence 1 (reproduced below) ?

27. The historian argued that we ought to learn


1045

A more about the process by which individuals B like Sam Houston were identified by others C

1050

as leaders. No error D E

28. Quick to take advantage of Melanie Johnsons A


1055

B C

preoccupation in the history of the Johnson family, the genealogist proposed investigating that history for D a large fee. No error

1060

E 29. Contrasting with most other fifteenth-century rulers, A B C aristocracy in any overseas ventures. No error D E Portuguese kings could count on the support of the

1065

Many people complain about the negative statements made by candidates that are arising during political campaigns.
1115

1070

(A) (B) (C) (D) (E)

Delete people. Change complain to complained. Change are arising to is raised. Delete that are arising. Insert the course of after during.

1120 1075

1125

31. In context, which is the best version of sentence 3 (reproduced below) ? Their goal, after all, is if you elect them. (A) (As it is now) (B) Their goal, after all, would be if their opponent lost. (C) A political candidates goal. after all, is when the election is won. (D) The goal of political candidates, after all, is to win elections. (E) The goal of politics, after all, is for you to elect this person. 32. In context, which of the following most logically replaces It in sentence 9 (reproduced below) ?
1185

Which is the best way to deal with sentence 14 (reproduced below) ? The media report the lies, they say it is because they are newsworthy. (A) (B) (C) (D) (E) Leave it as it is. Delete it. Change report to verify. Change they say its because to saying that. Change they are newsworthy to it is news.

1130

1190

1135

35. Which of the following is best to add after sentence 15 as a concluding sentence?
1195

1140

It has become so common that it is almost taken for granted. (A) (B) (C) (D) (E) This strategy This lack of planning This complaint This lie This promise
1200

1145

(A) These tactics may be unnecessary, but they do have a bright side after all. (B) Restrictions such as this, if rigorously enforced, will control negative campaigning. (C) In conclusion, the media should refuse to participate in it. (D) Therefore, as much as political campaigns cost, we deserve better. (E) This practice only worsens the negative aspects of our political campaigns.

1205

33. What should be done with sentence 6 (reproduced below) ?


1150

Now that I am almost old enough to vote. I pay more attention to the character of candidates. (A) (B) (C) (D) (E) Leave it as it is. Delete it. Insert Consequently, at the beginning. Add than I formerly did at the end. Rephrase the sentence and begin with Shouldnt I pay.

1155

1160

1165

1170

1175

1180

SECTION 8

CRITICAL READING
19 Questions

Time - 20 minutes

1210 Directions: For each question in this section, select the best answer from among the choices given and fill in the corresponding circle on the answer sheet.

1215
Each sentence below has one or two blanks, each blank indicating that something has been omitted. Beneath the sentence are five words or sets of words labeled A through E. Choose the word or set of words that, when inserted in the sentence, best fits the meaning of the sentence as a whole. Example: Hoping to ------- the dispute, negotiators proposed a compromise that they felt would be ------- to both labor and management. (A) (B) (C) (D) (E) enforce .. useful end .. divisive overcome .. unattractive extend .. satisfactory resolve .. acceptable
1245

3. Although Eudora Welty and William Faulkner wrote in distinctively different styles, ------- between the two is ------- because they both lived in and wrote about Mississippi. (A) (B) (C) (D) (E) comparison .. inevitable cooperation .. destructive discord .. legendary similarity .. unlikely rivalry .. redundant

1250

1255

4. Cito Gaston, one of the least ------- baseball managers, surprised reporters by weeping openly after his team won the play-offs. (A) somber (B) demonstrative (D) bountiful (E) wistful (C) insufferable

12605. That critics writing is so obscure and dense that upon first reading, one finds its ------- hard to penetrate.

1. A swindlers ------- is usually a gullible person who is unable to resist the swindlers traps.
1220

(A) brevity (B) rigidity (C) floridity (D) harmony (E) opacity
12656. Oil companies seeking permission to drill in Alaskan wildlife refuge areas argued that, for animals. the effects of previous drilling in comparable areas have been -------. 1270

(A) peer (B) ally (C) prey (D) nemesis (E) superior 2. Improvement~ in refrigeration and transportation in the nineteenth century ------- the ------- of available food for many families in the United States.

1225

(A) (B) (C) (D) (E)

slowed .. distribution accelerated .. perishability expanded .. variety lowered .. amount created .. dearth

(A) irrepressible (B) counterproductive (C) negligible (D) momentous (E) magnanimous

1230

1235

1240

40

The two passages below are followed by questions based on their content and on the relationship between the two passages. Answer the questions on the basis of what is stated or implied in the passages and in any introductory material that may be provided.

1275
Questions 7-19 are based on the following passages. The narrator of Passage 1 describes the behavior of his friend Jerry, with whom he is rooming in an unspecified African country. In Passage 2, a different narrator describes himself while visiting an English couple in London. Both fictional works were published in the early 1980s. Passage 1 Jerry was deceitful, but at the time I did not think he was imaginative enough to do any damage. And yet his was not the conventional double life that most White people led in Africa. Jerry had certain ambitions: ambition makes 1280more liars than egotism does. But Jerry was so careful, his lies such modest calculations, that he was always believed. He said he was from Boston, Belmont actually, he told me. when I said I was from Medford. His passport said Watertown. He felt he had to conceal it. That explained 1285a lot: the insecurity of living on the lower slopes of the long hill. between the smoldering steeples of Boston and the clean, high-priced air of Belmont. We are probably no more class-conscious than the British, but when we make class an issue. it seems more than snobbery, It becomes 1290a bizarre spectacle, a kind of attention-seeking, and I cannot hear an American speaking of his or her social position without thinking of a human fly, one. of those tiny people in grubby capes whom one sometimes sees clinging to the brickwork of a tall building. 1295 What had begun as fantasy had, after six months of his repeating it in our insignificant place, made it seem like fact. I had the impression that it was one of the reasons Jerry wanted to stay in Africa. If you tell enough lies about yourself, they take hold. It becomes impossible ever to go back, 1300since that means facing the truth. In Africa, no one could dispute what Jerry. said he was: a wealthy Bostonian, from a family of some distinction, adventuring in philanthropy before inheriting his fathers business. Passage 2 Anna and Chris made me at ease the first day in their 1305polished living room-though I was not sure why these people would bother putting themselves out for me at all. And when they kept inviting me back for dinner parties and extending their hospitality, I wondered if maybe they were bored, or if their ignorance of American types was 1310such that they failed to see that I was not at all of their social class: I kept expecting some crude regional expression to betray me; and, once I thought of it in those terms, I knew I would have to make sure they saw that side of me-to do less would be like trying to pass. Yet whatever I said

1315seemed to make no difference in their acceptance. I then


suspected that my rough-edgedness itself was entertaining to them as a source of vitality, their diversion-of-the-month. This would have made more sense if the Hodgkinsons were bored, dried-up people who needed to feast on any new 1320stranger, but they were not; they were in the world and leading stimulating lives and I finally had to come to the anxious conclusion that they simply liked me. The truth was I had changed, though I was perhaps the last to see it. While still feeling myself a child from 1325the slums; I had gotten a university education, acquired a taste for esoteric culture; and now, when I thought back to my students in East Harlem, where I felt I should really belong, it seemed that I was a stranger there as well. Yet I did not fit in with people born to middle-class comfort either. 1330It seemed there was no group at all in which I could feel at home. Perhaps anyone with the tiniest sensitivity comes to that banal conclusion. But what I was seeing now with horror, in the accepting eyes of those a class above me, was that I had already partly metamorphosed into them, My only 1335hope of growing seemed to point in an upward social direction; but that direction aroused in me a characteristic disapproval and distaste. I was by no means attracted by everything I saw in well-off peoples lives, and the momentary need to accept their hospitality and keep secret my criticism of them 1340made me feel like a hypocrite. 7. Jerry in Passage 1 and the narrator of Passage 2 are similar in that both (A) (B) (C) (D) feel a strong desire to advance socially feel insecurity about their backgrounds are unsuccessful in deceiving others are determined to remain genuine in the face of pressure to conform (E) have been unduly influenced by the lifestyles of their friends 8. Jerry differs most from the narrator of Passage 2 in his (A) apparent satisfaction with his present circumstances (B) ability to differentiate fantasy from reality (C) willingness to devote his time to philanthropic concerns (D) refusal to accept the labels and judgments of others (E) eagerness to befriend people of all social and economic classes

1400

9. The first sentence of Passage 1 implies that


1345

(A) (B) (C) (D) (E)

the truth can sometimes be more damaging than a lie the narrator failed to recognize Jerrys deceptive nature the narrator is intolerant of Jerrys background the narrators view of Jerry changed over time Jerry was unaware of his effect on others

15. The statement in lines 44-45 (to feast ... stranger) suggests that some hosts
1405

135010. In line 6, modest most nearly means

1355

(A) (B) (C) (D) (E)

shy self-conscious secretive decent moderate

1410

(A) resent being relied on for the latest gossip (B) are anxious about making a good impression on strangers (C) get immense satisfaction from making their guests feel inferior (D) pretend to lead m.ore interesting lives than they actually do (E) live vicariously through their guests 16. In line 45, the phrase in the world indicates that the Hodgkinsons are

1415

11. In the context .of Passage 1, insignificant (line 21) suggests that
1360

1365

(A) Jerrys lying is unlikely to have major consequences in Africa (B) Jerry does not realize how commonplace his behavior is in Africa (C) Jerry has lost the ability to distinguish between reality and fantasy (D) the narrators .own reputation has been harmed by association with Jerry (E) the narrator believes Jerrys behavior is silly 12. Passage 1 indicates that Jerry feels as he does about his life in Africa because (A) the inhabitants cannot easily verify his American social status (B) the inhabitants will not give him the social acceptance that he craves (C) he was treated with the same respect as when he was in America (D) he is free from the constraints .of family and social obligations (E) he is free to befriend people .of varied social backgrounds 13. The two passages differ in that, unlike Jerry, the narrator of Passage 2 has

(A) (B) (C) (D) (E)

preoccupied with the mundane aspects of life familiar with upper-class social conventions suspicious of spirituality stylish and urbane, but ruthless in contact with interesting people and ideas

1420

17. In line 47, anxious most nearly means (A) (B) (C) (D) (E) meticulous impatient uneasy frightened eager

1425

1370

18. Which best characterizes how the subject of identity is treated in these two passages? (A) Passage 1 suggests that identity can be self-created, while Passage 2 contends that it is determined by external and internal factors. (B) Passage 1 de-emphasizes the importance of ancestral background to ones identity, while 1435 Passage 2 emphasizes its importance. (C) Passage 1 argues that the individual chooses his or her identity, while Passage 2 affirms that identity is imposed by others. (D) Both passages downplay the impact of ones 1440 physical surroundings on ones identity. (E) Neither Passage 1 nor Passage 2 considers the psychological effect of denying parts of ones identity.
1430 144519. Which generalization about class attitudes is most strongly supported by both passages?

1375

1380

1385

1390

(A) reluctantly decided to return to the United States (B) found that social advancement is frequently impossible to .obtain (C) belatedly rediscovered his l.ove for his childhood home (D) undergone a change in attitude about social class (E) recently stopped lying about his background 14. In lines 36-39 .of Passage 2, the narrators perspective changes from

1450

1395

(A) (B) (C) (D) (E)

suspicion of his hosts to .outright mistrust .of them estrangement to a sense .of camaraderie insecurity to feelings .of despondency apprehensiveness to a desire to reveal himself rejection of his social status to an acceptance of it

1455

(A) Charm and personality are more important than ones social position. (B) Only the very wealthy are concerned with social position. (C) It is only after having lived abroad that Americans come to believe in the possibility of a society without class distinctions. (D) Americans choose to live abroad primarily to escape the confinement of social class. (E) Even when living abroad, Americans consider their status in American society crucial to individual identity.

45 1460 SECTION 10 WRITING

Time -10 minutes 14 Question


Directions: For each question in this section, select the best answer from among the choices given and fill in the corresponding circle on the answer sheet.

1465

The following sentences test correctness and effectiveness of expression. Part of each sentence or the entire sentence is underlined; beneath each sentence are five ways of phrasing the underlined material. Choice A repeats the original phrasing; the other four choices are different. If you think the original phrasing produces a better sentence than any of the alternatives, select choice A; if not, select one of the other choices. In making your selection, follow the requirements of standard written English; that is, pay attention to grammar, choice of words, sentence construction, and punctuation. Your selection should result in the most effective sentence clear and precise, without awkwardness or ambiguity. EXAMPLE: Laura Ingalls Wilder published her first book and she was sixty-five years old then. (A) and she was sixty-five years old then (B) when she was sixty-five (C) at age sixty-five years old (D) upon the reaching of sixty-five years (E) at the time when she was sixty-five

2. As patients, the medical directors of the clinic believe that you are entitled to know the reason for the increase 1490 in fees. (A) As patients, the medical directors of the clinic believe that you (B) The belief of the clinics medical directors about patients is that you (C) You, as patients, are believed by the clinical medical directors, and you (D) The medical directors of the clinic, who believe that you, as patients, (E) The medical directors of the clinic believe that as patients, you 3. Lecturing at the university, read the poetry of Margaret Atwood was the advice Professor Clark gave her audience.
1505

1495

1500

1510

(A) read the poetry of Margaret Atwood was the advice Professor Clark gave her audience (B) the poetry of Margaret Atwood was what Professor Clark advised her audience to read (C) her audience was advised by Professor Clark to read the poetry of Margaret Atwood (D) Margaret Atwoods poetry, advised Professor Clark, was what her audience should read (E) Professor Clark advised her audience to read the poetry of Margaret Atwood 4. Lois has learned more about Arna Bontemps writings than the rest of us because of being her favorite author. (A) us because of being her favorite author (B) us; this is the result of Bontemps being her favorite author (C) us because Bontemps is her favorite author (D) us as a result of Bontemps being her favorite author (E) us since Bontemps is her favorite as an author

1515

1. Some of the Smithsonian Institutions most prized 1470 items, from Duke Ellingtons musical transcripts to First Ladies gowns, coming from unsolicited donations. (A) (B) (C) (D) (E) coming from they come from they have come from came from which came from

1520

1475

1525

1480

1485

1530

5. The five autobiographical volumes by Maya Angelou begin with her childhood in Arkansas and culminate in her adult years in Egypt and Ghana. (A) begin with her childhood in Arkansas and culminate (B) that begin with her childhood in Arkansas and culminate (C) have begun with her childhood in Arkansas and culminating (D) beginning with her childhood in Arkansas and culminating (E) are begun with her childhood in Arkansas and culminated

1590

9. Trees are able to collect large amounts of water from fog in some areas as much as thirty inches annually. (A) in some areas as much as thirty inches annually (B) in some areas having thirty inches per year (C) in some places collecting about thirty inches per year annually (D) collecting the equal of thirty inches annually in some places (E) which in some areas amounts to thirty inches collected annually 10. Prized for their rarity, gourmets will spend a small fortune on wild truffles rather than settle for common mushrooms.

1535

1595

1540

1600

15456. Rilke, the great German poet, could not continue his search for angelic spirits until he can rely on a strength greater than his own.

1605

1550

(A) can rely on a strength greater than his own (B) could rely on a strength greater than his own (C) would be able to rely on a strength greater than his own (D) can rely on a strength greater than his strength (E) could rely on a strength greater than his strength

1610

15557. To ensure that the bread will have the same consistency from batch to batch, it is the quality control specialist who checks small random samples of dough from each lot. 1560

1615

1565

(A) it is the quality control specialist who checks small random samples of dough from each lot (B) the quality control specialist checks small random samples of dough from each lot (C) small random samples of dough being checked from each lot by the quality control specialist (D) the quality control specialist checks samples of dough small and randomly from each lot (E) the quality control specialist is the one checking small random samples from each lot of dough

(A) Prized for their rarity, gourmets will spend a small fortune on wild truffles rather than settle for common mushrooms. (B) Prized as rare, gourmets will spend a small fortune on wild truffles as opposed to settling for common mushrooms. (C) Prized for their rarity, wild truffles command a small fortune among gourmets unwilling to settle for common mushrooms. (D) As prized for rarity, wild truffles, being costly, command a small fortune for gourmets unwilling to settle for common mushrooms. (E) Wild truffles prized for their rarity by gourmets who will spend a small fortune but not to settle for common mushrooms. 11. Evidence from surveys and interviews show friendships made in high school tend to last longer than those made in college. (A) show friendships made in high school tend to last (B) show high school friendships that tend to last (C) is showing high school friendships tending to last (D) shows that friendships made in high school tend to last (E) shows friendships in high school tends to last

1620

1625

15708. Surface mining is safer, quicker, and cheaper than deep mining, but the greater is its toll in human misery.

1630

1575

(A) (B) (C) (D) (E)

the greater is its toll in human misery it has a greater human misery toll in its human misery toll it is greater there is the greater toll in human misery its toll in human misery is greater

1580

1585

163512. Growing up in a family where music was a daily part of life, Steve and Rick shared a determination to become singing duos known nationwide.

1695

1640

(A) (B) (C) (D) (E)

to become singing duos to become a singing duo of becoming singing duos that they would become singing duos of becoming a singing duo

14. Jacob Lawrence is best known for his depictions of modem urban life, and his celebrated painting Forward presents a rural scene from the life of abolitionist Harriet Tubman. (A) Jacob Lawrence is best known for his depictions of modem urban life, and (B) Jacob Lawrence is best known for his depictions of modem urban life, (C) Jacob Lawrence is best known for his depictions of modern urban life, however (D) Although Jacob Lawrence is best known for his depictions of modem urban life, (E) Inasmuch as Jacob Lawrence is best known for his depictions of modem urban life,

1700

1645

13. Before reading the front page of the newspaper, my sister reads the sports section, my brother reads the comics first. (A) Before reading the front page of the newspaper, my sister reads the sports section, my brother reads the comics first. (B) My sister reads the sports section before reading the front page of the newspaper and my brother, he reads the comics first. (C) Before reading the front page of the newspaper, my sister reads the sports section; my brother reads the comics first. (D) My brother reads the comics first with my sister reading the sports section before reading the front page of the newspaper. (E) Before reading the front page of the newspaper, my sister reads the sports section; my brother reading the comics first.

1705

1710

1650

1655

1660

1665

1670

1675

1680

1685

1690

50

Practice Test 2 SECTION 1 WRITING ESSAY

Time 25 minutes
1715

The essay gives you an opportunity to show how effectively you can develop and express ideas. You should, therefore, take care to develop your point of view, present your ideas logically and clearly, and use language precisely.

1720Your essay must be written on the lines provided on your answer sheet you will receive no other paper on which to write. You will have enough space if you write on every line, avoid wide margins, and keep your handwriting to a reasonable size. Remember that people who are not familiar with your handwriting will read what you write. Try to write or print so that what you are writing is legible to those readers. 1725

You have twenty-five minutes to write an essay on the topic assigned below. DO NOT WRITE ON ANOTHER TOPIC. AN OFF-TOPIC ESSAY WILL RECEIVE A SCORE OF ZERO.

1730Think carefully about the issue presented in the following quotations and the assignment below.

1735

Technology promises to make our lives easier, freeing up time for leisure pursuits. But the rapid pace of technological innovation and the split second processing capabilities of computers that can work virtually nonstop have made all of us feel rushed. We have adopted the relentless pace of the very machines that were supposed to simplify our lives, with the result that, whether at work or play, people do not feel like their lives have changed for the better. Adapted from Karen Finucan, Life in the Fast Lane

1740

Assignment:
1745

Do changes that make our lives easier not necessarily make them better? Plan and write an essay in which you develop your point of view on this issue. Support your position with reasoning and examples taken from your reading, studies, experience, or observations.

1750

Practice Test 2 SECTION 2


1755

CRITICAL READING
24 Questions

Time 25 minutes

Directions: For each question in this section, select the best answer from among the choices given and fill in the corresponding circle on the answer sheet.

1760
Each sentence below has one or two blanks, each blank indicating that something has been omitted. Beneath the sentence are five words or sets of words labeled A through E. Choose the word or set of words that, when inserted in the sentence, best fits the meaning of the sentence as a whole. Example: Hoping to ------- the dispute, negotiators proposed a compromise that they felt would be ------- to both labor and management. (A) (B) (C) (D) (E) enforce .. useful end .. divisive overcome .. unattractive extend .. satisfactory resolve .. acceptable

17904. Maggie is a procrastinator, naturally inclined to ------and to ------- discussions. (A) meddle .. scoff at (B) temporize .. prolong (C) misbehave .. disrupt 1795 (D) sneer .. terminate (E) withdraw .. intrude in

5. Just as glass windows offer buildings both light and insulation, certain atmospheric gases ------- incoming 1800 sunlight and ------- heat radiated from the ground, preventing warmth from escaping. (A) (B) (C) (D) (E) conduct .. release deflect .. transmit admit .. contain absorb .. dispense resist .. trap

1805

1. To avoid being -------, composer Stephen Sondheim strives for an element of surprise in his songs.
1765

6. The speaker, praised for her style yet ridiculed for her vacuity, often moved naive listeners with ------- alone 1810 and led them to believe that her speech had -------. (A) (B) (C) (D) (E) reason .. dalliance infelicity .. conviction rhetoric .. substance pragmatism .. futility boorishness .. integrity

(A) erratic (B) informal (C) elaborate (D) predictable (E) idiosyncratic 2. Because the pandas had already been weakened by disease and drought, a harsh winter would have had ------- consequences for them.

1815

1770

(A) preventive (D) unforeseen

(B) regressive (E) moderate

(C) catastrophic

7. The actor was noted for his ------- behavior: he quickly became irritated if his every whim was not immediately satisfied.
1820

3. For many of the villagers, marriage was a practical -------, one not necessarily ------- of love but nevertheless 1775 grounded largely in economic advantage. (A) (B) (C) (D) (E) arrangement .. devoid entertainment .. disparaging attitude .. consisting bargain .. worthy misfortune .. trusting

(A) fastidious (D) petulant

(B) sedulous (E) mercenary

(C) vindictive

8. Hayley Millss films have been called -------, although most of them are not so sentimental as to deserve that 1825 description. (A) treacly (B) cursory (C) prosaic (D) meticulous (E) consecrated

1780

1785

1830

55

The passages below are followed by questions based on their content; questions following a pair of related passages may also be based on the relationship between the paired passages. Answer the questions on the basis of what is stated or implied in the passages and in any introductory material that may be provided.

1835

Questions 9-10 are based on the following passage. That nineteenth-century French novelist Honor de Balzac could be financially wise in his fiction while losing all his money in life was an irony duplicated in 1840other matters. For instance, the very women who had been drawn to him by the penetrating intuition of the female heart that he showed in his novels were appalled to discover how insensitive and awkward the real man could be. It seems the true source of creation 1845for Balzac was not sensitivity but imagination. Balzacs fiction originally sprang from an intuition he first discovered as a wretched little school boy locked in a dark closet of his boarding school: life is a prison, and only imagination can open its doors. fianc 9. The example in lines 4-8 primarily suggests that (A) Balzacs work was not especially popular among female readers (B) Balzac could not write convincingly about financial matters (C) Balzacs insights into character were not evident in his everyday life (D) people who knew Balzac personally could not respect him as an artist (E) readers had unreasonable expectations of Balzac the man 10. The author mentions Balzacs experience as a schoolboy in order to (A) explain why Balzac was unable to conduct his financial affairs properly (B) point out a possible source of Balzacs powelful imagination (C) exonerate the boarding school for Balzacs lackluster performance . (D) foster the impression that Balzac was an unruly student (E) depict the conditions of boarding school life during Balzacs youth 11. The passage suggests that Jane Wrights medical training was made more difficult because (A) (B) (C) (D) (E) her father warned her not to study medicine her father flaunted his success she did not spend adequate time studying she shared her fathers desire for fame she was inevitably compared to her father

Questions 1112 are based on the following passage. Dr. Jane Wright insisted in later years that her father, surgeon Louis Wright, never pressured her to study medicine; indeed he warned her how hard becoming a doctor would be. His very fame, within and beyond the African American community, made 1855her training harder in some ways. His being so good really makes it very difficult, Wright told an interviewer soon after she graduated from medical school in 1945. Everyone knows who Papa is.
1850

12. The passage is primarily concerned with Jane Wrights (A) views of the medical profession (B) childhood recollections (C) perception of her father as a role model (D) reluctance to collaborate with her father (E) gratitude for her fathers encouragement

Questions 13-24 are based on the following passages. The following two passages consider the experiences of middle-class women in nineteenth-century England under the reign of Queen Victoria (1837-1901). Passage 1 is from a work of social history; Passage 2 is from a study of travel writing. Passage 1 In nineteenth-century England, middle-class women 1860were usually assigned domestic roles and faced severely limited professional career options. Of course, one can point to Englands monarch, Queen Victoria, as a famous example of a woman at work, and millions of workingclass women worked for wages in factories and private 1865homes on farms and in stores and markets. But aristocrats were often exempt from societal strictures that bound the middle class. and working-class women were usually looked down on as not being respectable for their efforts as workers. As the nineteenth century progressed, it was 1870assumed that a woman engaged in business was a woman without either her own inheritance or a man to support her. Middle-class women already shared with upper-middleclass men the societal stumbling blocks to active pursuit of business, which included the feeling that labor was 1875demeaning and not suitable for those with aspirations to gentility. But unlike a man, whose self-worth rose through his economic exertions, a woman who did likewise risked opprobrium for herself and possibly shame for those around her. Inequality in the working world made it exceedingly 1880difficult for a middle-class woman to support herself on her own, let alone support dependents. Thus, at a time when occupation was becoming a core element in masculine identity, any position for middle-class women other than in relation to men was considered anomalous. In the 1851 1885census, the Registrar General introduced a new fifth class of workers, exclusively made up of women: The fifth class comprises large numbers of the population that have no occupation; but it requires no argument to prove that the wife, the mother, the mistress of an English family fills offices and discharges duties of no ordinary importance; or that children are or should be occupied in filial or household duties, and in the task of education, either at home or at school.

Marriage became, more than ever, the only career option offering economic prosperity for women; in business, women appear only as faint shadows behind the scenes. The absence of women in business and financial records makes our knowledge of what middle-class women actually 1910did and how they survived economically quite fragmentary. What we do know is that womens ability to survive economically on their own became increasingly difficult in the course of the nineteenth century.
1905

Passage 2 In the second half of the nineteenth century in England, 1915under the rule of Queen Victoria, because of the long peace and the increasing prosperity, more and more women found themselves able to travel to Europe unescorted. With the increase in travel came an increase in the number of guidebooks, collections of travel hints, and diaries by 1920travellers many of which were written by or directed to women. Although nineteenth-century women traveled for a variety of reasons, ranging from a desire to do scientific research to involvement in missionary work, undoubtedly a major 1925incentive was the desire to escape from domestic confinement and the social restrictions imposed on the Victorian female in Britain. As Dorothy Middleton observes, Travel was an individual gesture of the housebound, mandominated Victorian woman. The caged birds of the 1930Victorian parlor found their wings and often took flight in other lands. In a less constrained environment they achieved physical and psychological freedom and some measure of autonomy. In Celebrated Women Travelers of the Nineteenth Century (1883), Davenport Adams comments: Fettered as 1935women are in European countries by restraints, obligations, and responsibilities, which are too often arbitrary and artificial ... it is natural enough that when the opportunity offers, they should hail even a temporary emancipation through travel. 1940 By the latter part of the nineteenth century, women travelers began to be singled out as exemplars of the new social and political freedom and prowess of women. Ironically, Mary Kingsley and other women travelers were opposed to or simply uninterested in the late Victorian 1945campaigns to extend womens political rights. Thus, when Mary Kingsley returned from West Africa in 1895, she was chagrined to discover that she was being hailed as a new woman because of her travels. Despite her often outspoken distaste for the new women agitating for greater 1950freedom, the travel books that she and others had written still suggested, as Paul Fussell has argued, an implicit celebration of freedom.

1890

This conception of women had been developing over a long 1895period. For example, in the late seventeenth century, trade tokens used by local shopkeepers and small masters in family businesses carried the initials of the mans and the womans first names and the couples surname, but by the late eighteenth century, only the initials of the male 1900proprietor were retained. This serves to confirm the view of one Victorian man, born in 1790, that whereas his mother had confidently joined in the family auctioneering business, the increased division of the sexes had seen the withdrawal of women from business life.

195513. Lines 18-21 suggest that for Victorian middle-class women, self-worth and economic exertions were thought to be

201518. In context, hail (line 80) most nearly means

1960

(A) (B) (C) (D) (E)

mutually exclusive constantly evolving the two keys to success essential to finding a husband easy to achieve

2020

(A) (B) (C) (D) (E)

call out to gesture to come from welcome summon

14. In line 24, occupation most nearly means


1965

19. In Passage 2, Mary Kingsleys attitude toward womens rights campaigns (lines 85-90) suggests
2025

(A) (B) (C) (D) (E)

military conquest pleasant diversion vocation settlement political repression

1970

15 . The author of Passage 1 considers trade tokens (lines 37-38) as evidence against the prevalence of a fifth class in the seventeenth century because they (A) served as legal currency (B) Were issued to both middle-class and workingclass women (C) helped neutralize gender stereotypes of the day (D) failed to identify women by their names and positions (E) identified men and women as partners in business 16. All of the following are referred to in Passage 1 as evidence of womens diminished social status in Victorian England EXCEPT the

2030

1975

(A) a single-minded dedication to equality between the sexes (B) a way in which dedication to one cause can lead to antagonism toward another (C) a striking inconsistency between her identity as a British citizen and her identity as a woman (D) an understanding of the link between womens struggle for freedom and the struggles of other groups (E) a contradiction between her personal motives and the way her actions are interpreted 20. According to Passage 2, nineteenth-century British women were motivated to travel by which of the following? 1. Educational pursuits II. Humanitarian concerns III. Entrepreneurial interests (A) (B) (C) (D) (E) I only III only I and II only I and III only II and III only

2035

1980

2040

1985

1990

(A) disparity between mens and womens career opportunities (B) shame risked by women who wished to enter commerce (C) exclusion of womens initials from trade tokens (D) influence of the queen (E) absence of financial records documenting womens activity 17. Which statement about British society, if true, would most directly support the view described in lines 42-46 ? . (A) Seventeenth-century women workers could raise their status by assuming greater responsibilities. (B) Women wrote more novels in the early nineteenth century than they did in the early eighteenth century. (C) Women and girls worked in factories throughout the nineteenth century. (D) The practice of married couples jointly running businesses died out in the early nineteenth century. (E) In the seventeenth century, formal academic institutions were closed to women.

2045

21. Which British traveler of the Victorian era would best illustrate the argument made in Passage 2 ?
2050

1995

2055

2000

2060

(A) A middle-class woman who tours Greece and Egypt to examine ancient ruins. (B) An aristocratic woman who lives in the Asian capital where her father is the British ambassador. (C) A young woman and her husband, both missionaries, who relocate permanently in a distant country. (D) A nursemaid who accompanies an aristocratic family to its new home in New York City. (E) A young girl from a poor family who is sent by relatives to make her fortune in Australia.

2005

2010

60

2125 2065

22. The fifth class (line 29) in Passage 1 is most like which group in Passage 2 ? (A) (B) (C) (D) (E) Women who worked as missionaries The caged birds (line 71) The new woman (lines 89-90) Dorothy Middleton and Mary Kingsley Davenport Adams and Paul Fussell
2130

24. The information in Passage 1 supports which assumption about the women described in Passage 2 ? (A) They were discouraged from pursuing careers in their native country. (B) They sought to establish new businesses in foreign countries. (C) They traveled with children and other family members. (D) They were universally admired by British women from every class of society. (E) They were committed advocates of social reform.

2070

23. Passage 1 and Passage 2 share a general tone of


2075

2135

(A) (B) (C) (D) (E)

affectionate nostalgia analytical detachment personal regret righteous indignation open hostility

2080

2085

2090

2095

2100

2105

2110

2115

2120

2140 SECTION 5 CRITICAL READING


24 Questions
Directions: For each question in this section, select the best answer from among the choices given and fill in the corresponding circle on the answer sheet.

Time - 25 minutes

2145
Each sentence below has one or two blanks, each blank indicating that something has been omitted. Beneath the sentence are five words or sets of words labeled A through E. Choose the word or set of words that, when inserted in the sentence, best fits the meaning of the sentence as a whole. Example: Hoping to ------- the dispute, negotiators proposed a compromise that they felt would be ------- to both labor and management. (A) (B) (C) (D) (E) enforce .. useful end .. divisive overcome .. unattractive extend .. satisfactory resolve .. acceptable

2180

3. Inbreeding can promote the expression of ------- genes, those that make an animal subject to disease or impair reproductive efficiency. (A) ineffable (B) articulated (C) consummate (D) presumptive (E) deleterious

21854. The doctor ------- so frequently on disease-prevention techniques that his colleagues accused him of -------.

2190

(A) (B) (C) (D) (E)

vacillated .. inconsistency sermonized .. fidelity wavered .. steadfastness experimented .. inflexibility relied .. negligence

2150

1. Predictably, detail-oriented workers are ------- keeping track of the myriad particulars of a situation. (A) remiss in (B) adept at (C) humorous about (D) hesitant about (E) contemptuous of

5. A judicious biography must be ------- representation that depicts both the strengths and the weaknesses of 2195 the subject, avoiding the two extremes of ------- and indictment. (A) (B) (C) (D) (E) a polarized .. vindication an imaginative .. discernment a holistic .. censure a complimentary .. animosity an equitable .. eulogy

2200

21552. The controversial tax fueled a sustained -----that could not be ------- by the Prime Ministers impassioned speeches.

2160

(A) (B) (C) (D) (E)

rebellion .. challenged interrogation .. fortified conflagration .. fostered denial .. restrained uprising .. quelled

2165

2170

2175

65
The passages below are followed by questions based on their content; questions following a pair of related passages may also be based on the relationship between the paired passages. Answer the questions on the basis of what is stated or implied in the passages and in any introductory material that may be provided.
2205

Questions 6-9 are based on the following passages. Passage 1 Farm families are able to achieve efficiency only through a brutal work schedule that few people could tolerate. The farm family does physically demanding work and highly stressful work at least 14 hours a day (often at least 18 hours a day during harvest season), 22157 days a week, 365 days a year, without a scheduled vacation or weekends off, wrote Minnesota politician and farm alumnus Darrell McKigney. The farmer must endure all of this without any of the benefits that most United States labor unions demand. A dairy farmer, for 2220instance, cannot just take off for a two-week vacation and not milk the cows. Farmers lose perspective on the other things in life, one psychologist has written. The farm literally consumes them.
2210

7. Both passages serve to discourage the (A) reliance on polls for accurate information (B) desire of many farmers to take annual vacations (C) tendency of Americans to buy inexpensive foods (D) romanticization of farm life by nonfarmers (E) rise in price of home-grown produce 8. The author of Passage 1 would most likely assert which of the following about the majority (line 19, Passage 2) ? (A) They would be bored by the routine chores that are performed on a farm. (B) They have little understanding of the realities of farm life. (C) They admire the efficiency of the average family farm. (D) They wish to improve the arduous life of many farmers. (E) They are impressed by the current research on economical food production. 9. Unlike the author of Passage 2, the author of Passage 1 does which of the following? (A) (B) (C) (D) (E) Explains a study. Offers a solution. Argues a position. Discusses a phenomenon. Quotes an authority.

Passage 2 Americans have distanced themselves from the 2225ethics and morals of food production, except where it serves them to think nostalgically about family farms as the source of our better values. Little wonder that a poll taken by The New York Times finds a majority of Americans seeing farm life as superior to any other 2230kind of life in this country. As consumers, Americans have enjoyed relatively inexpensive food. What will happen if family farms disappear? What will we do without family farmers to watch over the system for us, to be our dupes, and to create that pleasant situation 2235through their own great discomfort? 6. Unlike Passage 2, Passage 1 is primarily concerned with the (A) ethical implications of food production (B) harsh working conditions on many farms (C) need for farmers to form a labor union (D) plentiful and varied food available in the United States (E) beliefs of many Americans regarding farm life

Questions 1015 are based on the following passage. This excerpt from a novel by a Chinese American author is about a Chinese American woman named June. During a family dinner party attended by some of Junes Chinese American friends, Waverly, a tax attorney, discusses an advertisement that June wrote for her. Waverly laughed in a lighthearted way. I mean, really, June. And then she started in a deep television-announcer voice: Three benefits, three needs; three reasons to buy ... Satisfaction guaranteed . ... 2240 She said this in such a funny way that everybody thought it was a good joke and laughed. And then, to make matters worse, I heard my mother saying to Waverly: True, one cant teach style. June is not sophisticated like you. She must have been born this way. 2245 I was surprised at myself, how humiliated I felt. I had been outsmarted by Waverly once again, and now betrayed by my own mother. Five months ago, some time after the dinner, my mother gave me my lifes importance, a jade pendant on a gold 2250chain. The pendant was not a piece of jewelry I would have chosen for myself. It was almost the size of my little finger, a mottled green and white color, intricately carved. To me, the whole effect looked wrong: too large, too green, too garishly ornate. I stuffed the necklace in my lacquer box 2255and forgot about it. But these days, I think about my lifes importance. I wonder what it means, because my mother died three months ago, six days before my thirty-sixth birthday. And shes the only person I could have asked to tell me 2260about lifes importance, to help me understand my grief. I now wear that pendant every day. I think the carvings mean something, because shapes and details, which I never seem to notice until after theyre pointed out to me, always mean something to Chinese people. I know I could ask 2265Auntie Lindo, Auntie An-mei, or other Chinese friends, but I also know they would tell me a meaning that is different from what my mother intended. What if they tell me this curving line branching into three oval shapes is a pomegranate and that my mother was wishing me fertility 2270and posterity? What if my mother really meant the carvings were a branch of pears to give me purity and honesty? And because I think about this all the time, I always notice other people wearing these same jade pendants not the flat rectangular medallions or the round white 2275ones with holes in the middle but ones like mine, a twoinch oblong of bright apple green. Its as though we were all sworn to the same secret covenant, so secret we dont even know what we belong to. Last weekend, for example, I saw a bartender wearing one. As I fingered mine, I asked 2280him, Whered you get yours? My mother gave it to me, he said. I asked him why, which is a nosy question that only one Chinese person can ask another; in a crowd of Caucasians, two Chinese people are already like family. 2285 She gave it to me after I got divorced. I guess my mothers telling me Im still worth something. And I knew by the wonder in his voice that he had no idea what the pendant really meant.

10. In lines 1-4, Waverly characterizes Junes advertisement as being (A) (B) (C) (D) (E) unsophisticated and heavy-handed somber and convoluted clear and concise humorous and effective clever and lively

11. In the context of the passage, the statement I was surprised at myself (line 10) suggests that June (A) had been unaware of the extent of her emotional vulnerability (B) was exasperated that she allowed Waverly to embarrass her in public . (C) was amazed that she could dislike anyone so much (D) had not realized that her mother admired her friend Waverly (E) felt guilty about how much she resented her own mother. 12. Junes observation in lines 10-11 (I had ... again) suggests that (A) June had expected Waverly to insult her (B) June had hoped to embarrass Waverly this time (C) Waverly had a private understanding with Junes mother (D) Waverly had made June feel inadequate on previous occasions (E) Waverly was a more talented writer than June was 13. For June, a significant aspect of what happened at the dinner party is that (A) her mother had taken great pains to make Waverly feel welcome (B) her mother had criticized her for arguing with Waverly (C) her mother had sided against her in front of family and friends (D) Waverly had angered Junes mother (E) Waverly had lied to Junes mother 14. The description of Junes encounter with the bartender primarily serves to suggest that (A) the relationship of mother and son is different from that of mother and daughter (B) June is not the only one who ponders the meaning of a jade pendant (C) a jade pendant symbolizes the mystery of life and death (D) June finally understands the true meaning of her jade pendant (E) strangers are easier to talk to than family members and friends 15. The passage indicates that the act of giving a jade pendant can best be described as (A) (B) (C) (D) (E) a widely observed tradition a mothers plea for forgiveness an example of a mothers extravagance an unprecedented act of generosity an unremarkable event in Junes life

Questions 16-24 are based on the following passage. This passage is from a book of nature writing published in 1991. In North America, bats fall into mainly predictable 2290categories: they are nocturnal, eat insects: and are rather small. But winging through their lush, green-black world, tropical bats are more numerous and have more exotic habits than do temperate species. Some of them feed on nectar that bat-pollinated trees have evolved to profit from 2295their visits. Carnivorous bats like nothing better than a local frog, lizard, fish, or bird, which they pluck from the foliage or a moonlit pond. Of course, some bats are vampires and dine on blood. In the movies, vampires are rather showy, theatrical types, but vampire bats rely on stealth and small, 2300pinprick incisions made by razory, triangular front teeth. Sleeping livestock are their usual victims, and they take care not to wake them. First, they make the classic incisions shaped like quotation marks; then, with saliva full of anticoagulants so that the victims blood will flow nicely, they 2305quietly lap their fill. Because this anticoagulant is not toxic to humans, vampire bats may one day play an important role in the treatment of heart patients that is, if we can just get over our phobia about them. Having studied them intimately, I now know that bats are sweet-tempered, useful, 2310and fascinating creatures. The long-standing fear that many people have about bats tells us less about bats than about human fear. Things that live by night live outside the realm of normal time. Chauvinistic about our human need to 2315wake by day and sleep by night, we come to associate night dwellers with people up to no good, people who have the jump on the rest of us and are defying nature, defying their circadian rhythms.* Also, night is when we dream, and so we picture the bats moving through a dreamtime, in which 2320reality is warped. After all, we do not see very well at night; we do not need to. But that makes us nearly defenseless after dark. Although we are accustomed to mastering our world by day, in the night we become vulnerable as prey. Thinking of bats as masters of the night threatens the 2325safety we daily take for granted. Though we are at the top of our food chain, if we had to live alone in the rain forest, say, and protect ourselves against roaming predators. we would live partly in terror, as our ancestors did. Our sense of safety depends on predictability, so anything living 2330outside the usual rules we suspect to be an outlaw, a ghoul. Bats have always figured as frightening or supernatural creatures in the mythology, religion, and superstition of peoples everywhere. Finnish peasants once believed that their souls rose from their bodies while they slept and flew 2335around the countryside as bats, then returned to them by morning. Ancient Egyptians prized bat parts as medicine for a variety of diseases. Perhaps the most mystical, ghoulish, and intimate relationship between bats and humans occurred among the Maya about two thousand years ago.

2340Zotzilaha Chamalcn, their bat god, had a human body but the stylized head and wings of a bat. His image appears often on their altars, pottery, gold ornaments, and stone pillars. One especially frightening engraving shows the bat god with outstretched wings and a question-mark nose, its 2345tongue wagging with hunger, as it holds a human corpse in one hand and the humans heart in the other. A number of other Central American cultures raised the bat to the ultimate height: as god of death and the underworld. But it was Bram Stokers riveting novel Dracula that turned 2350small, furry mammals into huge bloodsucking monsters in the minds of English-speaking people. If vampires were semihuman, then they could fascinate with their conniving cruelty, and thus a spill of horror books began to appear about the human passions of vampires.

* Circadian rhythms are patterns of daily change within ones body that are
determined by the time of day or night.

16. The authors main point in the passage is that (A) there are only a few kinds of bats (B) humans are especially vulnerable to nocturnal predators (C) bat saliva may have medicinal uses (D) only myth and literature have depicted the true nature of the bat (E) our perception of bats has its basis in human psychology . 17. As used in line 14, classic most nearly means (A) (B) (C) (D) (E) literary enduring elegant well-known significant

18. The discussion of vampire bats in the first paragraph (lines 1-24) primarily suggests that (A) vampire bats are potentially useful creatures (B) movies about vampires are based only on North American bats (C) most tropical bats are not carnivorous (D) the saliva of vampire bats is more toxic than commonly supposed (E) scientists know very little about the behavior of most bats

70

2355

19. In line 26, the quotation marks around the word normal serve to (A) emphasize the individuality of the authors writing (B) criticize the human obsession with time (C) emphasize the limitations of a point of view (D) demonstrate the authors agreement with the common use of the word 2365(E) indicate that this word would be stressed if it were spoken out loud
2360

22. The author develops the third paragraph (lines 43-66) by presenting
2420(A) (B) (C) (D) (E) 2425 23.

different sides of a single issue details that culminate in truth a thesis followed by specific illustrations a common argument followed by a refutation of it a common opinion and the reasons it is held The practices of which group mentioned in the last paragraph best substantiate the claim that bats are useful (line 21)? Finnish peasants Ancient Egyptians Ancient Maya A number of Central American cultures English-speaking people

20. Which of the following assertions detracts LEAST from the authors argument in the second paragraph 2370 (lines 25-42) ? (A) Many people work at night and sleep during the day. (B) Owls, which hunt at night, do not arouse our fear. (C) Most dangerous predators hunt during the day. 2375(D) Some cultures associate bats with positive qualities. (E) Some dream imagery has its source in the dreamers personal life.
238021. The examples cited in the third paragraph (lines 43-66) are primarily drawn from

(A) 2430(B) (C) (D) (E)

243524. The reference to Stokers work in lines 60-66 extends the authors idea that

(A) (B) (C) 2385(D) (E)

anthropology autobiography fiction psychiatry biology

(A) bats are sweet-tempered creatures (B) our fear of bats reveals more about us than about bats 2440(C) humans have always been curious about nocturnal creatures (D) bats can see better than humans at night (E) bats appear as supernatural creatures even in the folklore of distant nations

2445

2390

2395

2400

2405

2410

2415

SECTION 7

WRITING

Time -25 minutes 35 Question


2450 Directions: For each question in this section, select the best answer from among the choices given and fill in the corresponding circle on the answer sheet.

The following sentences test correctness and effectiveness of expression. Part of each sentence or the entire sentence is underlined; beneath each sentence are five ways of phrasing the underlined material. Choice A repeats the original phrasing; the other four choices are different. If you think the original phrasing produces a better sentence than any of the alternatives, select choice A; if not, select one of the other choices. In making your selection, follow the requirements of standard written English; that is, pay attention to grammar, choice of words, sentence construction, and punctuation. Your selection should result in the most effective sentence clear and precise, without awkwardness or ambiguity. EXAMPLE: Laura Ingalls Wilder published her first book and she was sixty-five years old then. (A) and she was sixty-five years old then (B) when she was sixty-five (C) at age sixty-five years old (D) upon the reaching of sixty-five years (E) at the time when she was sixty-five

3. The campus newspaper does not print as much


2475

world news as does my hometown.


(A) as does my hometown

2480

(B) as does my hometown newspaper (C) compared to what my hometown does (D) like my hometown newspaper does (E) like the one in my hometown does

4. During the labor dispute, barrels of potatoes were


emptied across the highway. and they thereby blocked it to all traffic.
2485

2490

(A) highway, and they thereby blocked it to all traffic (B) highway and therefore blocking it to all traffic (C) highway, by which all traffic was therefore blocked (D) highway, and therefore this had all traffic blocked (E) highway, thereby blocking all traffic 5. Having thought the problem through with some care, that the committee did not understand her solution frustrated the chairperson extremely.

2495

2500 24551. In a recent year, more tourists from the United States visited museums in Great Britain than Canada.

2460

(A) (B) (C) (D) (E)

Canada Canada did compared to Canadas Canadian ones in Canada

2505

(A) that the committee did not understand her solution frustrated the chairperson extremely (B) the chairpersons extreme frustration resulted from the committee not understanding her solution (C) the chairpersons frustration at the committees failing to understand her solution was extreme (D) the chairperson was extremely frustrated by the committees failure to understand her solution (E) the committees failing to understand her solution was an extreme frustration to the chairperson

2. Conners, a publishing and media services company, is acquiring Dispatch Education, it manufactures school 2465 uniforms. (A) (B) (C) (D) (E) Dispatch Education, it manufactures Dispatch Education, which manufactures Dispatch Education, manufacturing Dispatch Education; it is manufacturing Dispatch Education; for the manufacturing of

2470

75
6. The main reasons students give for failing to participate in the political process is that they have demanding assignments and work at part-time jobs. (A) is that they have demanding assignments and work at (B) are demanding assignments and they work at (C) are that they have demanding assignments and that they work at (D) is having demanding assignments and having to work at (E) are demanding assignments, in addition to working at
2570 9. Having command of pathos, tragedy, as well as humor, George Eliot is considered to be a great English novelist.

2510

2575

2515

2580

2520

(A) Having command of pathos, tragedy, as well as humor (B) Having command of pathos, tragedy, and her humorous side (C) By being in command of both pathos and tragedy and also humor (D) With her command of pathos and tragedy and being humorous (E) Because of her command of pathos, tragedy, and humor 10. Richard Wright moved many times in his life, moving from the South first he went to the North, then eventually to France from the United States. (A) moving from the South first he went to the North, then eventually to France from the United States (B) the first move he made was from the South to the North and eventually from the United States to France (C) first from the South to the North and eventually from the United States to France (D) moving first from the South, he came to the North and eventually to France (E) first from the South he moved to the North and ended up in France after leaving the United States 11. Though heavily dependent on the government for business and information while universities supply the space research center with talent, as a corporation it remains independent of both. (A) information while universities supply the space research center with talent, as a corporation it remains (B) information and on talent by universities. the space research center, a corporation (C) information and on universities for talent, the space research center is a corporation (D) information, universities supply the space research center with talent, but it is a corporation (E) information. universities supply the space research center with talent, while it remains a corporation

7. Archaeologists say that the Pueblo village of Acoma, 2525 which is 7,500 feet above sea level and 400 feet above the valley floor, is the oldest continuously inhabited spot in the United States. (A) which is 7,500 feet above sea level and 400 feet above (B) located 7,500 feet high above sea level while having measured 400 feet above (C) with a height 7,500 feet above sea level as well as 400 feet above that of (D) 7,500 feet higher than sea level, and it ascends 400 feet above (E) being 7,500 feet above sea level and 400 feet high measured from that of 8. Returning to Dayville after ten years, the small town seemed much livelier to Margo than it had been when she was growing up there. (A) Returning to Dayville after ten years, the small town seemed much livelier to Margo (B) Having returned to Dayville after ten years, it seemed a much livelier town to Margo (C) After Margo returned to Dayville in ten years, the small town seems much livelier (D) Margo returned to Dayville after ten years, the small town was seemingly much livelier (E) When Margo returned to Dayville after ten years, the small town seemed much livelier to her

2585

2590

2530

2595

2535

2540

2600

2605

2545

2610

2550

2615 2555

2560

2565

2620

The following sentences test your ability to recognize grammar and usage errors. Each sentence contains either a single error or no error at all. No sentence contains more than one error. The error, if there is one, is underlined and lettered. If the sentence contains an error, select the one underlined part that must be changed to make the sentence correct. If the sentence is correct, select choice E. In choosing answers, follow the requirements of standard written English. EXAMPLE: The other delegates and him immediately A B C accepted the resolution drafted by the D neutral states. No error E

15. Even though only parts of clay vessels may be A C the archaeologist because it is virtually indestructible. D No error
2660

2655recovered, these pottery shards are invaluable to

E 16. Along the curve of islands known as the Florida Keys A lies a reef of living coral, the only one of a kind in

2665 B

C E

the continental United States. No error

12. Fourteen years after the Galileo space probe was A launched from the space shuttle Atlantis, the mission
2625 2670

17. Paule Marshall, whose Barbadian background A has influenced her writing, describes many details B C D
2675and short stories. No error

was purposely ended when the Galileo disintegrates B in the dense atmosphere of the planet Jupiter. D
2630No error

of life in the Caribbean Islands vividly in her novels

C E

E 13. The labor union is negotiating a contract with A


2635the hospital that will satisfy the demands of

18. Because he is absent when his rivals voted against A


2680his proposal, Selby is worried about missing future

B C D E

meetings of the board of directors. No error

C D

the workers and be acceptable to all levels of


268519. In those cities in which public transportation

management. No error
2640

E 14. Many professional athletes are motivated by either A personal pride and love of their sport, but some seem
2690

is adequate, fewer traffic problems occur and pedestrians are rarely involved in accidents. C No error E D

2645

B interested only in money. No error D E

2650

2695

20. Social scientists agree that a system for exchanging A goods and services is not only present but also C B

25. Intense preoccupation on technique appears to be A C No error E


275026. Apparently impressed with our plans, the foundation

B D

2745the one trait that great pianists have in common.

2700of necessity in all societies. No error

21. The report Alexander is discussing, a report A


2705prepared jointly by he and the committee,

A awarded Carlos and I a grant to establish a network B


2755

B does not take into account the socioeconomic C status of those interviewed. No error
2710

C D E

of community centers throughout the city; No error

27. Also supported by the commission was the proposed A B C information on job-training opportunities. No error D E health clinics and the proposed center to distribute
2760

22. It is far easier to ride a bicycle than explaining in A


2715

B D E

words exactly how a bicycle is ridden. No error C

23. Jorge wanted, for the most part, to travel around A the world after graduation, but sometimes he
2720 2765

28. The quality of multivitamin tablets is determined A by how long its potency can be protected by B C ... D E
2770

B thought about taking a job at his mothers company C instead. No error E D

the manufacturers coating material. No error

29. The research study reveals startling proof of a A constant changing seafloor that comprises the B D C E
2775major part of the underwater landscape. No error

2725

24. Since some people are convinced that dowsing, A B a method of finding underground water with a
2730Y -shaped stick, is effective, but others condemn

the procedure as mere superstition. No error C


2735

2740

80

2830

Directions: The following passage is an early draft of an essay. Some parts of the passage need to be rewritten. Read the passage and select the best answers for the questions that follow. Some questions are about particular sentences or parts of sentences and ask you to improve sentence structure or word choice. Other questions ask you to consider organization and development. In choosing answers, follow the requirements of standard written English.

30. Which of the following, if inserted before sentence 1, would make a good introduction to the essay?

2780
Questions 30-35 are based on following passage. (1) Employers must be aware of their employees and the variety of situations that arise in the workplace. 2785(2) Employers should become familiar with the demands a worker faces. (3) But he or she too should also assume responsibility. (4) Some workplace problems are caused by the employers insufficient attention to the needs of the 2790workers. (5) One familiar situation is the concern of the boss for the customers satisfaction above all else. (6) Often unreasonable demands are made on an employee to satisfy the customers. (7) This results from an employers lack of consideration for employees. 2795(8) Workers often become resentful of an employer who is unconcerned about their needs. (9) Sometimes the employer does not listen fully to suggestions from employees this can make workers feel undervalued. (10) Many times employers must deal with an 2800employee who ties up the phone for hours or has friends who continually drop by during working hours. (11) If workers would take more responsibility, then maybe an employer would be a little more easily tempted to promote them. 2805 (12) There are many problems to be solved in the workplace. (13) In order to achieve a happy balance between boss and employee, the job of a worker should be clearly defined. (14) Employers should listen to workers ideas about improving working conditions. 2810

(A) To avoid problems in the workplace, one must first recognize the variety of workplaces that exist. 2835(B) Many employees do not feel free to communicate with their employers, which can cause difficulties: (C) Some employers have tried to respect their employees. 2840(D) Communication between an employer and employees is necessary for maintaining good working conditions. (E) In the future, relations between employers and employees will be different from what they 2845 are now. 31. In context, which of the following is the best way to revise and combine sentences 2 and 3 (reproduced below)?
2850

Employers should become familiar with the demands a worker faces. But he or she too should also assume responsibility.

(A) When employers become familiar with the demands placed on their workers, they would 2855 also assume responsibility. (B) Employers ought to become familiar with the demands their workers face, but workers, too, must assume responsibility for their jobs. (C) Employers who have familiarity with the 2860 demands their workers face also need to take responsibility for them. (D) Those employees whose employers are familiar with their demands need to take responsibility for their jobs. 2865(E) Employees and employers, familiar with the demands of the workplace, must also assume responsibility for them.

2815

2820

2825

2870

32. In context, the underlined portion of sentence 7 (reproduced below) could best be revised in which of the following ways? This results from an employers lack of consideration for employees. (A) (B) (C) (D) (E) In contrast is With unreasonable demands, they show This concern illustrates Such a distorted view shows Such treatment demonstrates

2930

34. Which of the following sentences, if inserted before sentence 10, would best improve the third paragraph? (A) The role of technology in the workplace is also important. (B) The success of any business depends on effective communication with customers. (C) Sometimes employers have legitimate complaints about their employees. (D) It is difficult to tell whether certain problems are caused by employees or employers. (E) Employees rarely complain without good reason. 35. Which of the following would make the most logical final sentence for the essay?

2875

2935

2880

2940

33. Which of the following is the best version of the underlined portion of sentence 9 (reproduced below) ?
2885

Sometimes the employer does not listen fully to suggestions of employees this can make workers feel undervalued. (A) (As it is now) (B) When sometimes they do not listen fully to suggestions from employees, this can make (C) Because the employers had not listened fully to suggestions from employees, they made (D) An employer who does not listen closely to suggestions of employees, making (E) Sometimes an employer does not listen closely to suggestions from employees, making

2945

2950

2890

2955

(A) Responsibility for removal of safety hazards from the workplace lies with the employer. (B) For most employers, open communication with employees seems somewhat difficult. (C) The challenge of technology offers new opportunities for opening up communication in the workplace. (D) Without clearly assigned duties, workers tend to lose their motivation. (E) Employers and employees should work together to improve conditions in the workplace.

2895

2900

2905

2910

2915

2920

2925

85

2960

SECTION 8

CRITICAL READING
19 Question

Time -20 minutes

Directions: For each question in this section, select the best answer from among the choices given and fill in the corresponding circle on the answer sheet.

2965
2995

Each sentence below has one or two blanks, each blank indicating that something has been omitted. Beneath the sentence are five words or sets of words labeled A through E. Choose the word or set of words that, when inserted in the sentence, best fits the meaning of the sentence as a whole. Example: Hoping to ------- the dispute, negotiators proposed a compromise that they felt would be ------- to both labor and management. (A) (B) (C) (D) (E) enforce .. useful end .. divisive overcome .. unattractive extend .. satisfactory resolve .. acceptable

3. As ------- as the disintegration of the Roman Empire must have seemed, that disaster nevertheless presented some ------- aspects. (A) (B) {C) (D) (E) momentous .. formidable decisive .. unavoidable unexpected .. ambiguous advantageous .. beneficial catastrophic .. constructive

3000

30054. The beauty of Mount McKinley is usually cloaked: clouds ------- the summit nine days out of ten.

(A) release (D) shroud

(B) elevate (E) attain

(C) entangle

30105. Madame C. J. Walker introduced her first hair-care product just as demand was reaching its peak; this ------- marketing made her a millionaire.

29701. Though Luis eagerly sought her -------, he subsequently chose not to heed that advice.

3015

(A) opportune (B) instantaneous (C) intermittent (D) dubious (E) extravagant

(A) secretiveness (C) understanding (E) concord


2975

(B) cooperation (D) counsel

6. A scientist should not automatically reject folkways that might at first seem silly or superstitious; scientific qualifications are not a license for -------, nor do they 3020 ------- prejudice or bias. (A) (B) (C) (D) (E) experimentation .. eliminate arrogance .. pursue humility .. advocate smugness .. legitimate rigidity .. console

2. As a young physics instructor, Richard Feynman discovered that he had the gift of sharing his -----his subject and making that excitement -------. (A) (B) (C) (D) (E) passion for .. contagious knowledge of .. inaudible contempt for .. praiseworthy propensity for .. futile commitment to .. impersonal

2980

3025

2985

2990

3030 The passage below is followed by questions based on its content. Answer the questions on the basis of what is stated or implied in the passage and in any introductory material that may be provided.

Questions 7-19 are based on the following passage. Since the advent of television, social commentators have been evaluating its role in a modem society. In the following excerpt from an essay published in 1992, a German social commentator offers a pointed evaluation of the evaluators. Television makes you stupid. Virtually all current theories of the medium come down 3035to this simple statement. As a rule, this conclusion is delivered with a melancholy undertone. Four principal theories can be distinguished. The manipulation thesis points to an ideological dimension. It sees in television above all an instrument 3040of political domination. The medium is understood as a neutral vessel, which pours out opinions over a public thought of as passive. Seduced, unsuspecting viewers are won over by the wire-pullers, without ever realizing what is happening to them. 3045 The imitation thesis argues primarily in moral terms. According to it, television consumption leads above all to moral dangers. Anyone who is exposed to the medium becomes habituated to libertinism, irresponsibility, crime, and violence. The private consequences are blunted, callous, 3050and obstinate-individuals; the public consequences are the loss of social virtues and general moral decline. This form of critique draws, as is obvious at first glance, on traditional, bourgeois sources. The motifs that recur in this thesis can be identified as far back as the eighteenth century 3055in the vain warnings that early cultural criticism sounded against the dangers of reading novels. More recent is the simulation thesis. According to it, the viewer is rendered incapable of distinguishing between reality and fiction. The primary reality is rendered unrecog3060nizable or replaced by a secondary, phantomlike reality. All of these converge in the stupefaction thesis. According to it, watching television not only undermines the viewers ability to criticize and differentiate, along with the moral and political fiber of their being, but also impairs 3065their overall ability to perceive. Television produces, therefore, a new type of human being, who can, according to taste, be imagined as a zombie or a mutant. All these theories are rather unconvincing. Their authors consider proof to be superfluous. Even the minimal criterion 3070of plausibility does not worry them at all. To mention just one example, no one has yet succeeded in putting before us even a single viewer who was incapable of telling the difference between a family quarrel in the current soap opera and one at his or her familys breakfast table. This 3075doesnt seem to bother the advocates of the simulation thesis.

Another common feature of the theories is just as curious but has even more serious consequences. Basically, the viewers appear as defenseless victims, the programmers 3080as crafty criminals. This polarity is maintained with great seriousness: manipulators and manipulated, actors and imitators, simulants and simulated, stupefiers and stupefied face one another in a fine symmetry. The relationship of the theorists themselves to television 3085raises some important questions. Either the theorists make no use of television at all (in which case they do not know what they are talking about) or they subject themselves to it, and then the question arises through what miracle is the theorist able to escape the alleged effects of television? 3090Unlike everyone else, the theorist has remained completely intact morally, can distinguish in a sovereign manner between deception and reality, and enjoys complete immunity in the face of the idiocy that he or she sorrowfully diagnoses in the rest of us. Or could fatal loophole 3095in the dilemma the theories themselves be symptoms of a universal stupefaction? One can hardly say that these theorists have failed to have any effect. It is true that their influence on what is actually broadcast is severely limited, which may be considered 3100distressing or noted with gratitude, depending on ones mood. On the other hand, they have found ready listeners among politicians. That is not surprising, for the conviction that one is dealing with millions of idiots out there in the country is part of the basic psychological 3105equipment of the professional politician. One might have second thoughts about the theorists influence when one watches how the veterans of televised election campaigns fight each other for every single minute when it comes to displaying their limousine, their historic appearance before 3110the guard of honor, their hairstyle on the platform, and above all their speech organs. The number of broadcast minutes, the camera angles, and the level of applause are registered with a touching enthusiasm. The politicians have been particularly taken by the good old manipulation thesis.

7. In line 11, the term wire-pullers refers to the (A) (B) (C) (D) (E) bland technicians who staff television studios shadowy molders of public opinion self-serving critics of television hack writers who recycle old concepts slick advertisers of consumer goods

3115

8.

As used in line 14, consumption most nearly means (A) (B) (C) (D) (E) destruction viewing erosion purchasing obsession

3175

13. The authors attitude toward the evaluators of television can be best described as
3180

3120

9.
3125

The reference to the eighteenth century in lines 21-24 conveys what impression about cultural critiques based on moral grounds? (A) They are part of a tradition dating back to early civilization. (B) They were the main preoccupation of that eras social commentators. (C) They were once persuasive but now go mostly unheeded. (D) They are no more valid today than they were in those years. (E) They continue to appeal to people having no real understanding of art.

(A) (B) (C) (D) (E)

intrigued scornful equivocal indulgent nonchalant

318514. The author responds to the four theories of television primarily by

3130

3190

(A) (B) (C) (D)

offering contrary evidence invoking diverse authorities adding historical perspective blurring the line between the manipulator and the manipulated (E) implying that no reasonable person could take them seriously

3135

10. The author makes the comparison to the novel in lines 21-24 in order to
3140

319515. According to the passage, most current evaluations of television are based on which of the following assumptions about viewers?

(A) (B) (C) (D)

3145

point out televisions literary origins underscore the general decline of culture emphasize televisions reliance on visual imagery expose narrow-minded resistance to new forms of expression (E) attack the cultural shortcomings of television producers 11. The terms primary (line 27) and secondary (line 28) are used to refer to the distinction between

3200

I. Viewers are mostly interested in comedy programs. II. Viewers never engage their analytical faculties. III. Viewers see political content where there is none. (A) (B) (C) (D) (E) I only II only I and II only II and III only I, II, and III

3205

3150

(A) (B) (C) (D) (E)

an ideal democracy and our political system natural objects and human artifacts the everyday world and its fictional counterpart the morality of the elite and that of the populace the world view of scientists and that of mystics

16. In mapping out categories of theories about television, the author uses which of the following?
3210

3155

12. Advocates of the simulation thesis might best respond to the criticism in lines 37-44 by pointing out that the author (A) trivializes their theory by applying it too literally (B) concentrates excessively on a relatively insignificant point (C) is not a psychologist and so cannot properly evaluate their argument (D) attacks their theory in order to bolster one of the other three theories (E) fails to consider the impact of television on popular culture

(A) (B) (C) (D) (E)

Earnest reevaluation Incredulous analysis of academic documentation Somber warnings about the future Intentional falsification of data Description tinged with irony

3215

17. In line 59, sovereign is best understood to mean (A) (B) (C) (D) (E) excellent opulent elitist absolute oppressive

3160

3220

3165

3170

90

3225

18. The fatal loophole (line 62) is best summarized by which of the following statements? (A) Theorists are conspiring with the politicians. (B) Theorists are themselves victims of television. (C) All human beings occasionally behave like zombies and mutants. (D) Even serious thinkers need mindless entertainment occasionally. (E) Theorists have disregarded the enjoyment that television provides.
3290

19. In the last paragraph, the authors attitude toward politicians is primarily one of (A) (B) (C) (D) (E) humorous contempt outraged embarrassment worried puzzlement relieved resignation begrudging sympathy

3230

3295

3235

3240

3245

3250

3255

3260

3265

3270

3275

3280

3285

SECTION 10
3300

WRITING

Time -10 minutes 14 Question

Directions: For each question in this section, select the best answer from among the choices given and fill in the corresponding circle on the answer sheet.

3305

The following sentences test correctness and effectiveness of expression. Part of each sentence or the entire sentence is underlined; beneath each sentence are five ways of phrasing the underlined material. Choice A repeats the original phrasing; the other four choices are different. If you think the original phrasing produces a better sentence than any of the alternatives, select choice A; if not, select one of the other choices. In making your selection, follow the requirements of standard written English; that is, pay attention to grammar, choice of words, sentence construction, and punctuation. Your selection should result in the most effective sentence clear and precise, without awkwardness or ambiguity. EXAMPLE: Laura Ingalls Wilder published her first book and she was sixty-five years old then. (A) and she was sixty-five years old then (B) when she was sixty-five (C) at age sixty-five years old (D) upon the reaching of sixty-five years (E) at the time when she was sixty-five

3325

2. H. Ford Douglas, one of the few Black soldiers in White regiments during the early part of the Civil War, and eventually to recruit and command his own unit. (A) and eventually to recruit and command his own unit (B) eventually recruited and commanded his own unit (C) he eventually recruited and commanded his own unit (D) he eventually had his own unit that he recruited and commanded (E) having eventually recruited and commanded his own unit 3. Ignorance is not equivalent to stupidity, for ignorance can often be corrected while stupidity cannot.

3330

3335

3340

3345

(A) for ignorance can often be corrected while stupidity cannot (B) since you can often correct ignorance while the same is not true about stupidity (C) because it can be corrected and the other cannot (D) because of its correctible nature (E) because the two differ regarding correctibility 4. Journalists should present a balanced view of the news but with their goal to stir discussion and unsettle complacent thinkers.

3350

1. In scenarios reminiscent of the old science fiction movie Fantastic Voyage, medical researchers hope 3310 exploring the body with miniature robots sent into the bloodstream. (A) (B) (C) (D) (E) hope exploring hope to explore hope it can explore have hopes to explore are having hopes of exploring

(A) (B) (C) (D) (E)

with their goal to stir should also stir aiming at the same time to stir also trying to stir its goal should also be in stirring

3355

3315

3320

95
5. The ancient Spartans tested the endurance of potential warriors, devised various ordeals, including one that 3360 required them to run bare-legged through fields of stinging nettles. (A) (B) (C) (D) (E) warriors, devised warriors devising warriors; and devised warriors by devising warriors with the devising of 8. Although Central Park in Manhattan is better known than Prospect Park in Brooklyn, the designer of both parks, Frederick Law Olmsted, preferred Prospect Park. (A) Although Central Park in Manhattan is better known than Prospect Park in Brooklyn, the designer of both parks, Frederick Law Olmsted, preferred Prospect Park. (B) Central Park in Manhattan being better known than Prospect Park in Brooklyn, the designer of both, Frederick Law Olmsted, preferred the latter. (C) Although not as well known as Central Park, Frederick Law Olmsted, he designed both parks, preferred Prospect Park. (D) The designer of both Central Park and Prospect Park was Frederick Law Olmsted, he preferred Prospect Park. (E) Although more people know about Manhattans Central Park than Prospect Park in Brooklyn, Frederick Law Olmsted, having designed both, has preferred the latter. 9. Because Uranus is nearly three billion kilometers from the Sun and is enveloped by a thick methane cloud layer, this blocks almost all solar radiation.
3445

3420

3425

3365

6. The lawyers representing the parking-lot operators asserted as to the defensibility of their practices 3370 as legal and ethical. (A) as to the defensibility of their practices as legal and ethical (B) as to their practices and their defensibility on legal and ethical grounds (C) that their practices, that is the operators, are defensible in legal terms as well as ethics (D) that in regards to defensibility their practices are legally and ethically defensible (E) that the practices of the operators are legally and ethically defensible 7. Toni Morrison was honored by Harvard University not only as a great novelist but also she wrote eloquently of the history of African American culture.
3385

3430

3435

3375

3440

3380

(A) (B) (C) (D) (E)

she wrote eloquently of the history having written an eloquent history writing eloquently about the history being an eloquent historian as an eloquent historian

3450

3390

(A) layer, this blocks almost all solar radiation (B) layer, this accounts for its receiving almost no solar radiation (C) layer is the reason why it receives almost no solar radiation (D) layer, almost no solar radiation reaches the planet (E) layer, it blocks almost all solar radiation from reaching the planet

3395

3400

3405

3410

3415

3515 3455

10. Lacking good instruction, my mistakes in creating a graph to illustrate historical trends were numerous. (A) my mistakes in creating a graph to illustrate historical trends were numerous (B) I made numerous mistakes in creating a graph to illustrate historical trends (C) there were numerous mistakes in the graph I created to illustrate historical trends (D) I created a graph to illustrate historical trends with numerous mistakes (E) the graph I made for illustrating historical trends had numerous mistakes
3520

13. Some people believe that one day we will establish not only bases on the Moon, but also a landing onNeptune will occur. (A) we will establish not only bases on the Moon, but also a landing on Neptune will occur (B) not only bases on the Moon will be established, but also a landing on Neptune will be made (C) we will not only establish bases on the Moon but also land on Neptune (D) we will not only establish bases on the Moon, but we will land on Neptune in addition (E) we will not only establish bases on the Moon, but we will land on Neptune 14. The city is populated by many people who, although their common language is English, the languages at home range from speaking Armenian to Zapotec. (A) the languages at home range from speaking Armenian to Zapotec (B) speaking at home is in languages ranging from Armenian to Zapotec (C) the languages range from Armenian to Zapotec at home (D) speak languages at home that range from Armenian to Zapotec (E) they are speaking languages at home ranging from Armenian to Zapotec

3460

3525

3465

347011. It is a myth that mathematicians are so absorbed with abstractions and thus have no practical interests.

3530

3475

(A) (B) (C) (D) (E)

so absorbed with abstractions and thus absorbed by abstractions and therefore so absorbed in abstractions that they absorbed in so much abstraction that they too abstract, and so they

3535

12. By simply entering an Internet website or calling a tollfree number, a catalog order can be placed for almost 3480 anything from cheesecakes to fully equipped desktop computers. (A) (B) (C) (D) (E) a catalog order can be placed by placing a catalog order they will place your catalog order you can place a catalog order your catalog order can be placed

3540

3545

3485

3490

3495

3500

3505

3510

Practice Test 3 SECTION 1 WRITING ESSAY

Time 25 minutes
3550

The essay gives you an opportunity to show how effectively you can develop and express ideas. You should, therefore, take care to develop your point of view, present your ideas logically and clearly, and use language precisely.

3555Your essay must be written on the lines provided on your answer sheet you will receive no other paper on which to write. You will have enough space if you write on every line, avoid wide margins, and keep your handwriting to a reasonable size. Remember that people who are not familiar with your handwriting will read what you write. Try to write or print so that what you are writing is legible to those readers. 3560

You have twenty-five minutes to write an essay on the topic assigned below. DO NOT WRITE ON ANOTHER TOPIC. AN OFF-TOPIC ESSAY WILL RECEIVE A SCORE OF ZERO.

3565Think carefully about the issue presented in the following quotations and the assignment below.

3570

A mistakenly cynical view of human behavior holds that people are primarily driven by selfish motives: the desire for wealth, for power, or for fame. Yet history gives us many examples of individuals who have sacrificed their own welfare for a cause or a principle that they regarded as more important than their own lives. Conscience that powerful inner voice that tells us what is right and what is wrong can be a more compelling force than money, power, or fame.

3575

Assignment:

Is conscience a more powerful motivator than money, fame, or power? Plan and write an essay in which you develop your point of view on this issue. Support your position with reasoning and examples taken from your reading, studies, experience, or observations.

3580

100

Practice Test 3 SECTION 3


3585

CRITICAL READING
24 Questions

Time 25 minutes

Directions: For each question in this section, select the best answer from among the choices given and fill in the corresponding circle on the answer sheet.

3590
3620

Each sentence below has one or two blanks, each blank indicating that something has been omitted. Beneath the sentence are five words or sets of words labeled A through E. Choose the word or set of words that, when inserted in the sentence, best fits the meaning of the sentence as a whole. Example: Hoping to ------- the dispute, negotiators proposed a compromise that they felt would be ------- to both labor and management. (A) (B) (C) (D) (E) enforce .. useful end .. divisive overcome .. unattractive extend .. satisfactory resolve .. acceptable

3. The frequent name changes that the country has undergone ------ the political turbulence mat has attended its recent history. (A) argue against (B) contrast with (C) testify to (D) jeopardize (E) sustain

3625

4. Brachiopods, clamlike bivalves of prehistoric times, were one of the most------- forms of life on the Earth: 3630 more than 30,000 species have been ------- from fossil records. (A) plentiful .. subtracted (B) ornate .. retrieved (C) multifarious .. catalogued (D) scarce .. extracted (E) anachronistic .. extrapolated

3635

3595

1. For a long time. most doctors maintained that taking massive doses of vitamins was relatively harmless; now, however, some are warning that excessive dosages can be -------. (A) healthy (D ) toxic (B) expensive (E) inane (C) wasteful

5. Some interactive computer games are so elaborately contrived and require such ------- strategies that only 3640 the most ------- player can master them. (A) byzantine .. adroit (B) nefarious .. conscientious (C) devious . . lackadaisical (D) onerous .. slipshod (E) predictable .. compulsive

3645

3600

2. In Jamaica Kincaids novel Lucy, the West Indian heroine ------- her employers world, critically examining its assumptions and values. (A) idealizes (B) avoids (C) beautifies (D) scrutinizes (E) excludes

3605

3610

3615

Each passage below is followed by questions based on its content. Answer the questions on the basis of what is stated or implied in the passage and in any introductory material that may be provided.

Questions 6-7 are based on the following passage. The critic Edmund Wilson was not a self-conscious letter writer or one who tried to sustain studied mannerisms. Nor did he resort to artifice or entangle himself in circumlocutions. The young, middle-aged, and old Wilson speaks directly through his letters, which are informal for 3655the most part and which undisguisedly reflect his changing moods. On occasion in response, perhaps, to the misery of a friend or a public outrage or a personal challenge he can become eloquent, even passionate, but that is not his prevailing tone.
3650 3660

Questions 8-9 are based on the following passage. The belief that it is harmful to the Black community for authors to explore the humanity of our leaders can have troubling effects. At the least, it promotes the belief that our heroes have to be perfect to be useful. At worst, it censors our full investigation of Black life. If our paint3665ings of that life are stock and cramped, their colors drab and predictable, the representations of our culture are likely to be untrue. They will not capture the breadth and complexity of Black identity.

6. Based on the information in the passage, Wilsons letters can best be described as (A) (B) (C) (D) (E) cynical spontaneous critical preachy witty

8. The passage implies that Black leaders have sometimes been portrayed as being (A) (B) (C) (D) (E) overly sentimental deeply complex above reproach without regret beyond understanding

7. The reference to the young, middle-aged, and old Wilson (line 4) serves to suggest the (A) multifaceted nature of Wilsons literary persona (B) maturity Wilson displayed even as a youth (C) effect aging had on Wilsons temperament (D) longevity of Wilsons literary career (E) consistency of Wilsons letter-writing style

9. In context, the paintings (lines 15-16) are best understood as a reference to (A) (B) (C) (D) (E) realistic sculptures historical biographies whimsical novels political cartoons colorful theorems

105
Questions 10-18 are based on the following passage. The following passage was written by a physicist in 1986. When astronomers point their telescopes to the nearest 3670galaxy, Andromeda, they see it as it was two million years ago. Thats about the time Australopithecus* was basking in the African sun. This little bit of time travel is possible because light takes two million years to make the trip from there to here. Too bad we couldnt turn things around and 3675observe Earth from some cozy planet in Andromeda. But looking at light from distant objects isnt real time travel, the in-the-flesh participation in past and future found in literature. Ever since Ive been old enough to read science fiction, Ive dreamed of time traveling. The possibilities 3680are staggering. You could take medicine back to fourteenthcentury Europe and stop the spread of plague, or you could travel to the twenty-third century, where people take their annual holidays in space stations. Being a scientist myself, I know that time travel is 3685quite unlikely according to the laws of physics. For one thing, there would be a causality violation. If you could travel backward in time, you could alter a chain of events with the knowledge of how they would have turned out. Cause would no longer always precede effect. For example, 3690you could prevent your parents from ever meeting. Contemplating the consequences of that will give you a headache, and science fiction writers for decades have delighted in the paradoxes that can arise from traveling through time. 3695 Physicists are, of course, horrified at the thought of causality violation. Differential equations for the way things should behave under a given set of forces and initial conditions would no longer be valid, since what happens in one instant would not necessarily determine 3700what happens in the next. Physicists do rely on a deterministic universe in which to operate, and time travel would almost certainly put them and most other scientists permanently out of work. Still, I dream of time travel. There is something very 3705personal about time. When the first mechanical clocks were invented, marking off time in crisp, regular intervals, it must have surprised people to discover that time flowed outside their own mental and physiological processes. Body time flows at its own variable rate, oblivious 3710to the most precise clocks in the laboratory. In fact, the human body contains its own exquisite timepieces, all with their separate rhythms. There are the alpha waves in the brain; another clock is the heart. And all the while tick the mysterious, ruthless clocks that regulate aging. 3715 Recently, I found my great-grandfathers favorite pipe. Papa Joe, as he was called, died more than seventy years ago, long before I was born. There are few surviving photographs or other memorabilia of Papa Joe. But I do have his pipe, which had been tucked away in a drawer somewhere 3720for years and was in good condition when I found it. I ran a pipe cleaner through it, filled it with some tobacco I had on hand, and settled down to read and smoke. After a couple of minutes, the most wonderful and foreign blend of smells began wafting from the pipe. All the different occasions 3725when Papa Joe had lit his pipe, all the different places he had been that I will never know all had been locked up in that pipe and now poured out into the room. I was vaguely aware that something had got delightfully twisted in time for a moment, skipped upward on the page. There 3730is a kind of time travel to be had, if you dont insist on how it happens.
* An extinct humanlike primate

10. The author mentions Australopithecus in line 3 in order to (A) note an evolutionary progression in the physical world (B) dramatize how different Earth was two million years ago (C) commend the superior work of astronomers in isolating a moment early in time (D) establish a link between the length of time that Africa has been inhabited and the discovery of the Andromeda galaxy (E) emphasize the relatively long period of human life compared to the age of the universe 11. The statement in lines 6-7 (Too bad ... Andromeda) suggests that (A) scientists would like to observe events that occurred on Earth in the distant past (B) there may be planets in Andromeda that are reachable through space travel (C) the study of Andromeda would offer interesting comparisons to planet Earth (D) a planet in Andromeda will be a likely observation point for Earth in the future (E) Andromeda is much older than Earth 12. The author mentions plague (line 13) and space stations (line 15) primarily to (A) give an example of the themes of novels about time travel (B) suggest contrasting views of the future (C) scoff at the scientific consequences of time travel (D) give examples of the subjects that scientists are interested in (E) suggest why time travel is such a fascinating topic

3735

13. The author introduces the third paragraph with the words Being a scientist in order to (A) explain an intense personal interest in the topic (B) lend an air of authority to the discussion of time travel (C) suggest why certain forms of literature are so appealing (D) provoke those who defend science fiction (E) help illustrate the term causality violation 14. In discussing causality violations (lines 16-35), the author addresses concerns about all of the following EXCEPT (A) (B) (C) (D) (E) anticipatory knowledge of events the belief in a deterministic universe the mechanics of space travel cause-and-effect relationships differential equations based on known forces

379516. The author mentions the brain and the heart (lines 44-45) in order to

3800

3740

(A) demonstrate the rhythmical qualities of timepieces (B) explain the historical significance of mechanical clocks (C) emphasize how the two organs interact to regulate internal rhythms (D) illustrate the bodys different internal clocks (E) demystify the precision of organic processes

3745

380517. The author uses the word ruthless (line 46) to suggest that

3810

3750

(A) (B) (C) (D) (E)

people are bewildered by the prospect of aging the human body has mysterious capacities some people age more rapidly than others do peoples sense of time changes as they age the process of aging is relentless

15. Which of the following, if true, would undermine the validity of me authors assumption about the impact of 3755 mechanical clocks (When the first . .. the laboratory) in lines 37-42 ? (A) People were oblivious to time on a physical level before clocks were invented. (B) People have always perceived time as composed of discrete, uniform intervals. (C) Concern about time was unnecessary until clocks were invented, (D) Mental and physiological processes are very predictable. (E) Body time does not move at a constant rate.

18. The author mentions that something . skipped upward on the page (lines 60-61) to suggest that
3815

(A) (B) (C) (D) (E)

3820

he reread a portion of the page his vision was affected by the smoke he traveled back in time in his imagination his reading reminded him of Papa Joe he believes that reading is the best way to recreate the past

3760

3765

3770

3775

3780

3785

3790

Questions 1924 are based on the following passage. The following passage is an excerpt from a book about twentieth-century developments in art. The author refers here to the modern art that emerged shortly after the turn of the century. Many people found this art shocking. If the new art is not accessible to everyone, which certainly seems to be the case, this implies that its impulses are not of a generically human kind. It is an art not for 3825people in general but for a special class who may not be better but who are evidently different. Before we go further, one point must be clarified. What is it that the majority of people call aesthetic pleasure? What happens in their minds when they like a work of 3830art; for example, a play? The answer is easy. They like a play when they become interested in the human destinies that are represented, when the love and hatred, the joys and sorrows of the dramatic personages so move them that they participate in it all as though it were happening in real life. 3835And they call a work good if it succeeds in creating the illusion necessary to make the imaginary personages appear like living persons. In poetry the majority of people seek the passion and pain of the human being behind the poet. Paintings attract them if they find in them figures of men or 3840women it would be interesting to meet. It thus appears that to the majority of people aesthetic pleasure means a state of mind that is essentially indistinguishable from their ordinary behavior. It differs merely in accidental qualities, being perhaps less utilitarian, 3845more intense, and free from painful consequences. But the object toward which their attention and, consequently, all their other mental activities are directed is the same as in daily life: people and passions. When forced to consider artistic forms proper-for example, in some surrealistic or 3850abstract art-most people will only tolerate them if they do not interfere with their perception of human forms and fates. As soon as purely aesthetic elements predominate and the story of John and Susie grows elusive, most people feel out of their depth and are at a loss as to what to make 3855of the scene, the book, or the painting. A work of art vanishes from sight for a beholder who seeks in that work of art nothing but the moving fate of John and Susie or Tristan and Isolde.* Unaccustomed to behaving in any mode except the practical one in which feelings are aroused 3860and emotional involvement ensues, most people are unsure how to respond to a work that does not invite sentimental intervention. Now this is a point that has to be made perfectly clear. Neither grieving nor rejoicing at such human destinies as 3865those presented by a work of art begins to define true artistic pleasure; indeed, preoccupation with the human content of the work is in principle incompatible with aesthetic enjoyment proper.
* Tristan and Isolde were star-crossed lovers in a medieval romance.

19. The passage is primarily concerned with the (A) lives artists lead as opposed to the ones they imagine (B) emotional impact of a paintings subject matter (C) nature of the pleasure that most people find in a work of art (D) wide variety of responses that audiences have to different works of art (E) contrast between the formal elements of the new art and those of the old 20. As used in line 18, figures most nearly means (A) (B) (C) (D) (E) crude images abstractions representations numbers famous persons

21. It is most likely that the story of John and Susie (line 32) refers to (A) a fictional work that the author will proceed to critique (B) a typical narrative of interpersonal relationships (C) an account of an affair in the form of a mystery (D) a legendary couple that has fascinated artists through the ages (E) a cryptic chronicle of renowned historical personages 22. The author suggests that the majority of people resist modem art because they (A) consider modem artists to be elitist (B) are too influenced by critics to view the art on its own merits (C) are annoyed by its social message (D) find in it little of human interest to engage them (E) find it too difficult to guess at the artists source of inspiration

110

3870

23. The authors attitude toward the majority of people can best be described as (A) (B) (C) (D) (E) genuinely puzzled aggressively hostile solemnly respectful generally indifferent condescendingly tolerant

24. The authors assumption in the final paragraph (lines 42-47) is that
3920

3875

3925

3880
3930

(A) aesthetic pleasure is a response to the purely artistic elements in a work of art (B) aesthetic enjoyment of a work of art must focus on the artists intentions as much as on the artists actual accomplishments (C) responses to a work of art vary and cannot be easily defined (D) the evocation of emotional responses by a traditional work of art depends on the moral conventions of the artists society (E) the majority of people trying to interpret a work of art will concentrate on the artistic technique

3885

3890

3895

3900

3905

3910

3915

SECTION 6
3935

WRITING

Time -25 minutes 35 Question

Directions: For each question in this section, select the best answer from among the choices given and fill in the corresponding circle on the answer sheet.

3940
3960

The following sentences test correctness and effectiveness of expression. Part of each sentence or the entire sentence is underlined; beneath each sentence are five ways of phrasing the underlined material. Choice A repeats the original phrasing; the other four choices are different. If you think the original phrasing produces a better sentence than any of the alternatives, select choice A; if not, select one of the other choices. In making your selection, follow the requirements of standard written English; that is, pay attention to grammar, choice of words, sentence construction, and punctuation. Your selection should result in the most effective sentence clear and precise, without awkwardness or ambiguity. EXAMPLE: Laura Ingalls Wilder published her first book and she was sixty-five years old then. (A) and she was sixty-five years old then (B) when she was sixty-five (C) at age sixty-five years old (D) upon the reaching of sixty-five years (E) at the time when she was sixty-five

2. After marching for four hours in temperatures exceeding ninety degrees, the band members were as soaked as if marching through a rainstorm. (A) (B) (C) (D) (E) if marching having marched if from marching if they had marched if they would have marched

3965

39703. The harmful effects of smoking on the vascular system is increasingly well documented.

3975

(A) (B) (C) (D) (E)

is increasingly well documented is more and more documented are increasingly well documented are increasing in better documentation has increased in better documentation

4. The issue the council debated, which was whether repeal of rent control will improve housing or just 3980 increase profits for landlords. (A) debated, which was whether repeal of rent control will improve (B) debated was if they would repeal rent control would this improve (C) debated was that repeal of rent control would result in improved (D) debated was will repealing rent control mean improvement in (E) debated was whether repeal of rent control would improve

3985

1. The delegates coming this far, they did not want to return without accomplishing something.
3945

3990

(A) (B) (C) (D) (E)

The delegates coming this far, they Coming this far, the delegates felt they Having come this far, the delegates To come this far, the delegates The delegates came this far, so that they

3950

3955

115
3995
5. American journalist Harriet Quimby, the first woman 8. Being as she is a gifted storyteller, Linda Goss is an

4000

to pilot a plane across the English Channel, doing it just nine years after the Wright brothers first powered flight. (A) Harriet Quimby, the first woman to pilot a plane across the English Channel, doing it (B) Harriet Quimby the first woman who piloted a plane across the English Channel, and who did so (C) Harriet Quimby became the first woman to have piloted a plane across the English Channel and did it (D) Harriet Quimby became the first woman to pilot a plane across the English Channel, accomplishing this feat (E) Harriet Quimby was the first woman piloting a plane across the English Channel, the feat was accomplished
6. Naomi and Charles will represent Ammonton High in

expert at describing people and places.


4060

(A) (B) (C) (D) (E)

Being as she is a gifted storyteller In being a gifted storyteller A gifted storyteller Although she is a gifted storyteller Telling stories giftedly

4005

4065

9. Although the English artist William Blake never

having painted portraits, he regarded them as merely mechanical reproductions that, despite their popularity, lacked true creativity.
4070

4010

4015

4075

the debating contest, their work in this having been excellent this year.
4020

(A) Although the English artist William Blake never having painted portraits, he regarded them (B) The English artist William Blake never painted portraits, he regarded them (C) Never having painted a portrait, they were regarded by the English artist William Blake (D) The English artist William Blake never painted portraits; however, regarding them (E) The English artist William Blake never painted portraits because he regarded them

4025

(A) contest, their work in this having been excellent this year (B) contest; they have done excellent work this year in this (C) contest, for this year they have done excellent work in this (D) contest, for their work as public speakers has been excellent this year (E) contest; their work as public speakers having been excellent this year

4080 10. The heat was already overwhelming and lasted a week, which duration made it seem sheer torture.

4085

40307. The poems colorful images and its verbal wit give the reader pleasure.

4090

(A) overwhelming and lasted a week, which duration made it seem (B) overwhelming, and because of lasting a week, it made it seem (C) overwhelming and lasted the duration of a week to make it to seem (D) overwhelming, and its lasting a week made it seem (E) overwhelming and, by lasting a week, making it seem

4035

(A) (B) (C) (D) (E)

give the reader pleasure please the one who is reading gives pleasure to the one who reads it give one pleasure in the reading of it gives one pleasure in reading it

11. The reason first novels are so often their writers 4095 best work is that it draws upon all the experiences of childhood.

4040

4100

(A) (B) (C) (D) (E)

is that it draws upon is that these first efforts draw upon is because of these first efforts drawing from is because of them drawing upon is their drawing from

4045

4050

4055

4105
The following sentences test your ability to recognize grammar and usage errors. Each sentence contains either a single error or no error at all. No sentence contains more than one error. The error, if there is one, is underlined and lettered. If the sentence contains an error, select the one underlined part that must be changed to make the sentence correct. If the sentence is correct, select choice E. In choosing answers, follow the requirements of standard written English. EXAMPLE: The other delegates and him immediately A B C accepted the resolution drafted by the D neutral states. No error E

14. People which need immediate reeducation for A


4140employment are impatient with the prolonged

B D No error
4145

debate over funding for the new national program.

E 15. The thoughtful student wonders what Patrick Henry meant when he talked about liberty

4150

A C

because most of the members of the House of Burgesses then having been slaveholders. No error

12. Directed by George Wolfe, the Broadway musical


4110Bring in Da Noise, Bring in Da Funk telling how

D
4155

16. One challenge that writer Eleanor Wong Telemaque faced was how preserving her ethnic identity A
4160while becoming more accessible to readers who

A C

tap dancing evolved from the African American experience in the decades following the Civil War.
4115

D No error E 13. The question of whether certain chemical fertilizers


4165

B D backgrounds. No error E

are not accustomed to writers from other cultural

4120

A are a curse or are they a blessing is still being B debated. No error E C D


4170

17. No one objects to his company, even though he has A made insulting remarks about almost every member B D No error E
4175

of the group, when he is a remarkably witty man.

4125

4130

4135

418018. Just how critical an improved balance of trade is to

22. Although the politician was initially very sensitive to


4230

A C it is now. No error
4185

B D

A be criticized by the press, he quickly became more B D C confident about responding to reporters sometimes

a healthy economy has never been more clearer than

E 19. Critics contend that reforms in welfare A has not managed to bring the high percentage

4235pointed questions. No error

E 23. Although Pre-Raphaelite artists advocated the close A


4240study of nature , their paintings sometimes seem

4190

B of our nations children living in poverty the economic security that they need to thrive. C D

B D

C E

elaborately artificial to modem viewers. No error

4195No error

E 20. Crabs living in polluted waters will come A


4200in contact with large numbers of disease-causing

424524. Lions and tigers may be identical in size, but the

A C No error
4250

B D

tiger is the fiercer animal and the lion the strongest.

B microorganisms because it feeds by filtering C nutrients from water. No error


4205

E 25. The decline in science education during the D period had two causes: less funding for scientific

E
4255

A C defense. No error E

B D

21. The new system, which uses remote cameras A in the catching of speeding motorists ,
4210

research with a decrease in jobs related to space and

B D No error E

may undermine the police departments authority.

4260

4215

4220

4225

120

26. The number of awards given this year to biochemists A


4265accentuate the significant gains being made in the

Directions: The following passage is an early draft of an essay. Some parts of the passage need to be rewritten. Read the passage and select the best answers for the questions that follow. Some questions are about particular sentences or parts of sentences and ask you to improve sentence structure or word choice. Other questions ask you to consider organization and development. In choosing answers, follow the requirements of standard written English.
4310Questions 30-35 are based on the following passage.

D E

study of the chemistry of living organisms. No error

427027. The novel Pride and Prejudice by Jane Austen

was once more widely read and was more popular A


4275

in high schools in the United States than Charlotte Bront . No error D E

28. Until it can be replaced by a faster , more efficient,


4280

and more economical means of transportation, trucks will carry most of the freight within and through C
4285metropolitan areas. No error

D E

29. Nearly all of the editors of the magazine A


4290agree that of the two articles to be published ,

B D
4295

C E

(1) This summer I felt as if I were listening in on the Middle Ages with a hidden microphone. (2) No, there were no microphones in those days. (3) But there were letters, and 4315sometimes these letters speak to me like voices from very long ago. (4) A book I found contained selected letters from five generations of a family. (5) The Pastons, who lived in a remote part of England over 500 years ago. 4320 (6) Getting anywhere in the Middle Ages was really hard, with deep rivers and few bridges and sudden snowstorms coming on in the empty lands between settlements. (7) An earl rebelled in London, so that a messenger rode for days to tell the distant head of the 4325Paston family of a feared civil war. (8) Through the letters a modem reader can sense their anxieties about rebellious sons and daughters, belligerent neighbors, outbreaks of plague, and shortages of certain foods and textiles. (9) Unbelievably, there is a 1470 love 4330letter. (10) The man who wrote it ends I beg you, let no one see this letter. (11) As soon as you have read it, bum it, for I would not want anyone to see it. (12) I was sitting on the front porch with bare feet on the hottest afternoon in July and I read that with a shiver. (13) I had been part of a 4335centuries-old secret.

Fujimuras is the more exciting . No error

4300

4305

4340

30. The best way to describe the relationship of sentence 2 to sentence 1 is that sentence 2 (A) anticipates a readers possible response to sentence 1 (B) provides historical background for sentence 1 (C) repeats the idea presented in sentence 1 (D) introduces a contrasting view of sentence 1 (E) corrects an inaccuracy stated in sentence I

4400

33. In context, which of the following is the best version of the underlined portion of sentence 7 (reproduced below) ? An earl rebelled in London, so that a messenger rode for days to tell the distant head of the Paston family of a feared civil war. (A) (B) (C) (D) (E) (As it is now) An earl had rebelled in London, so For example, with a rebelling earl in London While an earl rebels in London, Once, when an earl rebelled in London,

4345

4405

435031. Which of the following sentences would be most logical to insert before sentence 4 ?

4410

4355

(A) I first came across these letters while browsing in a library. (B) No, I am not dreaming; I have been reading them. (C) On the contrary, microphones are a recent invention. (D) Obviously, a library can open the door to mystery. (E) However, letters are not the oldest form of communication. 32. In context, which is the best version of the underlined portions of sentences 4 and 5 (reproduced below) ? A book I found contained selected letters from five generations of a family. The Pastons, who lived in a remote part of England over 500 years ago. (A) (B) (C) (D) (E) (as it is now) a family. The Pastons, living a family; it was the Pastons living the Paston family, who lived the family named Paston and living

34. In context, which is the best revision to make to sentence 8 (reproduced below) ?
4415

Through the letters a modern reader can sense their anxieties about rebellious sons and daughters, belligerent neighbors, outbreaks of plague, and shortages of certain foods and textiles. (A) (B) (C) (D) (E) Insert ones reading of after Through. Change their to the Pastons . Change sense to record. Delete some of the examples. Insert etc. after textiles.

4360

4420

4365

442535. All of the following strategies are used by the writer of the passage EXCEPT

4370

4430

(A) (B) (C) (D) (E)

background explanation imaginative description rhetorical questions personal narration direct quotation

4375

4380

4385

4390

4395

125 4435 SECTION 7 CRITICAL READING


24 Questions
Directions: For each question in this section, select the best answer from among the choices given and fill in the corresponding circle on the answer sheet.
4440 4470

Time 25 minutes

Each sentence below has one or two blanks, each blank indicating that something has been omitted. Beneath the sentence are five words or sets of words labeled A through E. Choose the word or set of words that, when inserted in the sentence, best fits the meaning of the sentence as a whole. Example: Hoping to ------- the dispute, negotiators proposed a compromise that they felt would be ------- to both labor and management. (A) (B) (C) (D) (E) enforce .. useful end .. divisive overcome .. unattractive extend .. satisfactory resolve .. acceptable

5. Many ------- of the style of painting exemplified by Marcel Duchamps work focused on Duchamps Nude Descending a Staircase as the------- of what they 4475 detested about modem art. (A) (B) (C) (D) (E) critics .. epitome proponents .. realization advocates .. embodiment debunkers .. rejection belittlers .. reversal

4480

6. Colonial American playwright Mercy Otis Warren was known for her political -------: her keen judgment and insight were widely acknowledged.
4485

1. Commerce on the remote island was conducted exclusively by -------, exchanging goods for goods.
4445

(A) (D) (B) (E) (C)

partisanship irreverence intemperance interest acumen

4490

(A) credit (B) loan (C) faith (D) patronage (E) barter
4495

7. Johnsons writing is considered ------- and -----because it is filled with obscure references and baffling digressions. (A) (B) (C) (D) (E) deceiving .. ingenuous arcane .. abstruse spare .. didactic lucid .. definitive concise .. esoteric

2. The existence of environmental contamination is no longer a point of -------; government, industry, and the 4450 public agree that it is a serious problem. (A) concern (B) cooperation (D) relevance (E) dispute (C) urgency

3. In rock climbing, survival depends as much on -------, 4455 the ability to perceive without conscious reasoning, as on physical strength. (A) autonomy (D) sophistry (B) incoherence (E) receptivity (C) intuition

45008. Because the congresswoman has been so openhanded with many of her constituents, it is difficult to reconcile this ------- with her private -------.

4505

44604. Using computer labs to ------- classroom instruction is most effective when the curriculum ------- lab exercises and classroom teaching in a coordinated manner.

(A) (B) (C) (D) (E)

selfishness .. inattention insolence .. virtue magnanimity .. pettiness opportunism .. ambition solicitousness .. generosity

4465

(A) (B) (C) (D) (E)

supplement .. integrates substantiate .. undermines remedy .. compromises disparage .. reinforces foster .. curtails

4510

The passages below are followed by questions based on their content; questions following a pair of related passages may also be based on the relationship between the paired passages. Answer the questions on the basis of what is stated or implied in the passages and in any introductory material that may be provided.

4515

Questions 9-12 are based on the following passages. Passage 1 What accounts for the inexorable advance of the giant sports utility vehicle (SUV), into our lives? Why do we 4520want high-clearance trucks with four-wheel drive and front bumpers as big as battering rams? A large part of the answer lies in the fake Western names so many of them carry. No one much cares about what those names denote (lakes, frontier towns, mountain ranges); what 4525matters is their connotations of rugged individualism, mastery over the wilderness, cowboy endurance. The names simply magnify the appeal of these vehicles that are the Frankensteinian concoctions of our private anxieties and desires. Passage 2 When a major manufacturer launched an SUV named for an Alaskan mountain, an auto-trade publication discussed the subtleties of its name. It proposed that even though most buyers will never venture into territory any less trampled than the parking lot of the local shopping 4535mall, the important goal of the marketing hype is to plant the image in customers minds that they can conquer rugged terrain. Perhaps were trying to tame a different kind of wilderness. Indeed, in an age when many who can afford to do so live in limited-access communities 4540in houses guarded by sophisticated surveillance systems, the SUV is the perfect transportation shelter to protect us from fears both real and imagined.
4530

9. Passage 1 and Passage 2 both support which of the following generalizations about buyers of SUVs? (A) (B) (C) (D) (E) They intend to drive them on rough terrain. They wish to live in mountainous regions. They are wealthier than most other car buyers. They are influenced by marketing strategies. They are insecure about their social status.

10. Which of the following aspects of SUVs is addressed in Passage 1 but not in Passage 2 ? (A) (B) (C) (D) (E) Their imposing bulk Their escalating cost The psychology of their owners Their environmental impact The significance of their names

11. Which of the following in Passage 1 exemplifies the subtleties mentioned in Passage 2, line 15 ? (A) (B) (C) (D) (E) inexorable advance (line1) battering rams (line 4) lakes, frontier towns, mountain ranges (line 7) connotations (line 8) Frankensteinian concoctions (line 11)

12. Passage 1 and the article cited in Passage 2 both indicate that the imagery used to market SUVs is intended to (A) appeal to drivers primitive instincts (B) stir yearnings for a simpler way of life (C) engender feelings of power and control (D) evoke the beauty of unspoiled nature (E) create an aura of nonconformity

Questions 13-24 are based on the following passages. These two passages discuss different aspects of the impact of the First World War (1914-1918) on British people and society. Passage 1 is from a book that examines the depition of the war in literature, letters, and newspapers; Passage 2 is from a book that examines the differences between mens and womens experiences of war. Passage 1 Even if the civilian population at home had wanted to know the realities of the war, they couldnt have without 4545experiencing them: its conditions were too novel, its industrialized ghastliness too unprecedented. The war would have been simply unbelievable. From the very beginning a fissure was opening between the army and the civilians. The causes of civilian incomprehension were numerous. 4550Few soldiers wrote the truth in letters home for fear of causing needless uneasiness. If they did ever write the truth, it was excised by company officers, who censored all outgoing mail. The press was under rigid censorship throughout the war. Only correspondents willing to file 4555wholesome, optimistic copy were permitted to visit France, and even they were seldom allowed near the battlefields of the front line. Typical of these reporters was George Adam, Paris correspondent of the Times. His Behind the Scenes at the Front, published in 1915, exudes cheer, as well as warm 4560condescension, toward the common British soldier, whom he depicts as well fed, warm, safe, and happy better off, indeed, than at home. Lord Northcliffe, the publisher of the Times, eventually assumed full charge of government propaganda. It is no 4565surprise to find Northcliffes Times on July 3, 1916, reporting the first days attack during the battle of the Somme* with an airy confidence which could not help but deepen the division between those on the spot and those at home. Sir Douglas Haig telephoned last night, says the Times, that 4570the general situation was favorable. It soon ascends to the rhetoric of heroic romance: There is a fair field ... and we have elected to fight out our quarrel with the Germans and to give them as much battle as they want. No wonder communication failed between the troops and those who 4575could credit prose like that as factual testimony. * The British army had nearly 60,000 casualties, the largest number for
any single day in the armys history.

her soldier fianc that the war put a barrier of indescribable experience between men and women whom they loved. 4585Sometimes (I wrote at the time) I fear that even if he gets through, what he has experienced out there may change his ideas and tastes utterly. The nature of the barrier thrust between Vera Brittain and her fianc, however, may have been even more complex 4590than she herself realized, for the impediment preventing a marriage of their true minds was constituted not only by his altered experience but by hers. Specifically, as young men became increasingly alienated from their pre-war selves, increasingly immured in the muck and blood of the battlefields, 4595increasingly abandoned by the civilization of which they had ostensibly been heirs, women seemed to become, as if by some uncanny swing of historys pendulum, ever more powerful. As nurses, as munitions workers, as bus drivers, as soldiers in the agricultural land army," even 4600as wives and mothers, these formerly subservient reatures began to loom larger. A visitor to London observed in 1918 that England was a world of women women in uniforms. The wartime poems, stories, and memoirs by women 4605sometimes subtly, sometimes explicitly explore the political and economic revolution by which the First World War at least temporarily dispossessed male citizens of the primacy that had always been their birthright, while permanently granting women access to both the votes and the 4610professions that they had never before possessed. Similarly, a number of these women writers covertly or overtly celebrated the release of female desires and powers which that revolution made possible, as well as the reunion (or even reunification) of women which was a consequence of such 4615liberated energies. Their enthusiasm, which might otherwise seem like morbid gloating, was explained by Virginia Woolf, a writer otherwise known for her pacifist sympathies: How . .. can we explain that amazing outburst in August 1914, when the daughters of educated men ... rushed into hospitals ... drove lorries, worked in fields and munitions factories, and used all their immense stores of charm ... to persuade young men that to fight was heroic ... ? So profound was (womans) unconscious loathing for the education of the private house that she would undertake any task, however menial, exercise any fascination, however fatal, that enabled her to escape. Thus consciously she desired our splendid Empire; unconsciously she desired our splendid war.

4620

4625

Passage 2 The First World War is a classic case of the dissonance between official, male-centered history and unofficial female history. Not only did the apocalyptic events of this war have very different meanings for men and women, such events 4580were in fact very different for men and women, a point understood almost at once by an involved contemporary like Vera Brittain. She noted about her relationship with

130

4630

13. Passage 2 is unlike Passage 1 in that Passage 2 (A) (B) (C) (D) (E) describes war as dehumanizing endorses the official view of the war discusses war in terms of how it affects women tries to identify the root causes of the conflict criticizes the censorship of information about the war

4690

18. In Passage 2, the author mentions Vera Brittain (line 40) primarily to
4695

4635

464014. The fissure (line 6) was primarily caused by the

4645

(A) civilians ignorance about the soldiers experience (B) discrepancy between the experiences of men and of women (C) behavior of the officers who led the battles (D) guilt that civilians felt about sending young men off to war (E) special privileges granted to war correspondents 15. The footnote about the battle of the Somme adds information that (A) shows how history has been rewritten to glorify the war (B) trivializes the dangers faced by most of the soldiers (C) emphasizes the inaccuracy of the published reports (D) suggests that the costs of war outweighed its benefits (E) offers a journalists personal reflection on the war

4700

(A) support an argument by quoting material written at the time of the war (B) present an example of the kind of powers women gained during the war (C) describe how a writer manipulated the facts about the war (D) discuss the wartime literature produced by women (E) dispute recent historians views of the war 19. In line 58, the reference to wives and mothers most directly implies the authors assumption that

4705

4650

4710

4655

(A) families prospered more when women became head of the household (B) soldiers were unaware of the fundamental change taking place in society (C) women embraced their chance to work outside the home (D) women were anxious about fulfilling family responsibilities (E) women in domestic roles had previously exercised little authority 20. In line 64, the revolution refers to (A) womens literary output during the war (B) womens pursuit of rights previously unavailable to them (C) the change that men underwent after experiencing war (D) the redistribution of power from the upper to the middle class (E) the growing equalization of mens and womens wages 21. The author of Passage 2 implies that womens enthusiasm might ... seem like morbid gloating (lines 74-75) because

4715

466016. In Passage 1, the author suggests that the attitudes of those at home (line 26) were strongly influenced by

4720

4665

(A) the governments inadequate control over propaganda (B) the lack of opportunities for soldiers to write home (C) the disparity between mens and womens views of war (D) efforts of pacifists to end the war (E) censored reports from the press

4725

467017. In line 33, credit most nearly means

4675

(A) (B) (C) (D) (E)

award believe enter supply enrich

4730

4735

4680

(A) womens progress caused the deterioration of mens status (B) women achieved recognition as the real peacemakers in the war (C) women boasted that the war would be lost without them (D) women celebrated the fact that they did not have to fight in the war (E) women were enjoying power while men were in battle

4740

4685

474522. In lines 84-85, the discussion of womens involvement with menial tasks and fatal fascinations primarily serves to emphasize the

4805

24. Which of the following statements about the effect of the First World War is supported by both passages? (A) (B) (C) (D) Officers resented the governments complacency. Women gained independence in postwar Britain. Soldiers felt isolated from parts of civilian society. Writers failed in their attempts to describe the atrocities of war. (E) War proved an undesirable way to resolve the European conflict.

4750

4755

(A) far-reaching consequences of womens roles during wartime (B) extent to which women felt stifled in their traditional roles (C) contrast between how women idealized war and what it was really like (D) desire by women to escape the horrors of war (E) risks that women took to fight in the war 23. What do Behind the Scenes at the Front (lines 16-17) and wartime poems, stories and memoirs (line 62) have in common?

4810

4815

4760

4765

(A) Both caused needless uneasiness among civilians. (B) Both deliberately reflected the views of the government. (C) Both changed the status quo for women in wartime Britain. (D) Both encouraged writers to take their craft more seriously. (E) Neither focused on the realities of the battlefield.

4770

4775

4780

4785

4790

4795

4800

135 SECTION 9
4820

CRITICAL READING
19 Questions

Time 20 minutes

Directions: For each question in this section, select the best answer from among the choices given and fill in the corresponding circle on the answer sheet.

Each sentence below has one or two blanks, each blank indicating that something has been omitted. Beneath the sentence are five words or sets of words labeled A through E. Choose the word or set of words that, when inserted in the sentence, best fits the meaning of the sentence as a whole. Example: Hoping to ------- the dispute, negotiators proposed a compromise that they felt would be ------- to both labor and management. (A) (B) (C) (D) (E)
4825

4855

3. His peers respected him because he was both -----and -------: steadfast in his beliefs and tactful in his negotiations. (A) (B) (C) (D) (E) resourceful .. courteous tenacious .. manipulative determined .. demonstrative resolute .. diplomatic outspoken .. indiscriminate

4860

enforce .. useful end .. divisive overcome .. unattractive extend .. satisfactory resolve .. acceptable

48654. Considering that many women had little control over their own lives in medieval England, Margery Kempes fifteenth-century autobiography demonstrates a remarkable degree of -------. 4870

(A) (C) (E) (B) (D)

consecration autonomy simplicity rationalism effacement

1. As sea urchins are becoming scarcer, divers are -----to more dangerous depths to retrieve them, ------- the potential for diving injuries.
4830

(A) (B) (C) (D) (E)

swimming .. lessening descending .. increasing removing .. avoiding returning .. seeing climbing .. creating

48755. Following the decree banning ------- acts, suspected ------- could be forcibly detained without the filing of formal charges.

4880

4835

(A) rebellious .. conformists (B) apolitical .. loyalists (C) seditious .. insurrectionists (D) subversive .. nonpartisans (E) supportive .. opponents 6. By portraying a wide spectrum of characters in his one-man show, John Leguizamo provides a ------- to the theaters tendency to offer a limited range of roles to Latino actors. (A) corrective (D) stimulus (B) tribute (C) corollary (E) precursor

2. Anne mentioned Johns habitual boasting about his wardrobe as an example of his ------- ways. (A) erratic (D) tactful (B) egotistical (C ) flexible (E) inconspicuous

4885

4840

4890
4845

4850

The passage below is followed by questions based on its content. Answer the questions on the basis of what is stated or implied in the passage and in any introductory material that may be provided.
4895

Questions 7-19 are based on the following passage. The following passage, set in the early 1970s, is from a 1992 novel. The principal characters, Virginia and Clayton, are two cellists in a college orchestra. Shed met lots of crazy musicians, but no one like Clayton. He was as obsessed as the others, but he had a quirky sense of humor, a slow ironic counterpoint to his own beliefs. And he didnt look quite like anyone else. 4900He wore his hair parted dangerously near the middle and combed it in little ripples like Cab Calloway,1 though sometimes he let it fly up a bit at the ends in deference to the campus pressure for Afros. His caramel-colored skin darkened to toffee under fluorescent light but sometimes 4905took on a golden sheen, especially in the vertical shafts of sunlight that poured into his favorite practice room where shed often peek in on him an uncanny complexion, as if the shades swirled just under the surface. Virginias friends gave her advice on how to get him. 4910You two can play hot duets together, they giggled. As it turned out, she didnt have to plan a thing. She was reading one afternoon outside the Fine Arts Building when the day suddenly turned cold. If she went back to the dorm for a sweater, shed be late for orchestra rehearsal. So she 4915stuck it out until a few minutes before rehearsal at four. By that time, her fingers were so stiff she had to run them under hot water to loosen them up. Then she hurried to the cello room, where all the instruments were lined up like novitiates;2 she felt a strange reverence every time she 4920stepped across the threshold into its cool serenity. There they stood, obedient yet voluptuous in their molded cases. In the dim light their plump forms looked sadly human, as if they were waiting for something better to come along but knew it wouldnt. 4925 Virginia grabbed her cello and was halfway down the hall when she realized shed forgotten to leave her books behind. She decided against turning back and continued to the basement, where the five-till-four pandemonium was breaking loose. Clayton was stuffing his books into his 4930locker. Hey, Clayton, hows it going? As if it were routine, he took her books and wedged them in next to his. They started toward the orchestra hall. Virginia cast a surreptitious glance upward; five minutes 4935to four or not, Clayton was not rushing. His long, gangling frame seemed to be held together by molasses; he moved deliberately, negotiating the crush while humming a tricky passage from Schumann,3 sailing above the mob. After rehearsal she reminded him that her books were in 4940his locker. I think Ill go practice, he said. Would you like to listen? Ill miss dinner, she replied, and was about to curse herself for her honesty when he said, I have cheese and 4945soup back at the fraternity house, if you dont mind the walk. The walk was twenty minutes of agonizing bliss, with the wind off the lake whipping her blue, and Clayton too involved with analyzing the orchestras horn section to 4950notice. When they reached the house, a brick building with a crumbling porch and weeds cracking the front path, she was nearly frozen through. He heated up a can of soup, and plunked the cheese down in the center of the dinette table. Its not much, he apologized, but she was thinking 4955A loaf of bread, a jug of wine, 4 and felt sated before lifting the first spoonful. The house was rented to Alpha Phi Alpha, one of three Black fraternities on campus. It had a musty tennis-shoes-and-ripe-Iaundry smell. Books and jackets were strewn everywhere, dishes piled in the sink. 4960 When did you begin playing? she asked. I began late, Im afraid, Clayton replied. Ninth grade. But I felt at home immediately. With the music, I mean. The instrument took a little longer. Everyone said I was too tall to be a cellist. He grimaced. 4965 Virginia watched him as he talked. He was the same golden brown as the instrument, and his mustache followed the lines of the cellos scroll. So what did you do? she asked. Whenever my height came up, I would say, Remember 4970the bumblebee. What do bumblebees have to do with cellos? The bumblebee, aerodynamically speaking, is too large for flight. But the bee has never heard of aerodynamics, so it flies in spite of the laws of gravity. I merely wrapped my 4975legs and arms around the cello and kept playing. Music was the only landscape in which he seemed at ease. In that raunchy kitchen, elbows propped on either side of the cooling soup, he was fidgety, even a little awkward. But when he sat up behind his instrument, he had the Irresistible 4980beauty of someone who had found his place.
1 2 3 4

American jazz musician and bandleader (1907 -1994) Persons who have entered a religious order but have not yet taken final vows German composer (1810-1856) A reference to Edward Fitzgeralds A jug of wine, a loaf of bread, and thou, a line from The Rubaiyat of Omar Khayyam

5040

7. The passage is best described as


4985

4990

(A) a social commentary on classical musicians in the early 1970 s (B) a nostalgic depiction of students in college orchestras (C) a story of how one individual inspired many others (D) an introduction to a character through the perspective of another character (E) an illustration of a strained but enduring relationship

12. In line 42, crush most nearly means


5045

(A) (B) (C) (D) (E)

pressure crowd power infatuation critical condition

505013. In lines 42-43, the reference to Claytons humming creates an impression that he is

8. The references to fluorescent light (line 9) and 4995 sunlight (line 11) suggest the (A) way that Claytons demeanor brightened when Virginia was nearby (B) contrast between Claytons restraint and Virginias passion (C) attentiveness with which Virginia regarded Clayton (D) monotony of Claytons everyday routine (E) superficiality of Claytons beauty 9. The imagery in lines 12-13 (as if .... surface) conveys which of the following about Clayton? (A) (B) (C) (D) (E) His complicated nature His erratic reactions His unseemly complacency His passionate loyalty His tendency to argue
5065 5055

(A) (B) (C) (D) (E)

uncomfortable with making conversation amused by the plight of other musicians unaware of his effect on other people compelled to show off his talent immersed in his private world

5000

14. In the context of the passage, Claytons statement in line 46 (I think ... practice) emphasizes his
5060

5005

(A) (B) (C) (D) (E)

need to make a dramatic first impression willingness to disrupt a fixed routine consuming interest in music distaste for competition insecurity around other musicians

15. In line 52, the phrase agonizing bliss suggests that Virginias pleasure is tempered by (A) (B) (C) (D) (E) Claytons cold manner Claytons visible uneasiness her physical discomfort her overriding self-consciousness her sense that the happiness would be short-lived

5010

10. As contrasted with the language in the opening paragraph, the advice offered by Virginias friends (lines 14-15) functions primarily to
5015

5070

(A) (B) (C) (D) (E)

break the mood of abstracted musing introduce an element of foreboding poke fun at the pretensions of romantic music contradict Virginias opinion of Clayton counter Claytons offbeat sense of humor

5075

16. In lines 59-61, Virginias reaction to the meal most directly suggests that she (A) was contented enough without the food (B) was amused by Claytons attempts at hospitality (C) was suddenly aware of the chaos in the fraternity kitchen (D) felt guilty about making Clayton uneasy (E) did not find the atmosphere conducive to romance 17. The description in lines 70-72 emphasizes how

5020 11. As described in lines 22-25, the atmosphere in the cello room is most nearly one of

5080

5025

(A) (B) (C) (D) (E)

creativity emptiness urgency sanctity accomplishment

5085

5030

(A) (B) (C) (D) (E)

strikingly Clayton resembles fatuous musicians awkwardly Clayton behaves in social situations profoundly Clayton is affected by music closely Virginia associates Clayton with his cello strongly Virginia identifies with Claytons situation

5090
5035

140

18. In referring to the bumblebee (lines 77-80), Clayton 5095 conveys his (A) (B) (C) (D) (E) superstitious nature cunning instincts frail pride resolute determination volatile temperament

5155

19. In lines 82-85, the descriptions of Clayton in the kitchen and Clayton behind his instrument present a contrast between his (A) (B) (C) (D) (E) chaos and organization mediocrity and excellence pretension and genuineness laziness and dedication clumsiness and gracefulness

5160

5100

5165
5105

5110

5115

5120

5125

5130

5135

5140

5145

5150

SECTION 10
5170

WRITING

Time 10 minutes 14 Questions

Directions: For each question in this section, select the best answer from among the choices given and fill in the corresponding circle on the answer sheet.

The following sentences test correctness and effectiveness of expression. Part of each sentence or the entire sentence is underlined; beneath each sentence are five ways of phrasing the underlined material. Choice A repeats the original phrasing; the other four choices are different. If you think the original phrasing produces a better sentence than any of the alternatives, select choice A; if not, select one of the other choices. In making your selection, follow the requirements of standard written English; that is, pay attention to grammar, choice of words, sentence construction, and punctuation. Your selection should result in the most effective sentence clear and precise, without awkwardness or ambiguity. EXAMPLE: Laura Ingalls Wilder published her first book and she was sixty-five years old then. (A) and she was sixty-five years old then (B) when she was sixty-five (C) at age sixty-five years old (D) upon the reaching of sixty-five years (E) at the time when she was sixty-five

5195

2. The new bird sanctuary, consisting of one hundred acres of unspoiled tideland, and is protected by the state. (A) sanctuary, consisting of one hundred acres of unspoiled tideland, and is protected by the state (B) sanctuary is protected by the state, it consists of one hundred acres of unspoiled tideland (C) sanctuary, consisting of one hundred acres of unspoiled tideland, is protected by the state (D) sanctuary is protected by the state consisting of one hundred acres of unspoiled tideland (E) sanctuary to consist of one hundred acres of unspoiled tideland and to be protected by the state 3. Most people know about calories and nutrition, but they do not use this knowledge to lose weight permanently and keep it off. (A) (B) (C) (D) (E) permanently and keep it off permanent and have it stay off and have it be off permanently and make it permanent and keep it off permanently

5200

5205

5210

5215

5175
1. At Versailles after the First World War, the Allies believed they had drafted a treaty that would have ensured permanent peace.
5180

52204. No sooner had Andrea del Sarto traveled to France to work for the French king but his wife persuaded him to return to Italy.

5225

(A) (B) (C) (D) (E)

would have ensured permanent peace would ensure permanent peace had ensured permanent peace will ensure permanent peace ensures permanent peace

(A) but his wife persuaded him to return (B) but his wife had him persuaded into returning (C) than he was persuaded by his wife that he will return (D) but he was persuaded by his wife into returning (E) than his wife persuaded him to return

5230

5185

5190

145
5. During the 1980s and early 1990s, one reason highways in the United States became safer than ever, 5235 the use of seat belts increased to about 67 percent nationwide. (A) ever, the use of seat belts increased to about 67 percent nationwide (B) ever, nationwide, the use of seat belts increased to about 67 percent (C) ever, there was a nationwide increase in seat belt use to 67 percent (D) ever since they increased seat belt use to 67 percent nationwide (E) ever was that the use of seat belts nationwide increased to about 67 percent 9. One of the first people to recognize the talent of Langston Hughes, Jessie Fauset. was an editor 5295 at Crisis magazine, publishing Hughess poetry in 1921. (A) Jessie Fauset, was an editor at Crisis magazine, publishing (B) Jessie Fauset who edited Crisis magazine and published (C) Jessie Fauset edited Crisis magazine who published (D) Jessie Fauset, an editor at Crisis magazine, published (E) the editor, Jessie Fauset, published at Crisis magazine

5240

5300

5245

5305

6. Chaplin will not be remembered for espousing radical causes any more than they will remember Wayne for 5250 endorsing conservative political candidates. (A) (B) (C) (D) (E) any more than they will remember Wayne as will Wayne not be remembered any more than Wayne will be remembered just as they will not remember Wayne no more than Wayne will be remembered

10. Until being widely hunted for its ivory and blubber in the eighteenth century, walruses were plentiful in the 5310 waters of the northeastern United States. (A) (B) (C) (D) (E) Until being widely hunted for its Before having been widely hunted for its Up to them being widely hunted for their Until they were widely hunted for their Before they have been widely hunted for their

5255

5315

7. Civil rights leader and author W. E. B. Du Bois was interested in drama because he believed that if you represented historical events on stage it could have a 5260 greater, more lasting effect than any exhibit or lecture. (A) if you represented historical events on stage it (B) with the events of history represented on stage they (C) events which were represented historically on stage (D) by representing historical events on stage (E) representing historical events on stage

11. Jesse passed the California bar examination last year, and he has been practicing law in California ever since.
5320

(A) and he has been practicing law in California ever since (B) since that time he has practiced law there (C) where ever since he practices law (D) he has been practicing law in California since then (E) and since then is practicing law there 12. Persuading even the queasiest of readers to spend hours learning about an extravagant variety of invertebrates, the effect of Richard Conniffs Spineless Wonders is to render the repulsive beautiful. (A) the effect of Richard Conniffs Spineless Wonders is to render the repulsive beautiful (B) Richard Conniff renders the repulsive beautiful in Spineless Wonders (C) the effect of Spineless Wonders, by Richard Conniff, is to render the repulsive beautiful (D) Richard Conniff has had the effect of rendering the repulsive beautiful in Spineless Wonders (E) Spineless Wonders, by Richard Conniff, has effect in rendering the repulsive beautiful

5265

5325

8. Many colleges are adopting work-study programs. which offer practical advantages to both the students 5270 and the institutions. (A) programs, which offer practical advantages to both the students and (B) programs, which offers practical advantages to both the students and (C) programs, which offer both practical advantages to the students plus (D) programs; it offers practical advantages to both the students as well as (E) programs; this offers practical advantages both to the students and

5330

5275

5335

5280

5340

5285

5290

13. Most drivers know not only that excessive speeding on 5345 highways wastes gasoline, but also that it is dangerous. (A) know not only that excessive speeding on highways wastes (B) know that excessive speeding on highways could be wasteful of (C) are knowledgeable that excessive speeding on highways not only wastes (D) have known that excessive speeding on highways wastes not only (E) know that excessive speeding on highways not only by itself can waste

5405

14. Walt Disneys first success was his third Mickey Mouse film, in which Disney produced a cartoon with sound, and Mickey was made to talk. (A) (B) (C) (D) (E) sound, and Mickey was made to talk sound and making Mickey talk sound, with the result being Mickey talking sound in where Mickey talks sound and made Mickey talk

5410

5350

5355

5360

5365

5370

5375

5380

5385

5390

5395

5400

Practice Test 4 5415 SECTION 1 WRITING ESSAY

Time 25 minutes

5420The essay gives you an opportunity to show how effectively you can develop and express ideas. You should, therefore, take care to develop your point of view, present your ideas logically and clearly, and use language precisely.

Your essay must be written on the lines provided on your answer sheet you will receive no other paper on which to write. 5425You will have enough space if you write on every line, avoid wide margins, and keep your handwriting to a reasonable size. Remember that people who are not familiar with your handwriting will read what you write. Try to write or print so that what you are writing is legible to those readers. Important Reminders:
5430

5435

A pencil is required for the essay. An essay written in ink will receive a score of zero. Do not write your essay in your test book. You will receive credit only for what you write on your answer sheet. An off-topic essay will receive a score of zero. If your essay does not reflect your original and individual work, your test scores may be cancelled.

You have twenty-five minutes to write an essay on the topic assigned below.

5440Think carefully about the issue presented in the following excerpt and the assignment below.

Given the importance of human creativity, one would think it should have a high priority among our concerns. But if we look at the reality, we see a different picture. Basic scientific research is minimized in favor of immediate practical applications. The arts are increasingly seen as dispensable luxuries. Yet as competition heats up around the globe, exactly the opposite strategy is needed. Adapted from Mihaly Csikszentmihalyi, Creativity: Flow and the Psychology of Discovery and Invention

5445Assignment: Is creativity needed more than ever in the world today? Plan and write an essay in which you develop your point of view on this issue. Support your position with reasoning and examples taken from your reading, studies, experience, or observations..
5450

150

Practice Test 4

5455

SECTION 3

WRITING

Time -25 minutes 35 Question


Directions: For each question in this section, select the best answer from among the choices given and fill in the corresponding circle on the answer sheet.

The following sentences test correctness and effectiveness of expression. Part of each sentence or the entire sentence is underlined; beneath each sentence are five ways of phrasing the underlined material. Choice A repeats the original phrasing; the other four choices are different. If you think the original phrasing produces a better sentence than any of the alternatives, select choice A; if not, select one of the other choices. In making your selection, follow the requirements of standard written English; that is, pay attention to grammar, choice of words, sentence construction, and punctuation. Your selection should result in the most effective sentence clear and precise, without awkwardness or ambiguity. EXAMPLE: Laura Ingalls Wilder published her first book and she was sixty-five years old then. (A) and she was sixty-five years old then (B) when she was sixty-five (C) at age sixty-five years old (D) upon the reaching of sixty-five years (E) at the time when she was sixty-five

Claude McKay writing 54802. Many ancient Eastern rulers favored drinking vessels made of celadon porcelain because of supposedly revealing the presence of poison by cracking. (A) because of supposedly revealing the presence of poison (B) for being supposed that it would reveal the presence of poison (C) because of being supposed to reveal poison in it (D) for it was supposed to reveal that there is poison (E) because it was supposed to reveal the presence of poison

5485

5490

3. John believes that plants respond to human attention, which causes his talking to his African violets every 5495 night. (A) (B) (C) (D) (E) attention, which causes his talking attention and talking is what is done attention and his talks attention; for this reason has been his talking attention; he therefore talks

5500

4. All the demands on soprano Kathleen Battle for operatic performances, solo concerts, and special guest appearances, tempting her to sing too often and 5505 straining her voice. (A) appearances, tempting her to sing too often and straining (B) appearances not only tempt her to sing too often plus they strain (C) appearances tempts her not only into singing too often but then she strains (D) appearances, tempting her into singing too often and she therefore strains (E) appearances tempt her to sing too often and strain

5460
1. The poet Claude McKay was a native of Jamaica who spent most of his life in the United States but writing some of his poems in the Jamaican dialect.
5465

5510

5470

5475

(A) The poet Claude McKay was a native of Jamaica who spent most of his life in the United States but writing (B) Being that he was a Jamaican who spent most of his life in the United States, the poet Claude McKay writing (C) Although a native of Jamaica, the poet Claude McKay spent most of his life in the United States, he wrote (D) Although the poet Claude McKay spent most of his life in the United States, he was a native of Jamaica and wrote (E) Because he was a native of Jamaica who spent most of his life in the United States, the poet

5515

5520

5. One reason that an insect can walk on walls while a human cannot is that the mass of its tiny body is far 5525 lower than humans. (A) (B) (C) (D) (E) far lower than humans far lower than that of a humans body lower by far than humans far lower than a human far lower than is a humans body

9. The African tsetse fly does not need a brain, everything it has to do in life is programmed into its nervous 5585 system. (A) (B) (C) (D) (E) brain, everything brain due to everything which brain, for everything brain; since, everything brain whereas everything

5530

5590

6. In the 1980s, the median price of a house more than doubled, generally outdistancing the rate of inflation.
5535

10.

(A) generally outdistancing the rate of inflation (B) generally this outdistanced the rate of inflation (C) and the result was the general outdistancing of inflation (D) the general rate of inflation was thus outdistanced (E) thus generally inflation had been outdistanced 7. In the nineteenth century, reproductions of cathedrals or castles made entirely of ice was often a popular feature in North American winter carnivals. (A) (B) (C) (D) (E) was often a popular feature often were popular features often was featured popularly often being popular features have been featured popularly

She was concerned about how Hank would react to the incident, but in searching his face, he did not seem to be at all embarrassed or troubled. (A) in searching his face, he did not seem to be (B) by searching his face, it showed that he was not (C) a search of his face showed that he seemed not (D) searching his face, he did not seem to be (E) his face being searched showed that he was not Explaining modern art is impossible, partly because of its complexity but largely because of it rapidly changing. (A) (B) (C) (D) (E) of it rapidly changing it makes rapid changes of the rapidity with which it changes changing it is rapid it changes so rapid

5595

5540

5600

11.
5605

5545

55508. A fine orchestral performance will exhibit the skills of the musicians, their abilities to work as an ensemble, and how he or she responds to the conductor.

5610

5555

(A) (B) (C) (D) (E)

how he or she responds how to respond their responding their responses they respond

5560

5565

5570

5575

5580

155
5645

The following sentences test your ability to recognize grammar and usage errors. Each sentence contains either a single error or no error at all. No sentence contains more than one error. The error, if there is one, is underlined and lettered. If the sentence contains an error, select the one underlined part that must be changed to make the sentence correct. If the sentence is correct, select choice E. In choosing answers, follow the requirements of standard written English. EXAMPLE: The other delegates and him immediately A B C accepted the resolution drafted by the D neutral states. No error E

15. Twenty-five years after Alex Haleys Roots stimulate A many people to research their family histories, new
5650

C technology has been developed to make the task easier . No error D E

565516. For months the press had praised Thatchers handling

of the international crisis, and editorial views changed A quickly when the domestic economy worsened .
5660

B No error E

5615
12. The ambassador was entertained lavish by A Hartwright, whose company has a monetary
5620 5665

17. Experiments have shown that human skin provides A natural protection against a surprising large C number of infectious bacteria. No error D
5670

B D new country. No error E

interest in the industrial development of the

5625

18. In the aggressive society created by William Golding 13. Among the discoveries made possible by A C B D the invention of the telescope they found that
5675early on as the leader of the lost boys. No error

A in Lord of the Flies, both Ralph and Jack emerge

5630dark spots existed on the Sun in varying numbers.

No error E
563514. This liberal arts college has decided requiring

19. More than forty years have passed since a quarter A B


5680of a million people marched on Washington, D.C.,

A C language. No error
5640

in an attempt to secure civil rights for Black C


5685

all students to study at least one non-European D

D E

Americans. No error

569020. Careful analysis of pictures of the Moon reveal that

5735

25. The famous filmmaker had a tendency of changing A C D B his recollections, perhaps out of boredom at having

A parts of the Moons surface are markedly similar to B parts of the Earths . No error
5695

5740to tell interviewers the same story over and over.

E No error E
574526. Norwegian writer Sigrid Undset is like the novelist Sir

21. London differs from other cities, such as Paris and A


5700

B D B unlike his books , she dwells on the psychological


5750

New York, in that its shopping areas are so widely C spread out. No error E 22. The architects research shows that even when builders
5705

A Walter Scott in her use of historical backgrounds, but

C aspects of her characters. No error D E

construct houses of stone , they still use the hammer A D


5710

B E

C 27. The television station has received many complaints


5755

more than any tool. No error

A about the clothing advertisements, which some B D E C

23. Of the two options, neither the system of appointing A B C


5715are entirely satisfactory. No error 5760

viewers condemn to be tasteless. No error

judges to the bench nor the process of electing judges

28. The relationship between goby fish and striped shrimp are truly symbiotic , for neither can survive without

E A
5765the other. No error

24. Carlos cherished the memory of the day when him A


5720and his sister Rosa were presented with awards

B E

B in recognition of meritorious service to the C community. No error


5725

29. Winston Churchill, unlike many English prime A


5770ministers before him , had deep insight into the

D E D B

C E

workings of the human mind. No error

5730

5775

Directions: The following passage is an early draft of an essay. Some parts of the passage need to be rewritten. Read the passage and select the best answers for the questions that follow. Some questions are about particular sentences or parts of sentences and ask you to improve sentence structure or word choice. Other questions ask you to consider organization and development. In choosing answers, follow the requirements of standard written English. Questions 30-35 are based on the following passage. (1) My father has an exceptional talent. (2) The ability to understand people. (3) When I have a problem that I think no one else will understand, I take it to my father. (4) He listens intently, asks me some questions, and my feelings are seemingly known by him exactly. 5785(5) Even my twin sister can talk to him more easily than to me. (6) Many people seem too busy to take the time to understand one another. (7) My father, by all accounts, sees taking time to listen as essential to any relationship, whether it involves family, friendship, or 5790work. (8) At work, my fathers friends and work associates benefit from this talent. (9) His job requires him to attend social events and sometimes I go along. (10) I have watched him at dinner; his eyes are fixed on whoever is 5795speaking, and he nods his head at every remark. (11) My father emerges from such a conversation with what I believe is a true sense of the speakers meaning. (12) In the same way, we choose our friends. (13) My fathers ability to listen affects his whole 5800life. (14) His ability allows him to form strong relationships with his coworkers and earns him lasting friendships. (15) It allows him to have open conversations with his children. (16) Furthermore, it has strengthened his relationship with my mother. 5805(17) Certainly, his talent is one that I hope to develop as I mature.
5780 5840 5830

30. Of the following, which is the best way to revise and combine sentences 1 and 2 (reproduced below) ? My father has an exceptional talent. The ability to understand people. (A) My father has an exceptional talent and the ability to understand people. (B) My father has an exceptional talent that includes the ability to understand people. (C) My father has an exceptional talent: the ability to understand people. (D) My father has an exceptional talent, it is his ability to understand people. (E) Despite my fathers exceptional talent, he still has the ability to understand people. 31. Of the following, which is the best way to phrase sentence 4 (reproduced below) ?
5845

5835

He listens intently, asks me some questions, and my feelings are seemingly known by him exactly. (A) (As it is now) (B) Listening intently, he will ask me some questions and then my exact feelings are seemingly known to him. (C) As he listens to me and asks me some questions, he seems to be knowing exactly my feelings. (D) He listened to me and asked me some questions, seeming to know exactly how I felt. (E) He listens intently, asks me some questions, and then seems to know exactly how I feel. 32. In sentence 7, the phrase by all accounts is best replaced by

5850

5855

5860

(A) (B) (C) (D) (E)

however moreover to my knowledge like my sister but nevertheless

5865

33. Which of the following sentences should be omitted to improve the unity of the second paragraph? (A) (B) (C) (D) (E) Sentence 8 Sentence 9 Sentence 10 Sentence 11 Sentence 12

5810 5870

5815

5820

5825

160

5935 5875

34. In context, which of the following is the best way to phrase the underlined portion of sentence 16 (reproduced below) ? Furthermore, it has strengthened his relationship with my mother. (A) (B) (C) (D) (E) (As it is now) Further strengthening But it strengthens However, he is strengthening Considering this, he strengthens
5940

35. A strategy that the writer uses within the third paragraph is to (A) (B) (C) (D) (E) make false assumptions and use exaggeration include difficult vocabulary repeat certain words and sentence patterns argue in a tone of defiance turn aside from the main subject

5880

5945

5885

5890

5895

5900

5905

5910

5915

5920

5925

5930

SECTION 4
5950

CRITICAL READING
23 Questions

Time 25 minutes

Directions: For each question in this section, select the best answer from among the choices given and fill in the corresponding circle on the answer sheet.

5955
Each sentence below has one or two blanks, each blank indicating that something has been omitted. Beneath the sentence are five words or sets of words labeled A through E. Choose the word or set of words that, when inserted in the sentence, best fits the meaning of the sentence as a whole. Example: Hoping to ------- the dispute, negotiators proposed a compromise that they felt would be ------- to both labor and management. (A) (B) (C) (D) (E) enforce .. useful end .. divisive overcome .. unattractive extend .. satisfactory resolve .. acceptable
5985

2. Nations that share a border are, by definition, -------. (A) allied (B) partisan (C) contiguous (D) pluralistic (E ) sovereign

59903. Much of this authors work, unfortunately, is -------, with ------- chapter often immediately following a sublime one.

5995

(A) (B) (C) (D) (E)

mystical . . a superior uneven . . a mediocre predictable . . an eloquent enthralling . . a vapid flippant . . an intelligible

6000

4. In young children, some brain cells have a ------- that enables them to take over the functions of damaged or missing brain cells. (A) fragility (B) reminiscence (C) perniciousness (D) whimsicality (E) plasticity

1. Scientific discoveries are often thought of as the result of ------- effort, but many discoveries have, in fact, 5960 arisen from ------- or a mistake. (A) (B) (C) (D) (E) conscientious . . a method incidental . . a mishap collaborative . . a design persistent . . an extension systematic . . an accident

6005

5. Less government spending is ------- of this political party, a belief shared by most party members. (A) an acronym (B) a retraction (D) a plight (E) a prospectus (C) a tenet

5965

6010

5970

5975

5980

165
The passages below are followed by questions based on their content; questions following a pair of related passages may also be based on the relationship between the paired passages. Answer the questions on the basis of what is stated or implied in the passages and in any introductory material that may be provided.

6015

Questions 6-7 are based on the following passage. Duke Ellington considered himself the worlds greatest listener. In music, hearing is all. Judging by the two or three thousand pieces of music Ellington wrote, he could probably hear a flea scratching itself and put that rhythm 6020into one of his compositions. 5 For him the sounds of the world were the ingredients he mixed into appetizers, main courses, and desserts to satisfy the appetite of his worldwide audience. He wasnt averse to going out in a boat to catch the fish himself. He would raise the fowl 6025himself. But when that musical meal appeared before you none of the drudgery showed. 6. The author most likely refers to the flea in line 4 in order to (A) highlight Ellingtons prodigious memory (B) emphasize the quality of Ellingtons listening skills (C) indicate Ellingtons interest in different animal sounds (D) suggest that Ellingtons compositions were marked by rhythmic similarities (E) imply that Ellington could be overly concerned about minutia 7. In lines 5-11 (For him . . . drudgery showed), the authors point is primarily developed through the use of (A) (B) (C) (D) (E) comparison and contrast appeal to emotion exaggeration metaphor humor

Questions 8-9 are based on the following passage. In the summer of 1911, the explorer Hiram Bingham III bushwhacked his way to a high ridge in the Andes of Peru and beheld a dreamscape out of the past. There, set against 6030looming peaks cloaked in snow and wreathed in clouds, was Machu Picchu, the famous lost city of the Incas. This expression, popularized by Bingham, served as a magical elixir for rundown imaginations. The words evoked the romanticism of exploration and archaeology 6035at the time. But finding Machu Picchu was easier than solving the mystery of its place in the rich and powerful Inca empire. The imposing architecture attested to the skill and audacity of the Incas. But who had lived at this isolated site and for what purpose? 8. The words magical elixir (line 18) primarily emphasize the (A) (B) (C) (D) (E) motivation for an expedition captivating power of a phrase inspiration behind a discovery creative dimension of archaeology complexity of an expression

9. The mystery discussed in lines 21-24 is most analogous to that encountered in which of the following situations? (A) Being unable to locate the source of materials used to construct an ancient palace (B) Being unable to reconcile archaeological evidence with mythical descriptions of an ancient city (C) Being unable to explain how ancient peoples constructed imposing monuments using only primitive technology (D) Being unable to understand the religious function of a chamber found inside an ancient temple (E) Being unable to discover any trace of a civilization repeatedly mentioned by ancient authors

Questions 10-14 are based on the following passage. This passage is from the preface to a 1997 book by a United States journalist detailing a disagreement between doctors and family members about a childs medical treatment at a hospital in California. Under my desk I keep a large carton of cassette tapes. Though they have all been transcribed, I still like to listen to them from time to time. Some are quiet and easily understood. They are filled with the voices of American doctors, interrupted occasion6045ally by the clink of a coffee cup or beep of a pager. The restmore than half of themare very noisy. They are filled with the voices of the Lees family, Hmong refugees from Laos who came to the United States in 1980. Against a background of babies crying, children playing, doors 6050slamming, dishes clattering, a television yammering, and an air conditioner wheezing, I can hear the mothers voice, by turns breathy, nasal, gargly, or humlike as it slides up and down the Hmong languages eight tones; the fathers voice, louder, slower, more vehement; and my interpreters voice, 6055mediating in Hmong and English, low and deferential in each. The hubbub summons sense-memories: the coolness of the red metal folding chair, reserved for guests, that was always set up when I arrived in the apartment; the shadows cast by the amulet that hung from the ceiling and swung in 6060the breeze on its length of grocers twine; the tastes of Hmong food. I sat on the Lees red chair for the first time on May 19, 1988. Earlier that spring I had come to Merced, California, because I had heard that there were some 6065misunderstandings at the county hospital between its Hmong patients and medical staff. One doctor called them collisions, which made it sound as if two different kinds of people had rammed into each other, head on, to the accompaniment of squealing brakes and breaking glass. 6070As it turned out, the encounters were messy but rarely frontal. Both sides were wounded, but neither side seemed to know what had hit it or how to avoid another crash. I have always felt that the action most worth watching occurs not at the center of things but where edges meet. 6075I like shorelines, weather fronts, international borders. These places have interesting frictions and incongruities, and often, if you stand at the point of tangency, you can see both sides better than if you were in the middle of either one. This is especially true when the apposition is cultural. 6080When I first came to Merced, I hoped that the culture of American medicine, about which I knew a little, and the culture of the Hmong, about which I knew nothing, would somehow illuminate each other if I could position myself between the two and manage not to get caught in the cross6085fire. But after getting to know the Lees family and their
6040

daughters doctors and realizing how hard it was to blame anyone, I stopped analyzing the situation in such linear terms. Now, when I play the tapes late at night, I imagine what they would sound like if I could splice them together, 6090so the voices of the Hmong and those of the American doctors could be heard on a single tape, speaking a common language. 10. In line 17, summons most nearly means (A) (B) (C) (D) (E) sends for calls forth requests orders convenes

11. It can be inferred from lines 27-33 that collisions was NOT an apt description because the (A) clash between Hmong patients and medical staff was indirect and baffling (B) Hmong patients and the medical staff were not significantly affected by the encounters (C) medical staff was not responsible for the dissatisfaction of the Hmong patients (D) misunderstandings between the Hmong patients and the medical staff were easy to resolve (E) disagreement reached beyond particular individuals to the community at large 12. Which of the following views of conflict is best supported by lines 37-40 (These . . . one) ? (A) Efforts to prevent conflicts are not always successful. (B) Conflict can occur in many different guises. (C) In most conflicts, both parties are to blame. (D) You can understand two parties that have resolved their conflicts better than two parties that are currently in conflict. (E) You can learn more about two parties in conflict as an observer than as an involved participant.

6155 609513. According to lines 41-46 (When I . . . crossfire), the authors initial goal was to

14. At the end of the passage, the author suggests that it would be ideal if the
6160

6100

6105

(A) consider the perspectives of both the American doctors and the Lees family to see what insights might develop (B) serve as a counselor to the county hospitals Hmong patients in order to ease their anxieties (C) work out a compromise between the American doctors and the Lees family (D) acquire a greater knowledge of how the American medical culture serves patients (E) try to reduce the misunderstandings between the American doctors and the Lees family and promote good will

6165

(A) differences between the Lees family and the American doctors could be resolved quickly (B) concerns and opinions of the Lees family and the American doctors could be merged (C) American doctors could take the time to learn more about their Hmong patients (D) Hmong patients could become more vocal in defense of their rights (E) Hmong patients could get medical treatment consistent with their cultural beliefs

6170

6110

6115

6120

6125

6130

6135

6140

6145

6150

170

Questions 15-23 are based on the following passages. Cloning is the creation of a new individual from the unique DNA (or genetic information) of another. The successful cloning of a sheep named Dolly in 1997 sparked a debate over the implications of cloning humans. Each of the passages below was written in 1997. Passage 1 Cloning creates serious issues of identity and individuality. The cloned person may experience concerns about his or her distinctive identity, not only because the person will 6175be in genotype (genetic makeup) and appearance identical to another human being, but, in this case, because he or she may also be twin to the person who is the father or motherif one can still call them that. What would be the psychic burdens of being the child or parent of your 6180twin? The cloned individual, moreover, will be saddled with a genotype that has already lived. He or she will not be fully a surprise to the world. People will likely always compare a clones performance in life with that of the original. True, a cloned 6185persons nurture and circumstances in life will be different; genotype is not exactly destiny. Still, one must also expect parental and other efforts to shape this new life after the originalor at least to view the child with the original vision always firmly in mind. Why else then would they 6190clone from the star basketball player, mathematician, and beauty queenor even dear old dadin the first place? Since the birth of Dolly, there has been a fair amount of doublespeak on this matter of genetic identity. Experts have rushed in to reassure the public that the clone would in no 6195way be the same person, or have any confusions about his or her identity; they are pleased to point out that the clone of film star Julia Roberts would not be Julia Roberts. Fair enough. But one is shortchanging the truth by emphasizing the additional importance of the environment, rearing, and 6200social setting: genotype obviously matters plenty. That, after all, is the only reason to clone, whether human beings or sheep. The odds that clones of basketball star Larry Bird will play basketball are, I submit, infinitely greater than they are for clones of jockey Willie Shoemaker. Passage 2 Given all the brouhaha, youd think it was crystal clear why cloning human beings is unethical. But what exactly is wrong with it? What would a clone be? Well, he or she would be a complete human being who happens to share the same genes with another person. Today, we call such 6210people identical twins. To my knowledge no one has argued that twins are immoral. You should treat all clones like you would treat all monozygous [identical] twins or triplets, concludes Dr. H. Tristam Engelhardt, a professor
6205

of medicine at Baylor and a philosopher at Rice University. 6215Thats it. It would be unethical to treat a human clone as anything other than a human being. Some argue that the existence of clones would undermine the uniqueness of each human being. Can individuality, identity, and dignity be severed from genetic distinctive6220ness, and from belief in a persons open future? asks political thinker George Will. Will and others have fallen under the sway of what one might call genetic essentialism, the belief that genes almost completely determine who a person is. But a person who is a clone 6225would live in a very different world from that of his or her genetic predecessor. With greatly divergent experiences, their brains would be wired differently. After all, even twins who grow up together are separate peopledistinct individuals with different personalities and certainly no 6230lack of Wills individuality, identity, and dignity. But what about cloning exceptional human beings? George Will put it this way: Suppose a clone of basketball star Michael Jordan, age 8, preferred violin to basketball? Is it imaginable? If so, would it be tolerable 6235to the cloner? Yes, it is imaginable, and the cloner would just have to put up with violin recitals. Kids are not commercial property. Overzealous parents regularly push their children into sports, music, and dance lessons, but given the stubborn nature of individuals, those parents rarely manage 6240to make kids stick forever to something they hate. A ban on cloning wouldnt abolish pushy parents. 15. The authors of both passages agree that (A) genetic characteristics alone cannot determine a persons behavior (B) a formal code of ethical rules will be needed once human beings can be cloned (C) people who are cloned from others may have greater professional opportunities (D) identical twins and triplets could provide useful advice to people related through cloning (E) cloning human beings is a greater technological challenge than cloning sheep 16. In line 13, the author of Passage 1 uses the word True to indicate (A) acknowledgement that the passages opening arguments are tenuous (B) recognition of a potential counterargument (C) conviction about the accuracy of the facts presented (D) distrust of those who insist on pursuing cloning research (E) certainty that cloning will one day become commonplace

6245

17. The question in lines 18-20 (Why else . . . first place) chiefly serves to (A) suggest that some issues are not easily resolved (B) argue for the importance of parents in the lives of children (C) offer an anecdote revealing the flaw in a popular misconception (D) imply that cloning might displace more familiar means of reproduction (E) suggest the value perceived in a person who might be selected for cloning 18. In line 21, fair most nearly means (A) (B) (C) (D) (E) considerable pleasing ethical just promising

6305

21. In line 55, divergent experiences emphasizes that which of the following is particularly important for a developing child? (A) (B) (C) (D) (E) Character Heritage Intelligence Environment Personality

6250

6310

22. In the quotation in lines 61-64, George Will primarily draws attention to
6315

6255

6260

6320

(A) a weakness inherent in cloning theory (B) a goal that some advocates of cloning might share (C) the limitations of human individuality (D) the likelihood that children will rebel against their parents (E) the extent to which a cloned person might differ from the original person 23. Both passages base their arguments on the unstated assumption that (A) genetic distinctiveness is crucial to human survival as a species (B) public concern about human cloning will eventually diminish (C) human cloning is a genuine possibility in the future (D) individualism is less prized today than it has been in the past (E) technological advances have had a mostly positive impact on society

19. The author of Passage 1 mentions two sports stars (lines 31-33) in order to
6265

6270

(A) argue against genetic analysis of any sports stars physical abilities (B) distinguish between lasting fame and mere celebrity (C) clarify the crucial role of rigorous, sustained training (D) highlight the need for greater understanding of the athletes genetic data (E) suggest that athletes special skills have a genetic component 20. In line 49, open most nearly means (A) (B) (C) (D) (E) overt frank unrestricted unprotected public

6325

6330

6275

6335

6280

6285

6290

6295

6300

175 6340 SECTION 6 CRITICAL READING


25 Questions
Directions: For each question in this section, select the best answer from among the choices given and fill in the corresponding circle on the answer sheet.
6345 6375

Time - 25 minutes

Each sentence below has one or two blanks, each blank indicating that something has been omitted. Beneath the sentence are five words or sets of words labeled A through E. Choose the word or set of words that, when inserted in the sentence, best fits the meaning of the sentence as a whole. Example: Hoping to ------- the dispute, negotiators proposed a compromise that they felt would be ------- to both labor and management. (A) (B) (C) (D) (E) enforce .. useful end .. divisive overcome .. unattractive extend .. satisfactory resolve .. acceptable

4. Because howler monkeys rarely come out of the trees in their arboreal habitat, the continued well-being of the rain forest is ------- to their survival.
6380

(A) inadequate (B) tangential (C) indispensable (D) baneful (E) expeditious 5. Doug was both ------- and -------: he possessed penetrating acuity and discernment and was also extremely humble. (A) (B) (C) (D) (E) diligent . . supercilious perspicacious . . unpretentious obtuse . . penitent sagacious . . imposing apologetic . . unassuming

6385

6390

6. The Mona Lisa, shipped in a private cabin and received by important dignitaries, was treated more like ------6395 than a painting upon its arrival in the United States. (A) a perfectionist (B) a maverick (C) a potentate (D) an ascetic (E) an interloper
6400

63501. Common garlic has ------- properties; during the First World War British medics saved thousands of lives by wrapping wounds with garlic-soaked bandages. 6355

(A) curative (B) flavoring (C) inferior (D) questionable (E) infamous 2. In her poems, Alice Walker retrieves and ------- parts of Black culture that some have been all too quick to ------- the past as fossilized artifacts.

7. Despite its patent -------, this belief has become so ------- that no amount of rational argument will suffice to eradicate it. (A) (B) (C) (D) (E) validity . . inconsequential implausibility . . entrenched credibility . . prevalent absurdity . . outmoded novelty . . infrequent

6405

6360

(A) (B) (C) (D) (E)

revitalizes . . consign to conjoins . . exclude from realigns . . salvage from diffuses . . defer to refracts . . impose on

8. The charlatans seemingly frank and open demeanor was actually a ------- means of enlisting his patients 6410 confidence. (A) disingenuous (B) debilitating (C) diminutive (D) cathartic (E) prosaic
6415

6365

3. The modest acceptance speech of the Academy Awardwinning actress revealed a ------- that contrasted with her uninhibited screen performances. (A) theatricality (B) sullenness (C) flamboyance (D) reserve (E) nonchalance

6370

6420
The passages below are followed by questions based on their content; questions following a pair of related passages may also be based on the relationship between the paired passages. Answer the questions on the basis of what is stated or implied in the passages and in any introductory material that may be provided.

Questions 9-13 are based on the following passages. Passage 1 It is striking how our culture has wholeheartedly adopted the recycling ethic. Most parents have probably 6425received humbling lectures from their children after tossing a glass jar or newspaper in the trash can. But the popularity of recycling is even more surprising considering the inconveniences associated with it. Who hasnt experienced the annoyance of trying to satisfy complicated rules about what 6430can and cannot be recycled? Glass jarsbut not their tops? Plastics number 1 and 2but not number 3? Still there is no sign that the public is becoming impatient, so convinced are people of the virtues of recycling. Passage 2 Mandatory recycling programs arent good for posterity. 6435They offer mainly short-term benefits to a few groups like politicians and waste-handling corporationswhile diverting money from genuine social and environmental problems. Recycling programs actually consume resources. They require extra administrators and a continual public 6440relations campaign explaining what to do with dozens of different productsrecycle milk jugs but not milk cartons, index cards but not construction paper. Collecting a ton of recyclable items is three times more expensive than collecting a ton of garbage because crews pick up less material 6445at each stop. Recycling may be the most wasteful activity in the modern United States: a waste of time and money, a waste of human and natural resources. 9. Which statement best characterizes the relationship between Passage 1 and Passage 2 ? (A) Passage 1 presents ethical objections to an action that Passage 2 also censures. (B) Passage 1 mocks a group of people that Passage 2 praises. (C) Passage 1 describes a cultural phenomenon that Passage 2 criticizes. (D) Passage 1 discusses the historical foundations of recycling, whereas Passage 2 considers the future of recycling. (E) Passage 1 describes peoples fascination with recycling, whereas Passage 2 explains the process of sorting recyclables.

10. Unlike Passage 1, Passage 2 focuses primarily on recyclings (A) (B) (C) (D) (E) philosophical foundations economic impact popular appeal moral implications environmental benefits

11. The author of Passage 2 would most likely characterize the people mentioned in line 11 as (A) (B) (C) (D) (E) emotional indecisive unmotivated undemanding uninformed

12. The authors of both passages would most likely agree that recycling rules are (A) (B) (C) (D) (E) convoluted commendable unethical antiquated unenforceable

13. Compared to the tone of Passage 2, the tone of Passage 1 is more (A) (B) (C) (D) (E) pessimistic arrogant critical scholarly tempered

Questions 14-25 are based on the following passage. This passage is taken from a novel set in early twentiethcentury England. Mrs. Deverell is the widow of a shopkeeper who lived and worked in Volunteer Street; their daughter Angel has become a best-selling novelist. Here, Mrs. Deverell finds herself in a new home that she and Angel share in the prosperous village of Alderhurst. I never thought I would live in such a beautiful place, Mrs. Deverell told Angel when they first moved in. But 6450nowadays she often suffered from the lowering pain of believing herself happy when she was not. Who could be miserable in such a place? she asked. Yet, on misty October evenings or on Sundays, when the church bells began, sensations she had never known before came 6455over her. She sometimes felt better when she went back to see her friends on Volunteer Street; but it was a long way to go. Angel discouraged the visits, and her friends seemed to have changed. Either they put out their best china and 6460thought twice before they said anything, or they were defiantly informalYoull have to take us as you find usand would persist in making remarks like Pardon the apron, but theres no servants here to polish the grate. In each case, they were watching her for signs of grandeur 6465or condescension. She fell into little traps they laid and then they were able to report to the neighbors. It hasnt taken her long to start putting on airs. She had to be especially careful to recognize everyone she met, and walked up the street with an expression of anxiety which 6470was misinterpreted as disdain. The name Deverell Family Grocer stayed for a long time over the shop, and she was pleased that it should, although Angel frowned with annoyance when she heard of it. Then one day the faded name was scraped and burnt 6475away, and on her next visit to Volunteer Street, she saw that Cubbages Stores was painted there instead. She felt an unaccountable panic and dismay at the sight of this and at the strange idea of other people and furniture in those familiar rooms. Very nice folk, she was told. Shes 6480so friendly. Always the same. And such lovely kiddies. Mrs. Deverell felt slighted and wounded; going home she was so preoccupied that she passed the wife of the landlord of The Volunteer without seeing her. I wouldnt expect Alderhurst people to speak to a barkeeps wife, 6485the woman told everyone in the saloon bar. Even though it was our Gran who laid her husband out when he died. All of their kindnesses were remembered and brooded over; any past kindness Mrs. Deverell had doneand they were manyonly served to underline the change 6490which had come over her. At a time of her life when she needed the security of familiar things, these were put beyond her reach. It seemed to her that she had wasted her years acquiring skills which in the end were to be of no use to her: her weather-eye for

6495a good drying day; her careful ear for judging the gentle singing sound of meat roasting in the oven; her touch for the freshness of meat; and how, by smelling a cake, she could tell if it were baked. These arts, which had taken so long to perfect, fell now into disuse. She would never 6500again, she grieved, gather up a great fragrant line of washing in her arms to carry indoors. One day when they had first come to the new house, she had passed through the courtyard where sheets were hanging out: she had taken them in her hands and, finding them just at the right 6505stage of drying, had begun to unpeg them. They were looped all about her shoulders when Angel caught her. Please leave work to the people who should do it, she had said. You will only give offense. She tried hard not to give offense; but it was difficult. The smell of 6510ironing being done or the sound of eggs being whisked set up a restlessness which she could scarcely control. The relationship of mother and daughter seemed to have been reversed, and Angel, now in her early twenties, was the authoritative one; since girlhood she had been 6515taking on one responsibility after another, until she had left her mother with nothing to perplex her but how to while away the hours when the servants were busy and her daughter was at work. Fretfully, she would wander around the house, bored, but afraid to interrupt; she was 6520like an intimidated child.

14. Which interpretation of Mrs. Deverells statement in line 1 (I never . . . place) is most fully supported by the rest of the passage? (A) It reveals an unsatisfied longing for beauty and comfort. (B) It suggests that Mrs. Deverell is unprepared for her new life. (C) It illustrates Mrs. Deverells desire to impress her old friends. (D) It hints at Mrs. Deverells increasing discomfort with her daughters career. (E) It indicates Mrs. Deverells inability to be happy in any environment. 15. The sensations (line 7) might best be described as feelings of (A) (B) (C) (D) (E) anger and bitterness reverence and gratitude dejection and isolation nostalgia and serenity empathy and concern

180

16. The primary purpose of the second paragraph (lines 9-23) is to show Mrs. Deverells
6525

20. Lines 40-43 (All of . . . her) suggest which of the following about the customers in the saloon bar?
6585

(A) (B) (C) (D) (E)

surprise that her friends have not forgotten her nostalgia for her old neighborhood feelings of superiority toward her friends embarrassment about her former neighborhood changing relationship with her friends

6530

6590

17. The author most likely quotes Mrs. Deverells friends in lines 14-16 in order to (A) (B) (C) (D) (E) voice a concern dismiss a belief illustrate an attitude cite an authority mock an undertaking

(A) They do not recall those occasions when Mrs. Deverell was kind to them. (B) They feel that Mrs. Deverell is still essentially the same person that she has always been. (C) They are not especially well acquainted with Mrs. Deverell. (D) They are more generous toward themselves than they are toward Mrs. Deverell. (E) They do not generally share the opinions of the barkeepers wife. 21. Lines 45-52 (It . . . disuse) suggest which of the following about the way that Mrs. Deverell had viewed the task of running a household? (A) She had believed some elements of it were beneath her. (B) She had understood the importance of its sensory aspects. (C) She had developed a regimented system. (D) She had been afraid to ask Angel for her help. (E) She had relied on household help to perform certain chores. 22. The use of arts in line 51 most directly emphasizes the

6535

6595

6540

18. The speaker of the sentence quoted in lines 15-16 (Pardon . . . grate) most likely intends to (A) account for a peculiar style of dress (B) bemoan the lack of adequate help around the house (C) frankly apologize for the messiness of the familys home (D) indirectly express resentment about a difference in social status (E) overtly call attention to Mrs. Deverells arrogant behaviour 19. Mrs. Deverells reaction to the remarks quoted in lines 32-33 suggests that she thinks that these remarks (A) (B) (C) (D) (E) contain an implicit criticism mischaracterize the new family are a poor attempt at humor stem from an old grudge insult the memory of her husband

6600

6545

6605

6550

6610

6555

6615

(A) pride Mrs. Deverells family took in her housekeeping skills (B) expertise Mrs. Deverell brought to her household tasks (C) importance of maintaining an orderly home (D) rewards of preparing elaborate meals (E) pleasure Mrs. Deverell found in teaching young servants

6560

6620

6565

6570

6575

6580

23. Angels comments in lines 60-61 (Please . . . offense) imply that


6625

6685

25. In line 73, the author compares Mrs. Deverell to an intimidated child primarily in order to (A) criticize Mrs. Deverell for her naive view of the world (B) show that Mrs. Deverell continues to be diminished in her new home (C) imply that Mrs. Deverell cannot live up to her responsibilities (D) indicate the simplicity of Mrs. Deverells new life (E) justify Angels dismissal of her mothers feelings

6630

(A) Mrs. Deverell has inadequate housekeeping experience (B) many people enjoy the opportunity to perform household tasks (C) Mrs. Deverell often hurts the feelings of others (D) domestic tasks are unsuitable for Mrs. Deverells new social status (E) Mrs. Deverell is not a particularly efficient worker

6690

6695 663524. In line 69, perplex most nearly means

6640

(A) (B) (C) (D) (E)

trouble bewilder astonish entangle embarrass

6645

6650

6655

6660

6665

6670

6675

6680

185 6700 SECTION 9 CRITICAL READING


19 Questions
Directions: For each question in this section, select the best answer from among the choices given and fill in the corresponding circle on the answer sheet.
6705 6735

Time 20 minutes

Each sentence below has one or two blanks, each blank indicating that something has been omitted. Beneath the sentence are five words or sets of words labeled A through E. Choose the word or set of words that, when inserted in the sentence, best fits the meaning of the sentence as a whole. Example: Hoping to ------- the dispute, negotiators proposed a compromise that they felt would be ------- to both labor and management. (A) (B) (C) (D) (E) enforce .. useful end .. divisive overcome .. unattractive extend .. satisfactory resolve .. acceptable

3. Hawaii refers both to the group of islands known as the Hawaiian islands and to the largest island in 6740 that -------. (A) flora (B) sierra (C) archipelago (D) flotilla (E) savanna
6745

4. Given the exponential growth of scientific knowledge, medicine is far less ------- unsubstantiated fads than it used to be; its record of folly, however, remains an undeniable -------. (A) (B) (C) (D) (E) suspicious of . . qualification averse to . . encumbrance vulnerable to . . embarrassment dependent on . . impossibility ignorant of . . oversight

6750

6755 67101. Unable to discover how the fire started, the inspectors filed a tentative report stating that the cause was -------.

5. The aspiring writer, who remained ------- even after being rejected by several major publishers, felt certain of achieving literary -------. (A) hopeless . . vindication (B) disgruntled . . talent (C) optimistic . . abasement (D) undaunted . . celebrity (E) obsequious . . neglect

(A) noteworthy (D) conclusive

(B) definitive (C) fundamental (E) indeterminate

6760

67152. The celebrants at the ------- party for Cinco De Mayo were understandably ------- by the spectacle of the mariachi bands and the colorful piatas for the children. 6720

(A) (B) (C) (D) (E)

somber . . amused lavish . . dazzled novel . . jaded mundane . . astounded joyous . . stymied

6. Fred often used ------- to achieve his professional goals, even though such artful subterfuge alienated 6765 his colleagues. (A) chicanery (B) diligence (C) bombast (D) disputation (E) consensus

6770 6725

6730

The passage below is followed by questions based on its content. Answer the questions on the basis of what is stated or implied in the passage and in any introductory material that may be provided.

Questions 7-19 are based on the following passage. In the following passage from a newspaper commentary written in 1968, an architecture critic discusses old theaters and concert halls. After 50 years of life and 20 years of death, the great Adler and Sullivan Auditorium in Chicago is back in business again. Orchestra Hall, also in Chicago, was beautifully spruced up for its sixty-eighth birthday. In St. Louis, a 1925 movie palace has been successfully 6780transformed into Powell Symphony Hall, complete with handsome bar from New Yorks demolished Metropolitan Opera House. Sentimentalism? Hardly. This is no more than a practical coming of cultural age, a belated recognition 6785that fine old buildings frequently offer the most for the money in an assortment of values, including cost, and above all, that new cultural centers do not a culture make. It indicates the dawning of certain sensibilities, perspectives, and standards without which arts programs 6790are mockeries of everything the arts stand for. The last decade has seen city after city rush pell-mell into the promotion of great gobs of cultural real estate. It has seen a few good new theaters and a lot of bad ones, temples to bourgeois muses with all the panache of sub6795urban shopping centers. The practice has been to treat the arts in chamber-of-commerce, rather than in creative, terms. That is just as tragic as it sounds. The trend toward preservation is significant not only because it is saving and restoring some superior buildings 6800that are testimonials to the creative achievements of other times, but also because it is bucking the conventional wisdom of the conventional power structure that provides the backing for conventional cultural centers to house the arts. 6805 That wisdom, as it comes true-blue from the hearts and minds of real estate dealers and investment bankers, is that you dont keep old buildings; they are obsolete. Anything new is better than anything old and anything big is better than anything small, and if a few cultural values are lost 6810along the way, it is not too large a price to pay. In addition, the new, big buildings must be all in one place so they will show. Theyll not only serve the arts, theyll improve the surrounding property values. Build now, and fill them later. At the same time, tear down the past, rip out cultural 6815roots, erase tradition, rub out the architectural evidence that the arts flowered earlier in our cities and enriched them and that this enrichment is culture. Substitute a safe and sanitary status symbol for the loss. Put up the shiny mediocrities of the present and demolish the shabby masterpieces of the
6775

6820past. That is the ironic other side of the cultural explosion coin. In drama, and in life, irony and tragedy go hand in hand. Chicagos Auditorium is such a masterpiece. With its glowing, golden ambiance, its soaring arches and super6825stage from which whispers can be heard in the far reaches of the theater, it became a legend in its own time. One of the great nineteenth-century works of Louis Sullivan and Dankmar Adler and an anchor point of modern architectural history, it has been an acknowledged model of acoustical 6830and aesthetic excellence. (Interestingly, the Auditorium is a hard theater in which to install microphones today, and many modern performers, untrained in balance and projection and reliant on technical mixing of sound, find it hard to function in a near-perfect house.) 6835 Until October 1967, the last performance at the Auditorium was of Hellzapoppin in 1941, and the last use of the great stage was for bowling alleys during the Second World War. Closed after that, it settled into decay for the next 20 years. Falling plaster filled the hall, and the golden ceil6840ing was partly ruined by broken roof drains. Last fall the Auditorium reopened, not quite in its old glory, but close to it. The splendors of the house were traced in the eightcandlepower glory of carbon-filament lightbulbs of the same kind used in 1889 when the theater, and electricity, 6845were new. Their gentle brilliance picked out restored architectural features in warm gilt and umber. We have never had greater technical means or expertise to make our landmarks bloom. The question is no longer whether we can bring old theaters back to new brilliance, 6850but whether we can fill them when theyre done. As with the new centers, that will be the acid cultural test.

7. The principal function of the opening paragraph is to (A) introduce the concept of conventional arts centers (B) illustrate the trend toward revitalization of cultural landmarks (C) explore the connection between classical architecture and the arts (D) provide an explanation for the theaters resurgent popularity (E) contrast the beauty of old theaters with ordinary modern buildings

6855

8. On the basis of information provided in the rest of the passage, the word death (line 1) best conveys (A) (B) (C) (D) (E) flagging attendance wartime malaise demolition neglect disrepute

6915

12. The description in lines 20-21 (temples . . . centers) best serves to (A) scorn the architects commitment to historically accurate renovations (B) mock the timeworn theatrical works showcased in modern cultural centers (C) deprecate the appearance and character of many new theaters (D) downplay the governments efforts to support the arts (E) poke good-humored fun at commercial establishments 13. As described in lines 17-23, the practice refers to the (A) (B) (C) (D) (E) commercialization of culture preservation of cultural treasures construction of shopping centers government funding of the arts distortion of theatrical works

6860

6920

9. The bar mentioned in line 7 had apparently been (A) (B) (C) (D) (E) costly but symbolic beautiful but outdated enlarged and elongated treasured and imitated rescued and relocated
6925

6865

10. The question in line 9 is intended to


6870

(A) (B) (C) (D) (E)

6875

expose the folly of the new construction convey the emotional burdens of the past provide a typical explanation for the renovations lament the decline of cultural values address the publics indifference toward old buildings

6930

6935

14. In lines 27-30, the author uses the word conventional several times in order to (A) reveal the performers frustration with modern theaters (B) disparage the present-day treatment of the arts (C) parody the creative efforts of contemporary artists (D) emphasize the absurdity of a purely aesthetic approach to the arts (E) exaggerate the importance of tradition in the arts 15. The fifth paragraph (lines 31-39) primarily serves to (A) criticize the way in which cultural buildings are viewed as commodities (B) assess the positive impact of the architects backlash against mediocrity (C) contrast the business practices of real estate brokers with those of bankers (D) enumerate the costs and benefits of restoring historic landmarks (E) question the importance of the arts to society

11. In lines 13-14, the phrase new . . . make most directly suggests that
6880

6885

(A) modern architects lack the artistic reputations of their predecessors (B) the commercial treatment of culture encourages art that is mass-produced (C) culture evolves out of tradition and cannot be instantly created (D) historically significant venues positively influence the creative process (E) new cultural centers should be constructed in collaboration with artists

6940

6945

6890

6950

6895

6955

6900

6905

6910

190

16. What does the imagery in lines 40-43 suggest?


6960

(A) The dawning of an enlightened artistic sensibility has stimulated support for preserving historic theaters. (B) The ill-conceived mandate to destroy architectural masterpieces epitomizes the censorship of creative expression. (C) The desire for societal status symbols drives the construction of grandiose cultural centers. (D) The demolition of a historic landmark is tantamount to the destruction of an invaluable cultural legacy. (E) The restoration of intimate old theaters will speed the demise of large new arenas.

702018. In lines 56-60, the authors comment about microphones implies that

7025

6965

7030

6970

(A) the near-perfect acoustics in a new theater divert attention from the buildings aesthetic flaws (B) audience members seated in the theaters balcony cannot fully appreciate the nuances of the performers intonations (C) the performances of modern-day actors tend to be overly dependent on technology (D) the absence of technically sophisticated equipment has jeopardized the sound quality of performances (E) old theaters can remain viable because they readily accommodate the new sound technology that enhances a performance 19. Which challenge is emphasized by the author in the final paragraph (lines 73-77) ? (A) (B) (C) (D) (E) Designating theaters as historical landmarks Renewing a respect for architecture Providing opportunities for new artists Reviving classical plays Attracting appreciative audiences

17. In lines 49-56, the description of the building primarily 6975 serves to (A) convey an appreciation for the technical complexities of renovating theaters (B) illustrate how nineteenth-century architecture directly influenced modern building design (C) highlight some unique aspects of an example of fine architecture (D) explain why some people disdain innovative architecture (E) show how restoration can strip a building of its unique character

7035

7040

6980

7045

6985

7050 6990

6995

7000

7005

7010

7015

SECTION 10

WRITING

Time 10 minutes 14 Questions


7055 Directions: For each question in this section, select the best answer from among the choices given and fill in the corresponding circle on the answer sheet.

The following sentences test correctness and effectiveness of expression. Part of each sentence or the entire sentence is underlined; beneath each sentence are five ways of phrasing the underlined material. Choice A repeats the original phrasing; the other four choices are different. If you think the original phrasing produces a better sentence than any of the alternatives, select choice A; if not, select one of the other choices. In making your selection, follow the requirements of standard written English; that is, pay attention to grammar, choice of words, sentence construction, and punctuation. Your selection should result in the most effective sentence clear and precise, without awkwardness or ambiguity. EXAMPLE: Laura Ingalls Wilder published her first book and she was sixty-five years old then. (A) and she was sixty-five years old then (B) when she was sixty-five (C) at age sixty-five years old (D) upon the reaching of sixty-five years (E) at the time when she was sixty-five

70802. When the weather forecaster predicts a severe storm, this is when people usually rush to the supermarket to stock up on groceries.

7085

(A) (B) (C) (D) (E)

storm, this is when people usually rush storm is usually when people are rushing storm is why people usually rush storm, people usually rush storm, it usually rushes people

7090

3. When, after bleak and lonely years in an English public school, he returned to India, there was suddenly perceived by himself a strong desire to write about the people and land he loved. (A) (B) (C) (D) (E) there was suddenly perceived by himself he suddenly was perceived suddenly the feeling that came to him being he suddenly felt suddenly he had the feeling of

7095

7100

4. Curiosity about other people, about the ways they think and act, has caused Jeff to meet some fascinating characters as well as people which also really bore him. (A) (B) (C) (D) (E) people which also really bore him he encountered really boring people very boring people are also met some very boring people very boring people also

7105

7060
1. People were unprepared for the sinking of the Titanic simply because of believing that the ship was unsinkable.
7065 7110

(A) (B) (C) (D)

7070

of believing that the ship was unsinkable of having a belief in the ship as unsinkable they believed that the ship was unsinkable they believed the unsinkable nature of the ship (E) of a belief on their part of an unsinkable ship

5. During seasons when ticks carrying Lyme disease are most prevalent, signs could be posted to deter hikers about their venturing into tick-infested areas. (A) (B) (C) (D) (E) about their venturing from their venturing from venturing by not venturing not to venture

7115

7075

195
7120

6. After Morris had spent ten minutes giving an answer, Claudette found he had given her only one item of information beyond what she already knew.
7125

9. Astronomy is the study of celestial bodies in outer space, especially their positions, dimensions, 7185 movements, and composition. (A) especially their positions, dimensions, movements, and composition (B) and especially they are concerned with their positions, dimensions, movements, and composition (C) especially studying their positions, dimensions, movements, and composition (D) especially their positions, dimensions, movements, and with their composition (E) with special study of their positions, dimensions, movements, and including composition

(A) (B) (C) (D) (E)

beyond what she already knew beyond what she knows already beyond her knowledge at the current time to add to what she knew already presently in addition to her present knowledge then

7190

7130

7. Although the kings and queens of England are considered Canadas monarchs, true political power lies with the prime minister, that person is elected by the Canadian citizenry. (A) true political power lies with the prime minister, that person is elected (B) the person who holds true political power is the prime minister, which is elected (C) true political power lies with the prime minister, who is elected (D) the prime minister, the source of true political power, elected (E) true political power is with the prime minister and is elected 8. Led by vocalist Marlena McGhee Smalls, Gullah tradition is preserved by the help of the Hallelujah Singers of South Carolina through songs and stories. (A) Gullah tradition is preserved by the help of the Hallelujah Singers of South Carolina through songs and stories (B) the Hallelujah Singers of South Carolina help to preserve Gullah tradition through songs and stories (C) the songs and stories of Gullah tradition are preserved through the Hallelujah Singers of South Carolina (D) it is the Hallelujah Singers that help to preserve the songs and stories of Gullah tradition in South Carolina (E) South Carolinas Gullah tradition is preserved through songs and stories by the Hallelujah Singers

7195

7135

7140

10. All the talk about controlling noise, keeping rivers clean, and planting trees have not impressed people 7200 enough to be bringing about major changes in laws and lifestyles. (A) have not impressed people enough to be bringing (B) have not made enough of an impression on people to bring (C) have not made people impressed enough to bring (D) has not impressed people enough to bring (E) has not made enough people impressed for bringing

7205

7145

7150

7210 The furnace exploded, blowing off the door, spraying 11. greasy soot all over the basement floor, and it would rattle furniture and windowpanes throughout the building. 7215

7155

(A) (B) (C) (D) (E)

it would rattle it rattled causing the rattling of the result was to rattle rattling

7160

7165

7170

7175

7180

7220

12. The adaptation of a novel for the screen often requires major adjustments in plot because the one art form differs from the other in having other character7225 revelation techniques. (A) because the one art form differs from the other in having other character-revelation techniques (B) because the two art forms reveal character in different ways (C) because of the differing ways the two may use for revealing a character (D) inasmuch as there are different ways in the two art forms for character revelation (E) insofar as the two differ in how to reveal character 13. The opposing opinions expressed were that the school should be torn down and, on the other hand, to keep it as a historical landmark. (A) were that the school should be torn down and, on the other hand, to keep it (B) was that the school should be torn down or kept (C) were that the school should be torn down and that it should be kept (D) were about them tearing the school down and them keeping the school (E) were if they should tear the school down and keeping it

14. Feeling, perhaps, that their votes do not matter, the number of young people going to the polls 7285 are becoming increasingly smaller. (A) the number of young people going to the polls are becoming increasingly smaller (B) the number of young people going to the polls is increasingly smaller (C) increasingly smaller numbers of young people are going to the polls (D) young people are going to the polls in increasingly smaller numbers (E) young people, who in increasingly smaller numbers are going to the polls

7290

7230

7295

7235

7240

7245

7250

7255

7260

7265

7270

7275

7280

Practice Test 5 7300 SECTION 1 WRITING ESSAY

Time 25 minutes

The essay gives you an opportunity to show how effectively you can develop and express ideas. You should, therefore, take 7305care to develop your point of view, present your ideas logically and clearly, and use language precisely. Your essay must be written on the lines provided on your answer sheet you will receive no other paper on which to write. You will have enough space if you write on every line, avoid wide margins, and keep your handwriting to a reasonable size. 7310Remember that people who are not familiar with your handwriting will read what you write. Try to write or print so that what you are writing is legible to those readers. Important Reminders:

7315

A pencil is required for the essay. An essay written in ink will receive a score of zero. Do not write your essay in your test book. You will receive crdit only for what you write on your answer sheet. An off-topic essay will receive a score of zero. If your essay does not reflect your original and individual work, your test scores may be cancelled.

7320

You have twenty-five minutes to write an essay on the topic assigned below.

Think carefully about the issue presented in the following excerpt and the assignment below.
7325

A colleague of the great scientist James Watson remarked that Watson was always lounging around, arguing about problems instead of doing experiments. He concluded that There is more than one way of doing good science. It was Watsons form of idleness, the scientist went on to say, that allowed him to solve the greatest of all biological problems: the discovery of the structure of DNA. It is a point worth remembering in a society overly concerned with efficiency. Adapted from John C. Polanyi, Understanding Discovery

Assignment:
7330

Do people accomplish more when they are allowed to do things in their own way? Plan and write an essay in which you develop your point of view on this issue. Support your position with reasoning and examples taken from your reading, studies, experience, or observations.

200

Practice Test 5 7335 SECTION 4 CRITICAL READING


24 Questions
Directions: For each question in this section, select the best answer from among the choices given and fill in the corresponding circle on the answer sheet.

Time 25 minutes

7340
Each sentence below has one or two blanks, each blank indicating that something has been omitted. Beneath the sentence are five words or sets of words labeled A through E. Choose the word or set of words that, when inserted in the sentence, best fits the meaning of the sentence as a whole. Example: Hoping to ------- the dispute, negotiators proposed a compromise that they felt would be ------- to both labor and management. (A) (B) (C) (D) (E) enforce .. useful end .. divisive overcome .. unattractive extend .. satisfactory resolve .. acceptable
7385

4. Latoyas ------- is shown by her ability to be -------: she can see her own faults more clearly than anyone 7375 else can. (A) perceptiveness . . self-centered (B) objectivity . . restrictive (C) cynicism . . self-destructive (D) open-mindedness . . complacent (E) insightfulness . . self-critical 5. The bearded dragon lizard is a voracious eater, so ------that it will consume as many insects as possible. (A) abstemious (B) cannibalistic (D) insatiable (E) unpalatable (C) slovenly

7380

6. Because drummer Tony Williams paved the way for later jazz-fusion musicians, he is considered a ------of that style.
7390

1. Some fans feel that sports events are ------- only when 7345 the competitors are of equal ability, making the outcome of the game -------. (A) (B) (C) (D) (E) successful . . assured boring . . questionable dull . . foreseen interesting . . predictable exciting . . uncertain

(A) connoisseur (B) revivalist (D) disparager (E) progenitor

(C) beneficiary

7. The politicians speech to the crowd was composed of nothing but -------, a bitter railing against the partys 7395 opponents. (A) digressions (B) diatribes (C) platitudes (D) machinations (E) acclamations
7400

7350

2. Alfred Schnittkes musical compositions are -------: phrases are clipped, broken into sections, and split 7355 apart by long rests. (A) garnished (B) improvisational (C) fragmented (D) cautious (E) uniform 3. The consumer advocate claimed that while drug 7360 manufacturers ------- the supposed advantages of their proprietary brands, generic versions of the same medications are often equally -------. (A) tout . . efficacious (B) research . . innocuous (C) market . . prohibitive (D) laud . . counterproductive (E) extract . . prescriptive

8. Favoring economy of expression in writing, the professor urged students toward a ------- rather than an ------- prose style. (A) (B) (C) (D) (E) spare . . ornate terse . . opinionated personal . . academic baroque . . embellished repetitive . . intricate

7405

7365

7370

7410
The passages below are followed by questions based on their content; questions following a pair of related passages may also be based on the relationship between the paired passages. Answer the questions on the basis of what is stated or implied in the passages and in any introductory material that may be provided.

Questions 9-12 are based on the following passages. Passage 1 Food has always been considered one of the most salient markers of cultural traditions. When I was a small child, food was the only thing that helped identify my family as 7415Filipino American. We ate pansit lug-lug (a noodle dish) and my father put patis (salty fish sauce) on everything. However, even this connection lessened as I grew older. As my parents became more acculturated, we ate less typically Filipino food. When I was twelve, my mother 7420took cooking classes and learned to make French and Italian dishes. When I was in high school, we ate chicken marsala and shrimp fra diablo more often than Filipino dishes like pansit lug-lug. Passage 2 Jean Anthelme Brillat-Savarinwho in 1825 conf7425dently announced, Tell me what you eat, and I will tell you who you arewould have no trouble describing cultural identities of the United States. Our food reveals us as tolerant adventurers who do not feel constrained by tradition. We play with our food far more readily 7430than we preserve the culinary rules of our varied ancestors. Americans have no single national cuisine. What unites American eaters culturally is how we eat, not what we eat. As eaters, Americans mingle the culinary traditions of many regions and cultures. We are multiethnic eaters. 9. Which of the following statements best captures the relationship between the two passages? (A) Passage 1 notes problems for which Passage 2 proposes solutions. (B) Passage 1 presents claims that are debunked by Passage 2. (C) Passage 2 furnishes a larger context for the experiences described in Passage 1. (D) Passage 2 provides an update of the situation depicted in Passage 1. (E) Passage 2 uses material presented in Passage 1 to correct a popular misconception.

10. The author of Passage 2 would most likely regard the mothers willingness to make French and Italian dishes (lines 9-10, Passage 1) as (A) (B) (C) (D) (E) laughably pretentious understandably conservative typically American a regrettable compromise a surprising attitude

11. The two passages differ in their discussions of food primarily in that Passage 1 (A) considers specific dishes eaten by particular people, whereas Passage 2 comments on a cultures general attitude toward eating (B) contrasts the cuisines of different cultures, whereas Passage 2 emphasizes culinary practices common to all cultures (C) presents an abstract theory of food, whereas Passage 2 offers a historical analysis of consumption (D) emphasizes the role of nostalgia in food preferences, whereas Passage 2 rejects that approach as overly sentimental (E) outlines some popular choices in cuisine, whereas Passage 2 underscores those that are more unusual 12. Unlike the author of Passage 2, the author of Passage 1 makes significant use of (A) (B) (C) (D) (E) direct quotation sociological analysis hypothetical assumptions historical sources personal experience

205
Questions 13-24 are based on the following passages. The passages below discuss the possibility of locating intelligent life on other planets. Passage 1 has been adapted from a 1999 book on the history of the universe. Passage 2 was excerpted from a 2000 book on the scientific quest for extraterrestrial life. Passage 1 Generations of science-fiction movies have conditioned us to consider bug-eyed monsters, large-brained intellectual humanoids, and other rather sophisticated extraterrestrial creatures as typical examples of life outside Earth. The reality, however, is that finding any kind of life at all, even 7440something as simple as bacteria, would be one of the most exciting discoveries ever made. The consensus within the scientific community seems to be that we eventually will find not only life in other parts of the galaxy but also intelligent and technologically advanced 7445life. I have to say that I disagree. While I believe we will find other forms of life in other solar systems (if not in our own), I also feel it is extremely unlikely that a large number of advanced technological civilizations are out there, waiting to be discovered. The most succinct support 7450for my view comes from Nobel laureate physicist Enrico Fermi, the man who ran the first nuclear reaction ever controlled by human beings. Confronted at a 1950 luncheon with scientific arguments for the ubiquity of technologically advanced civilizations, he supposedly 7455said, So where is everybody? This so-called Fermi Paradox embodies a simple logic. Human beings have had modern science only a few hundred years, and already we have moved into space. It is not hard to imagine that in a few hundred more years we will 7460be a starfaring people, colonizing other systems. Fermis argument maintains that it is extremely unlikely that many other civilizations discovered science at exactly the same time we did. Had they acquired science even a thousand years earlier than we, they now could be so much more 7465advanced that they would already be colonizing our solar system. If, on the other hand, they are a thousand years behind us, we will likely arrive at their home planet before they even begin sending us radio signals. Technological 7470advances build upon each other, increasing technological abilities faster than most people anticipate. Imagine, for example, how astounded even a great seventeenth-century scientist like Isaac Newton would be by our current global communication system, were he alive today. Where are 7475those highly developed extraterrestrial civilizations so dear to the hearts of science-fiction writers? Their existence is far from a foregone conclusion.
7435

Passage 2 Although posed in the most casual of circumstances, the Fermi Paradox has reverberated through the decades 7480and has at times threatened to destroy the credibility of those scientists seriously engaged in the Search for Extraterrestrial Intelligence (SETI) research program. One possible answer to Fermis question (If there are extraterrestrials, where are they?) is that extraterrestrials 7485have in fact often visited Earth, and continue to do so. This is the answer of those who believe in the existence of unidentified flying objects, or UFOs. But few scientists, even those engaged in SETI, take the UFO claims seriously. You wont find anyone around here who 7490believes in UFOs, says Frank Drake, a well-known SETI scientist. If one discounts the UFO claims, yet still believes that there are many technological civilizations in the galaxy, why have they not visited us? Drakes answer is straightforward: High-speed interstellar travel is so 7495demanding of resources and so hazardous that intelligent civilizations dont attempt it. And why should they attempt it, when radio communication can supply all the information they might want? At first glance, Drakes argument seems very persua7500sive. The distances between stars are truly immense. To get from Earth to the nearest star and back, traveling at 99 percent of the speed of light, would take 8 years. And SETI researchers have shown that, to accelerate a spacecraft to such a speed, to bring it to a stop, and 7505to repeat the process in the reverse direction, would take almost unimaginable amounts of energy. Astronomer Ben Zuckerman challenges Drakes notion that technological beings would be satisfied with radio communication. Drakes implicit assumption is 7510that the only thing were going to care about is intelligent life. But what if we have an interest in simpler life-forms? If you turn the picture around and you have some advanced extraterrestrials looking at the Earth, until the last hundred years there was no evidence of intelligent 7515life but for billions of years before that they could have deduced that this was a very unusual world and that there were probably living creatures on it. They would have had billions of years to come investigate. Zuckerman contends that the reason extraterrestrials havent visited us is that so 7520few exist.

13. Which statement about the Fermi Paradox is supported by both passages?
7525

17. Passage 1 suggests that the Fermi Paradox depends most directly on which assumption?
7585

7530

7535

(A) It articulates a crucial question for those interested in the existence of extraterrestrials. (B) It clarifies the astronomical conditions required to sustain life on other planets. (C) It reveals the limitations of traditional ideas about the pace of technological change. (D) It demonstrates the scientific communitys fascination with the concept of interstellar travel. (E) It suggests that advanced extraterrestrial civilizations may be uninterested in our culture. 14. Which statement best describes a significant difference between the two passages?

7590

7595

(A) Extraterrestrial civilizations may not wish to be discovered by human beings. (B) Extraterrestrial civilizations would most likely have discovered technology at about the same time human beings discovered it. (C) Extraterrestrial technology would develop at roughly the same rate as human technology. (D) Extraterrestrial civilizations would inevitably use technology for aggressive ends. (E) Science is a more powerful form of human knowledge than are art and literature. 18. The claim made in Passage 1 that a consensus exists (lines 8-11) would most likely be interpreted by the author of Passage 2 as

7540

7545

7550

(A) Passage 1 analyzes a literary form, while Passage 2 argues that literature has little bearing on science. (B) Passage 1 presents an argument, while Passage 2 surveys current opinion in a debate. (C) Passage 1 concludes by rejecting the Fermi Paradox, while Passage 2 opens by embracing it. (D) Passage 1 describes a phenomenon, while Passage 2 details a belief system that would reject such a phenomenon. (E) Passage 1 defends a viewpoint, while Passage 2 questions that viewpoints place in scientific research.

7600

7605

(A) evidence of compromise in the scientific community (B) an attack on SETI researchers (C) support for Fermis analysis (D) a revelation of an unexpected truth (E) an oversimplification of a complex debate 19. The author of Passage 1 mentions Isaac Newton (lines 37-40) in order to

7610

755515. The author of Passage 1 mentions monsters, humanoids, and creatures (lines 2-4) primarily to

7615

(A) emphasize the rapid rate of technological innovation (B) acknowledge the impact of a profound thinker (C) criticize the inflexibility of Newtons contemporaries (D) speculate about Newtons influence on current research (E) highlight the value of scientific curiosity 20. In lines 44-48, the author of Passage 2 indicates that the Fermi Paradox has been

7560

(A) question the literary value of science fiction (B) contrast fictional notions with a scientific perspective (C) offer examples of the human fear of the unknown (D) criticize science fiction for being unduly alarmist (E) suggest that scientific research has been influenced by science fiction 16. In line 17, ran most nearly means (A) (B) (C) (D) (E) fled accumulated traversed managed incurred

7620

7565

(A) (B) (C) (D) (E)

thoroughly misunderstood surprisingly influential overwhelmingly perplexing intermittently popular frequently misquoted

7625

7570

7575

7580

763021. How would Frank Drake (line 56, Passage 2) most likely respond to the statement by the author of Passage 1 about humans colonizing other systems (line 26) ? 7635

7690

23. In line 63, radio communication is cited as a (A) (B) (C) (D) (E) complex interaction technological relic common occurrence practical alternative dramatic advance

7640

(A) The means to accomplish such a project may be beyond our reach. (B) Interstellar colonization is as morally problematic as was colonization on Earth. (C) We would do better to study indigenous life-forms rather than search for extraterrestrial creatures. (D) Humans would be wise to consider that they themselves are subject to colonization. (E) Funding for such an undertaking would pose a thorny political issue for any government.

7695

24. Both the author of Passage 1 and Ben Zuckerman (line 73, Passage 2) imply that researchers seeking life 7700 on another planet should focus on which of the following? (A) (B) (C) (D) (E) Seasonal variations in color due to plant life Evidence of the most basic forms of life Signs of artificially created structures Signals that might be radio communications Changes in geological surface features

764522. In line 57, claims most nearly means

7705

7650

(A) (B) (C) (D) (E)

demands assertions rights territories compensations

7710

7655

7660

7665

7670

7675

7680

7685

210

SECTION 5
7715

WRITING

Time - 25 minutes 35 Questions

Directions: For each question in this section, select the best answer from among the choices given and fill in the corresponding circle on the answer sheet.

7720
The following sentences test correctness and effectiveness of expression. Part of each sentence or the entire sentence is underlined; beneath each sentence are five ways of phrasing the underlined material. Choice A repeats the original phrasing; the other four choices are different. If you think the original phrasing produces a better sentence than any of the alternatives, select choice A; if not, select one of the other choices. In making your selection, follow the requirements of standard written English; that is, pay attention to grammar, choice of words, sentence construction, and punctuation. Your selection should result in the most effective sentence clear and precise, without awkwardness or ambiguity. EXAMPLE: Laura Ingalls Wilder published her first book and she was sixty-five years old then. (A) and she was sixty-five years old then (B) when she was sixty-five (C) at age sixty-five years old (D) upon the reaching of sixty-five years (E) at the time when she was sixty-five

7740

2. Winslow Homer, one of Americas foremost artists, spent his last 27 years and painted on the scenic Maine coast.
7745

(A) (B) (C) (D) (E)

spent his last 27 years and painted spent his last 27 years having painted spent his last 27 years painting having spent his last 27 years doing his painting spending his last 27 years painting

7750

3. Researchers are experimenting with various techniques for preventing the accumulation in water of high levels of nitrogen, which can kill plants and animals. (A) nitrogen, which can kill plants and animals (B) nitrogen; plants and animals can be killed (C) nitrogen, that is what can kill plants and animals (D) nitrogen, they could kill plants and animals (E) nitrogen, and they can kill plants and animals

7755

77604. When the news spread how new goldfields were discovered in Nome, Alaska, thousands abandoned Dawson, the site of the previous gold rush.

7765

(A) how new goldfields were discovered (B) how there was discovery of new goldfields (C) about new goldfields, which they discovered (D) about new goldfields, and they were discovered (E) about new goldfields that had been discovered 5. When the Berlin Wall, long a symbol of the Cold War, began to be torn down in 1989, five million people went to Berlin to celebrate that. (A) (B) (C) (D) (E) to celebrate that for its celebration to celebrate in celebration of that in celebrating

1. The library is older than it but still just as beautiful as the courthouse.
7725

7770

7730

(A) older than it but still just as beautiful as the courthouse (B) older and it is just as beautiful as the courthouse (C) older than the courthouse; it is just as beautiful as it (D) older than the courthouse but just as beautiful (E) just as beautiful as the courthouse and it is older than it

7775

7735

7780

6. To complete the music program, a student must present one vocal performance, one instrumental performance, and composing one original work.
7785

9. New Zealands Kaikoura Peninsula, a ruggedly beautiful spit of land, borders an undersea canyon that 7845 is home to the sperm whale and the giant squid. (A) (B) (C) (D) (E) borders an undersea canyon that is bordering an undersea canyon, and it borders an undersea canyon, which is which borders an undersea canyon, is the border of an undersea canyon, being

(A) (B) (C) (D) (E)

and composing one original work and one original composition with one original composition and to compose one original work as well as the students original composition

7850

7790

7. Zookeepers have expanded ones definition of care to include concern for the animals mental state as well as for its physical well-being. (A) (B) (C) (D) (E) have expanded ones definition of care to include have expanded ones definition of care, including expand their definition of care, they include expanding the definition of care to include have expanded their definition of care to include

10. In similarity with some other great works, the enduring horror tale Frankenstein was first published anonymously; its author, Mary Shelley, wrote the 7855 novel when she was not quite nineteen years old. (A) (B) (C) (D) (E) In similarity with As Like what happened with Like the case with Like

7795

7860 78008. The time and the place for such a large event is subject to approving from the mayors office.

7805

7810

(A) The time and the place for such a large event is subject to approving from the mayors office. (B) For such a large event, the time and the place are subject to the mayors offices approving them. (C) The time and the place for such a large event are subject to the approval of the mayors office. (D) The time and place for such a large event are subject to be approved by the office of the mayor. (E) Subject to the approval of the mayors office are the time and place for such a large event taking place.

11. The book is useful because it offers not just philosophy and theory but also tells you what and how to live every day.
7865

7870

(A) but also tells you what and how to live every day (B) but also it gives ways of everyday living (C) but also advice for everyday living (D) but also it gives practical advice for everyday life (E) and also tells you what to do and how to live every day

7815

7820

7825

7830

7835

7840

215
7875
The following sentences test your ability to recognize grammar and usage errors. Each sentence contains either a single error or no error at all. No sentence contains more than one error. The error, if there is one, is underlined and lettered. If the sentence contains an error, select the one underlined part that must be changed to make the sentence correct. If the sentence is correct, select choice E. In choosing answers, follow the requirements of standard written English. EXAMPLE: The other delegates and him immediately A B C accepted the resolution drafted by the D neutral states. No error E
7920 791014. The newly elected Prime Minister, to the dismay

A C

of opponents from other parties, have argued for the strict regulation of campaign financing. No error
7915

15. Studies have suggested that eating nutsalmonds A in particularmight help to lower blood cholesterol B levels in humans and reducing the risk of heart disease C by protecting the blood vessels. No error D
7925

12. The country found that its economy was growing A


7880

16. In English literature James Boswell is the prime A example of a biographer who, by ensuring the B
7930

more stronger, with an improved outlook and more B D C E opportunities for training and employment. No error

immortality of another author, has achieved C immortality for himself . No error D E

788513. The iris, the colored part of the eye, contains delicate

A patterns that are unique to each person, offering a B


7890

793517. Because the garden was untended , the windows

C E
7940

A had no shutters, and the lawn overrun by weeds, B D it was unoccupied. No error E C people passing by the old house assumed that

powerful means of identification. No error D

7895

7900

7905

7945

18. Until recently, most people entering politics feel that A


7950

799522. It was a Chinese American grower who finally

B D E
8000

A succeeded with adapting the now familiar B D C E orange tree to the American climate. No error

loss of privacy was a fair price to pay for the chance C to participate in policy making. No error

19. Only by tapping their last reserves of energy were the


7955

23. The survey indicated that workers in the United States hope that his or her wages will keep pace with
8005

A C to look like a lost cause. No error E D

B A D B E C

team members able to salvage what was beginning

the rising cost of living. No error

7960

20. When Doris Lessing published The Golden Notebook


8010

24. In Angkor, Cambodias ancient city, a clever A designed reservoir, five miles long and one mile wide, supplied fish and helped farmers to produce B
8015

in 1962, it instantly established herself as one of A


7965

B D

C C E 25. Last summer, when Marys aunt and uncle


8020

the most important literary voices of her generation. No error E

three crops of rice annually. No error

797021. Not many authors have described the effects

B flew from Turkey to visit their relatives and tour A D B C her house to meet them. No error
8025

of environmental pollution as effective as C Rachel Carson, whose work is still a model for
7975

the United States, Mary invited Sandhya and I to

nature writers. No error E

7980

7985

7990

8030

26. Ongoing research by several scientists suggest that A C B regular periods of meditation reduce blood pressure

Directions: The following passage is an early draft of an essay. Some parts of the passage need to be rewritten. Read the passage and select the best answers for the questions that follow. Some questions are about particular sentences or parts of sentences and ask you to improve sentence structure or word choice. Other questions ask you to consider organization and development. In choosing answers, follow the requirements of standard written English.

8035

and are likely to contribute to other improvements D in health. No error E

8080
804027. Because the American Indian rodeo includes games

Questions 30-35 are based on the following passage. and exhibitions developed as early as the seventeenth A century, they predate by a few hundred years
8045

B D

C E

the form of rodeo now seen on television. No error

28. Five years in the writing , her new book is


8050

A both a response to her critics mistrust with B D C her earlier findings and an elaboration of her

8055

original thesis. No error E 29. Despite its cultural importance, the Daily Gazette A

8060

lost 70 percent of its subscribers since 1920 and, B by 1955, was losing as much as $200,000 a year. C No error D

(1) No one owns Antarctica. (2) The nations of 8085the world agreedsome of them reluctantlythat all countries would share the continent for the purposes of scientific research. (3) Governed by the Antarctic Treaty, written in 1959 and adopted in 1961, which has been signed by 27 countries. (4) Another 17 countries have 8090agreed to abide by the treaty in order to participate in research being done in Antarctica. (5) In Antarctica, relations among the researchers and their countries are both simpler and more complicated than in the rest of the world. (6) Relations are simpler because 8095each country has only a few scientists on this isolated continent. (7) Treaty clauses assure that the research there is nonmilitary. (8) On the other hand, when conflicts do arise, there is no clear process for dealing with them. (9) Decisions that can make or break the preservation 8100of Antarcticas unique environment and its scientific opportunities depend on a political system designed to have nobody in command. (10) Clear decision making has become a more urgent challenge as more tourists are attracted to Antarctica. 8105(11) Scientists living on Antarctica were not always as careful to preserve the pristine environment as they are now. (12) Tour operators are working with treaty members to devise regulations, and there are plans to assess the environmental impacts of tours. (13) But regulations 8110and assessment plans may prove difficult to settle on and enforce in a place where jurisdiction is unclear. 30. What must be done to sentence 3 ?
8115

8065

8070

8120

(A) Combine it with sentence 2, putting a comma after research. (B) Begin it with the words Antarctica is. (C) Replace the comma after 1961 with a semicolon. (D) Place the words it was before written. (E) Change 1961, which to 1961. It.

8075

220

812531. What should be done with sentence 7 ?

818534. Which sentence should be deleted?

8130

(A) Change assure to make sure. (B) Change Treaty clauses to The clauses agreed to by the diverse countries represented there. (C) Place sentence 7 before sentence 6. (D) Combine sentence 7 with sentence 6 by changing the period after continent to a comma. (E) Combine sentence 7 with sentence 6 by changing continent. Treaty to continent and also because treaty. 32. In context, which is the best revision of the underlined portion of sentence 9 (reproduced below) ? Decisions that can make or break the preservation of Antarcticas unique environment and its scientific opportunities depend on a political system designed to have nobody in command. (A) (B) (C) (D) (E) create or destroy support or dispute be determined by be critical to be an adjustment to

8190

(A) (B) (C) (D) (E)

1 2 11 12 13

35. The third paragraph would be improved if which of the following sentences were added?
8195

8135

8140

8200

(A) Scientists collect ice cores by driving a hollow tube deep into the miles-thick ice sheets. (B) If the West Antarctic ice sheet melted, global seas would rise by 15 to 20 feet. (C) It is the driest place in the world, yet it contains 70 percent of Earths freshwater. (D) One cruise ship encountered 30-foot waves all the way across the Drake Passage. (E) Last year, more than 10,000 tourists visited Antarctica, bringing soiled boots, climbing gear, and trash to many locations.

8145

8205

33. The purpose of the second paragraph is to


8150

8155

(A) present a situation that has contradictory elements (B) offer a solution to a problem discussed in the first paragraph (C) present an argument and its final resolution (D) examine a theory in light of new discoveries (E) discuss a theory that will be refuted in the third paragraph

8160

8165

8170

8175

8180

SECTION 7
8210

CRITICAL READING
24 Questions

Time 25 minutes

Directions: For each question in this section, select the best answer from among the choices given and fill in the corresponding circle on the answer sheet.

8215
Each sentence below has one or two blanks, each blank indicating that something has been omitted. Beneath the sentence are five words or sets of words labeled A through E. Choose the word or set of words that, when inserted in the sentence, best fits the meaning of the sentence as a whole. Example: Hoping to ------- the dispute, negotiators proposed a compromise that they felt would be ------- to both labor and management. (A) (B) (C) (D) (E) enforce .. useful end .. divisive overcome .. unattractive extend .. satisfactory resolve .. acceptable

8245

2. The new antifungal agent has such ------- uses, from treating Dutch elm disease to rescuing water-damaged works of art from molds, that it is considered one of the more ------- antibiotics. (A) disturbing . . explicit (B) innovative . . precipitous (C) mysterious . . recognized (D) varied . . versatile (E) similar . . discriminating 3. The child had a tendency toward aggressive behavior, a ------- fighting rather than resolving differences amicably. (A) propensity for (B) confusion about (C) disregard of (D) hostility toward (E) compunction about 4. Physical exercise often has a ------- effect, releasing emotional tension and refreshing the spirit.

8250

8255

8260

1. Black Americans in Flight, a mural honoring several aviation pioneers, also ------- the 1992 spaceflight of 8220 astronaut Mae Jemison. (A) discerns (B) introduces (C) approximates (D) commemorates (E) asserts

8265

(A) pejorative (B) debilitating (D) retentive (E) tenacious

(C) cathartic

5. Because rap and hip-hop offer such ------- commentary on contemporary issues, they are often said to be 8270 sharp-edged musical genres. (A) nebulous (B) trenchant (D) prosaic (E) benign (C) circumspect

8225

8230

8235

8240

8275

225
Each passage below is followed by questions based on its content. Answer the questions on the basis of what is stated or implied in the passages and in any introductory material that may be provided.

Questions 6-7 are based on the following passage. Mechanical pencils rule, my fifteen-year-old grandniece, Genevieve, declared when I invited her to 8280be her generations voice on school supplies. Nobody sharpens anymore. Then, continuing with a fashion mavens hyperbole and arbitrary imperatives, she gave a passionate disquisition on types of clickers, new grips, smaller lead sizes, and other niceties of pencil selection. 8285As she consigned the yellow-painted wooden pencil to the wastebasket of history, I felt a rush of nostalgia for the perfumed sharpener shavings of my youth. 6. In lines 4-5, the author refers to a fashion mavens tone primarily in order to (A) imply that Genevieve has only a superficial appreciation of mechanical pencils (B) suggest that Genevieve is excessively concerned about her clothing (C) illustrate some of the exaggerated claims made by mechanical pencil manufacturers (D) emphasize the unpredictability of trends in consumer tastes (E) indicate that Genevieve expresses her opinions with authority and flair 7. The author mentions sharpener shavings (line 10) in order to portray a mood of (A) unrestrained joy (B) sentimental reminiscence (C) bitter disappointment (D) cautious optimism (E) dark foreboding

Questions 8-9 are based on the following passage. Black holes are the most efficient engines of destruction known to humanity. Their intense gravity is a one-way 8290ticket to oblivion, and material spiraling into them can heat up to millions of degrees and glow brightly. Yet, they are not all-powerful. Even supermassive black holes are minuscule by cosmic standards. They typically account for less than one percent of their galaxys mass. Accordingly, 8295astronomers long assumed that supermassive holes, let alone their smaller cousins, would have little effect beyond their immediate neighborhoods. So it has come as a surprise over the past decade that black hole activity is closely intertwined with star formation occurring farther out in the 8300galaxy. 8. Which best describes the function of the statement in lines 20-23 (So it . . . galaxy) ? (A) It summarizes the points made in the first four lines of the passage. (B) It provides support for the argument asserted in the preceding statement. (C) It introduces a new view of information presented earlier in the passage. (D) It challenges recent scientific findings. (E) It offers examples to support a theory. 9. Which of the following most resembles the relationship between black hole activity and star formation (lines 21-22) as described in the passage? (A) A volcanic eruption on one continent results in higher rainfall totals on another continent. (B) Industrial emissions in one region lead to an increase in airborne pollutants in adjacent regions. (C) A drought in a wilderness area causes a significant loss of vegetation in that area. (D) Decreased oil production in one country results in higher gas prices in oil-dependent countries. (E) Overfishing in a gulf leads to an increase in the population of smaller aquatic organisms.

Questions 10-15 are based on the following passage. The following passage is an excerpt from a 1909 novel. Georgia, the main character, is a reporter in an otherwise all-male newsroom. Georgia was to be married. It was the week before Christmas, and on the last day of the year she would become Mrs. Joseph Tank. She had told Joe that if they were to be married at all they might as well get it 8305over with this year, and still there was no need of being married any earlier in the year than was necessary. She assured him that she married him simply because she was tired of having paper bags waved before her eyes everywhere she went and she thought if she were once officially 8310ssociated with him people would not flaunt his idiosyncrasies at her that way. And then Ernestine, her best friend, approved of getting married, and Ernestines ideas were usually good. To all of which Joe responded that she certainly had a splendid head to figure it out that way. 8315Joe said that to his mind reasons for doing things werent very important anyhow; it was doing them that counted. Yesterday had been her last day on the paper. She had felt queer about that thing of taking her last assignment, though it was hard to reach just the proper state, for the 8320last story related to pork-packers, and pork-packing is not a setting favorable to sentimental regrets. It was just like the newspaper business not even to allow one a little sentimental harrowing over ones exodus from it. But the time for gentle melancholy came later on when she was 8325sorting her things at her desk just before leaving, and was wondering what girl would have that old deskif they cared to risk another girl, and whether the other poor girl would slave through the years she should have been frivolous, only to have some man step in at the end and 8330induce her to surrender the things she had gained through sacrifice and toil. As she wrote a final letter on her typewritershe did hate letting the old machine goGeorgia did considerable philosophizing about the irony of working for things only 8335to the end of giving them up. She had waded through snowdrifts and been drenched in pouring rains, she had been frozen with the cold and prostrated with the heat, she had been blown about by Chicago wind until it was strange there was any of her left in one piece, she had had front 8340doorsyes, and back doors tooslammed in her face, she had been the butt of the alleged wit of menials and hirelings, she had been patronized by vapid women as the poor girl who must make her living some way, she had been roasted bybut never mindshe had had 8345a beat* or two! And now she was to wind it all up by marrying Joseph Tank, who had made a great deal of

money out of the manufacture of paper bags. This from herwho had always believed she would end her days in New York, or perhaps write a realistic novel exposing 8350some mighty evil!
* the area regularly covered by a reporter

10. Based on information presented in the passage, which best describes what Georgia was tired of (line 8) ? (A) Being forced to earn a living (B) Being teased about Joseph Tank (C) Being considered a hack writer by some of her colleagues (D) Being betrayed by her supposed friends (E) Being the only woman in the newsroom 11. The second paragraph suggests that Georgia believes the proper state (line 19) would be one of (A) (B) (C) (D) (E) excitement wistfulness amusement annoyance relief

12. In line 27, poor most nearly means (A) (B) (C) (D) (E) pitiable indigent inferior humble petty

13. Which most resembles the irony mentioned in line 34 ? (A) A worker moving to a distant state to take a job, only to be fired without warning (B) An executive making an important decision, only to regret it later (C) An athlete earning a starting position on a good team, only to quit in midseason (D) A student studying for a major exam, only to learn that it has been postponed (E) A person purchasing an expensive umbrella, only to lose it on the first rainy day

14. The description in lines 35-45 (She . . . two!) primarily serves to


8355

841515. In context, the phrase This from her (lines 47-48) helps to suggest that a

8360

(A) suggest that Georgia envied those women who did not have to work (B) imply that Georgia would be unlikely ever to consider working as a reporter again (C) indicate the role that weather plays in the everyday life of a reporter (D) exaggerate Georgias reluctance to relinquish her job (E) how the adversities Georgia had to overcome as a reporter

8420

(A) (B) (C) (D) (E)

specific feeling is quite heartfelt stated viewpoint is highly personal certain decision is out of character particular behavior is extremely upsetting given attitude is unsurprising

8425

8365

8370

8375

8380

8385

8390

8395

8400

8405

8410

230

Questions 16-24 are based on the following passage. The following passage is adapted from a book about television and popular culture. Ridiculing television, and warning about its inherent evils, is nothing new. It has been that way since the medium was invented, and television hasnt exactly been lavished with respect as the decades have passed. 8430I suspect, though, that a lot of the fear and loathing directed at television comes out of a time-honored, reflexive overreaction to the dominant medium of the moment. For the past several decades, television has been blamed for corrupting our youth and exciting 8435our adults, distorting reality, and basically being a big, perhaps dangerous, waste of time. Before TV, radio and film were accused of the same things. And long before thatin fact, some 2,500 years earlier philosophers were arguing that poetry and drama 8440should be excluded from any ideal city on much the same grounds. In Book 10 of the Republic, Plato (428-348 B.C.) attacks epic poet Homer (c. 850 B.C.) and the tragedians on several grounds, all of which have a familiar 8445ring. Their productions are appearances and not realities, he gripes. Drawing, and in fact all imitation . . . [is] quite removed from the truth. The audience, as well as the art form, troubled Plato, whose remarks are colored by an implied disdain for the popularity of public performances. 8450The common people, as Plato so charitably calls them, are drawn to peevish and diverse characterssuch as Odysseus and other heroes in the Iliad and the Odyssey who (to Plato, anyway) engage in such questionable displays of emotion as spinning out a long melancholy 8455lamentation or disfiguring themselves in grief. To Plato, baring such intimate sorrows is not to be condoned. (Clearly, he would have given thumbs down to the central characters of Shakespeares Hamlet and Macbeth.) If you receive the pleasure-seasoned Muse1 of song and epic, 8460Plato warns, pleasure and pain will be kings in your city, instead of law. Finally, Plato sums up his anti-arts argument with the cold, sweeping pronouncement that poetry is not to be taken seriously. One academic who has studied and written extensively 8465about both Plato and television suggests that Plato, rather than being anti-arts, was merely an elitist. Plato wanted to ban poetry readings and live theater, the argument goes, because, being free and accessible and raucous and extremely popular, they were the mass entertainment 8470of that era. If, instead of tragedy and poetry, and Homer and Aeschylus,2 you read mass entertainment or popular media, youll recognize Platos arguments as the ancestor of all the reasons we have today for being suspicious of television. 8475 To wit: poetry, by which Plato means drama, confuses us between appearance and reality. The action it presents

is too extreme and violent. Most important, its a corrupting influence, perverting its audience by bombarding it with inferior characters and vulgar subjectsand con8480stituting, in Platos own words, a harm to the mind of its audience. If Platos Republic had become reality, it would have been a republic with a lot of empty libraries, theaters, and museumsif, indeed, those repositories of the arts would 8485have survived at all. Platos personal utopia never came to passbut throughout the centuries, wherever and whenever a new medium of artistic expression attracted a lot of people, someone has been ready, waiting, and eager to attack its content and fear its impact.
1 2

The Muses inspired poetry and song in Greek mythology. Aeschylus (525-456 B.C.) was a Greek tragic dramatist.

16. The opening paragraph primarily serves to (A) criticize the way television distorts the truth (B) examine the evolution of television as a medium (C) place contemporary criticism of television in a historical context (D) directly compare television and drama as art forms (E) explain why television, radio, and drama appeal to the masses 17. Which of the following television shows would be LEAST vulnerable to the criticism expressed in lines 8-11 (For . . . time) ? (A) A melodrama in which police detectives attempt to solve crimes (B) A soap opera depicting interpersonal conflicts in a fictional law firm (C) A comedy whose primary characters are supernatural (D) A documentary on the state of education in the nation (E) A talk show that encourages people to confront each other in front of a studio audience 18. In line 26, drawn most nearly means (A) (B) (C) (D) (E) brought depicted selected attracted shaped

8490

19. Which of the following best characterizes Platos view of the heroes mentioned in line 27 ?
8495

(A) (B) (C) (D) (E)

Admiration Curiosity Distrust Disappointment Contempt

22. The fourth paragraph (lines 50-56) indicates that Platos principal objection to poetry (line 50) 8555 was its (A) (B) (C) (D) (E) confusing language widespread popularity depiction of turbulent events influence on peoples morals misrepresentation of historical figures

8560 850020. The academic (line 39) indicates that Plato was primarily characterized by his

8505

(A) (B) (C) (D) (E)

insight artistry cynicism irreverence snobbishness

23. The author of the passage would probably agree with which of the following statements about the utopia referred to in line 60 ?
8565

21. The primary purpose of the statements in lines 39-45 (One . . . that era) is to
8510

8570

8515

(A) provide an interpretation of a viewpoint described in the previous paragraph (B) show how Platos view of politics should be understood in todays terms (C) put divergent interpretations of Plato into historical perspective (D) account for the appeal of Platos writings (E) signal a digression in the passage

(A) It would have encouraged new artistic ventures. (B) It would have stifled human creativity. (C) It is an ideal that we should continue to work towards. (D) It may come to pass because of the popularity of television. (E) It was a notion rejected by Greek philosophers. 24. The comment about a new medium of artistic expression (line 62) primarily suggests that

8575

8580 8520

(A) the author holds a fatalistic view of the future for artistic expression (B) certain societies in the past have been slow to accept new art forms (C) people often disguise their true feelings when it comes to art (D) the popular response to a new art form will often overcome opposition to it (E) a popular new art form will always receive some form of negative response

8585 8525

8530

8535

8540

8545

8550

235 8590 SECTION 8 CRITICAL READING


19 Questions
Directions: For each question in this section, select the best answer from among the choices given and fill in the corresponding circle on the answer sheet.

Time 20 minutes

8595
Each sentence below has one or two blanks, each blank indicating that something has been omitted. Beneath the sentence are five words or sets of words labeled A through E. Choose the word or set of words that, when inserted in the sentence, best fits the meaning of the sentence as a whole. Example: Hoping to ------- the dispute, negotiators proposed a compromise that they felt would be ------- to both labor and management. (A) (B) (C) (D) (E) enforce .. useful end .. divisive overcome .. unattractive extend .. satisfactory resolve .. acceptable
8640

8630

3. Since other seabirds customarily nest in colonies on ocean cliffs and islands, the marbled murrelets ------nesting in forests many miles from the sea must be considered -------. (A) (B) (C) (D) (E) ambivalence about . . hypothetical indifference to . . bold insistence upon . . evident aversion to . . dangerous predilection for . . atypical

8635

4. The cause of Mozarts ------- is a long-standing medical -------: over the years, physicians have suggested more than 100 possibilities, including poisoning, malnutrition, kidney disease, and heart failure. (A) (B) (C) (D) (E) mortality . . phenomenon bereavement . . controversy genius . . enigma demise . . mystery death . . trial

8645 86001. As a child, Mary ------- her stringent upbringing; however, as she grew older she began to appreciate her grandmothers strict -------.

8605

(A) (B) (C) (D) (E)

tolerated . . autonomy despised . . discipline embraced . . authority disavowed . . abstinence loathed . . liberality

86505. At first the children were -------, but as the morning progressed they began to laugh and talk eagerly.

(A) ostentatious (B) myopic (D) puerile (E) reticent

(C) solicitous

2. His mouth stinging and burning, Virgil hurried to the 8610 kitchen for water to wash away the dishs ------- taste. (A) earthy (D) acrid (B) exotic (C) cloying (E) succulent

86556. Oren missed the plays overarching significance, focusing instead on details so minor that they would best be described as -------.

(A) pragmatic (D) picayune


8660

(B) indelible (C) moribund (E) impervious

8615

8620

8625

The passage below is followed by questions based on its content. Answer the questions on the basis of what is stated or implied in the passage and in any introductory material that may be provided.
8665

Questions 7-19 are based on the following passage. The following passage is from a 1979 essay by a Native American writer. An understanding of any national literature depends very much on an awareness of the larger cultural context. Without some knowledge of language, of history, of 8670inflection, of the position of the storyteller within the group, without a hint of the social roles played by males and females in the culture, without a sense of the societys humor or prioritieswithout such knowledge, how can we, as reader or listener, penetrate to the core of meaning 8675in an expression of art? The difficulty of gaining access to the literature of a different culture may be illustrated by an exemplary folktale (in translation) from the Tanaina (Athabaskan) culture of south-central Alaska. It would typically be told to a 8680general audience within the society, including the full range of ages from young children to grandparents; it would be recounted with gesticulation and exaggeration by a performance specialist. It would be expected to have different meanings to the various categories of listeners instructive, 8685entertaining, reinforcing, or all three. Here is a brief version of the story: Once upon a time there was a porcupine woman who decided to do some hunting on the far side of the river. She went to the bank, where she met a beaver. 8690 Hello, she said to him. I need to do some hunting over there. Will you ride me across on your back? Id be glad to, replied the beaver. Hop on. So the porcupine woman climbed on his back, and he started swimming for the other side. When he had almost 8695made it, the porcupine woman said, Oh my! Ive forgotten to bring my sack. Ill need to go back to the other bank and get it. All right, said the beaver, and swam back. He was panting while the porcupine woman went to get her sack. 8700 Okay, she said. Lets go. So they started across again. The beaver was swimming much more slowly. When they had practically reached the other side, she said, Oh my! Ive forgotten to bring my needle. Well have to go back and get it. 8705 This time the beaver didnt say anythinghe didnt have enough breath! But he turned around and pulled them back to the shore and nearly passed out while she got her needle.

Hurry up, now, the porcupine woman said as she 8710climbed back on his back. He could hardly keep his nose above water, but he had almost made it to the far bank again when she said, Oh my! Ive forgotten my staff. Well have to . . . . Before she had finished her sentence the beaver had 8715flipped over in the water and dragged himself onto the bank, where he lay half dead. The porcupine woman managed to make the shore too, and climbed up onto a bear path. When she had caught her breath, she turned on the beaver and quilled him to death. 8720 The Tanaina live in an environment that could euphemistically be described as difficult. Survival, especially in the wild, is always precarious. Further, they were, in the precontact period, a nonliterate people. Oral communication was therefore the method of cultural transmission, 8725legal understanding, and meaningful communication. It is also necessary to know that a staff, as mentioned in the story, functions as both a walking stick and a weapon, and that in the Tanaina symbol system, porcupines were supposed to be rather ponderous, dull-witted creatures, and 8730beavers were thought to be energetic and industrious but overly spontaneous and erratic. For the reader armed with these data, the story becomes more accessible as a lesson in contract law, with several additional minor themes. A culturally attuned listener 8735would notice, for instance, that when the porcupine woman proposed passage to the beaver, he agreed without any stipulations or clarifications of the terms. He gave a basically open-ended agreementmade a contractand hence the porcupine woman was perfectly within her rights 8740both in demanding that he return three times and in quilling him to death when he reneged. The story is not, however, without its moral for the porcupine women of this world. Her stated aim is to go hunting, and yet she sets out without the three essentials of 8745that endeavor: a sack in which to carry home her game, a needle with which to sew up the intestines, and, most important, an implement with which to hunt and defend herself. True, she had an open-ended contract, but where does she wind up at the conclusion of the story? Sitting, 8750exhausted, quills used up, weaponless, and not only on the wrong side of the river from her home but on a bear path! The hunter is about to become the hunted, and all because of her own improvidence.

8755

7. In the opening paragraph, the author assumes that the meaning (line 8) is (A) (B) (C) (D) (E) culturally determined intensely personal essentially moralistic permanently inscrutable uniquely artistic

8815

12. In relation to the passage, the statements in lines 59-65 serve a function most similar to which of the following items? (A) (B) (C) (D) (E) A menu in a restaurant The key or legend to a map A department store directory The outline of a term paper An illustration of a fairytale

8760

8820

8765

8. In the context of the passage, which expression of art (line 9) would be the most difficult to interpret? (A) A contemporary play written by a prolific playwright (B) A fable from a nonliterate society with which anthropologists are very familiar (C) A single text produced by a previously unknown society (D) A sitcom from the early days of television (E) A single myth from an ancient culture with a welldocumented mythological structure 9. How does the author respond to the question posed in lines 3-9 ? (A) By proposing an innovative strategy (B) By confirming the futility of such analysis (C) By describing a personal experience with the problem (D) By illustrating his point within a particular context (E) By documenting a traditional approach to the problem

882513. The authors analysis of the folktale offers which insight into Tanaina beliefs?

8770

8830

8775

8835

(A) A fanciful story is most suitable for an audience of children. (B) A verbal exchange can establish a binding contract. (C) A person who behaves impulsively is most often sincere. (D) A shared task should be divided fairly between two people. (E) A painstaking plan may nonetheless fail to anticipate all problems. 14. The porcupine women of this world (lines 76-77) are best described as people who

8780

8840

8785 10. The author discusses Tanaina culture from the perspective of

(A) (B) (C) (D) (E)

plan inadequately for their own needs postpone necessary work in favor of leisure depend heavily upon help from their close friends return repeatedly to their favorite places flee quickly from any laborious task

8845

8790

(A) (B) (C) (D) (E)

a concerned parent a bewildered visitor a performance artist an informed outsider an indignant reader

15. The final paragraph (lines 76-87) suggests that the bear path mentioned in lines 51-52 is significant because it (A) foreshadows the arrival of a benevolent character from Tanaina folklore (B) suggests an alarming alternative to crossing the river (C) marks the boundary of the beavers natural surroundings (D) explains the porcupine womans fear of unfamiliar territory (E) poses a new peril for the porcupine woman

8850

11. The sentence in which difficult appears (lines 54-55) 8795 indicates that the author considers the word to be (A) (B) (C) (D) (E) an exaggeration an estimate an understatement a contradiction a preconception
8855

8800

8805

8810

240

8920 8860

16. In lines 83-87, the description of the porcupine woman emphasizes the discrepancy between her (A) (B) (C) (D) (E) social position and her private feelings physical wealth and her moral poverty hostile action and her ultimate gratitude original goal and her actual situation grandiose ambition and her real needs
8925

18. The authors attitude toward the Tanaina folktale is best described as (A) excitement at an unexpected discovery (B) admiration of the storytellers performance (C) appreciation of the folktale as a means of communicating values (D) enthusiasm for the Tanaina cultures concept of legality (E) enjoyment of the comical aspects of the folktale 19. Which statement is most consistent with the authors argument?
8935

8865

8870

17. As a commentary on legal relations, this folktale is best described as (A) (B) (C) (D) (E) an example of traditional practices an outline for social behavior a warning about ill-conceived assent a criticism of obsolete customs parody of actual situations

8930

8875

8940 8880

(A) Translating a literary text requires formal linguistic training. (B) Tales transmitted by a nonliterate society elude transcription in later eras. (C) Listening to a skilled storyteller is more instructtive than entertaining. (D) Simple enjoyment of a tale is incompatible with scholarly analysis. (E) To read a text is not necessarily to understand it.

8945 8885

8890

8895

8900

8905

8910

8915

SECTION 10
8950

WRITING

Time 10 minutes 14 Questions

Directions: For each question in this section, select the best answer from among the choices given and fill in the corresponding circle on the answer sheet.

8955

8975

The following sentences test correctness and effectiveness of expression. Part of each sentence or the entire sentence is underlined; beneath each sentence are five ways of phrasing the underlined material. Choice A repeats the original phrasing; the other four choices are different. If you think the original phrasing produces a better sentence than any of the alternatives, select choice A; if not, select one of the other choices. In making your selection, follow the requirements of standard written English; that is, pay attention to grammar, choice of words, sentence construction, and punctuation. Your selection should result in the most effective sentence clear and precise, without awkwardness or ambiguity. EXAMPLE: Laura Ingalls Wilder published her first book and she was sixty-five years old then. (A) and she was sixty-five years old then (B) when she was sixty-five (C) at age sixty-five years old (D) upon the reaching of sixty-five years (E) at the time when she was sixty-five

3. During the Fourth of July weekend in 1947, about six million people crowded onto the beach and into the amusements at Coney Island, Americas Playground, in Brooklyn, New York. (A) During the Fourth of July weekend in 1947, (B) It was the Fourth of July weekend in 1947 that (C) The Fourth of July weekend that occurred in 1947 was when (D) The Fourth of July weekend in 1947, (E) Occurring in 1947 over the Fourth of July weekend,

8980

8985

4. Because fiscal problems will force some cities to lay off firefighters, and so the state legislature must decide 8990 whether to provide those cities with financial aid. (A) (B) (C) (D) (E) firefighters, and so firefighters is a matter firefighters, firefighters; then firefighters; this is a problem

8995

5. A recently published history of comic books reveal that Batman was begun as an experiment but became an institution.
9000

1. The Amazon River carries more water than the water carried in any of the worlds other rivers.
8960

9005

(A) (B) (C) (D) (E)

the water carried in any of the worlds other rivers the water elsewhere in the worlds rivers any other river in the world all rivers in the world any of the worlds rivers elsewhere

9010

(A) recently published history of comic books reveal that Batman was begun as an experiment (B) recently published history of comic books reveals that Batman began as an experiment (C) recent published history of comic books revealed that Batman, who began as an experiment (D) history of comic books, recently published, revealing Batman first began as an experiment (E) history of comic books having been recently published, it reveals how Batman began as an experiment

89652. The owners of stadiums that bear the names of now bankrupt companies have a problem what to do about the names.

8970

(A) (B) (C) (D) (E)

what to do about the names what they should do about the names deciding what to do about those names to decide as to whether the names should stay should they change those names or not

245
90156. A discovery in New Jersey actually contributed to the early economic development of America and, in 1714, a worker uncovered a green rock containing copper. 907510. Santa Fe is one of the oldest cities in the United States, its adobe architecture, spectacular setting, and clear, radiant light have long made it a magnet for artists.

9020

(A) (B) (C) (D) (E)

America and, in 1714, a worker uncovered America when, in 1714, a worker uncovered America, thus, in 1714, a worker uncovered America, that being a worker in 1714 uncovering America, it was in 1714 a worker uncovered

9080

9025

7. Selected as an astronaut by NASA in 1990, over 719 hours in space were spent by Dr. Ellen Ochoa on three flights by 2001. (A) over 719 hours in space were spent by Dr. Ellen Ochoa on three flights by 2001 (B) by 2001, and on three flights, Dr. Ellen Ochoa spent over 719 hours in space (C) three flights and 719 hours were spent by Dr. Ellen Ochoa in space by 2001 (D) Dr. Ellen Ochoa, by 2001 spending over 719 hours in space on three flights (E) Dr. Ellen Ochoa had spent over 719 hours in space on three flights by 2001

9085

(A) Santa Fe is one of the oldest cities in the United States, its (B) Santa Fe, which is one of the oldest cities in the United States, its (C) Santa Fe, which is one of the oldest cities in the United States, has (D) Santa Fe is one of the oldest cities in the United States; its (E) Santa Fe, one of the oldest cities in the United States, and its 11. Dime novels, known in nineteenth-century England as penny dreadfuls, flourished because increased mechanization of printing and increased literacy rates made production of large numbers of these books profitable. (A) (B) (C) (D) (E) and increased literacy rates made with increased literacy rates also made and also literacy rates increased and made as well as increased literacy rates, making and literacy rates increased, making

9030

9090

9035

9095

8. The old maxim Let the buyer beware suggests that as a buyer we are responsible for inspecting merchandise 9040 for flaws before paying for it. (A) (B) (C) (D) (E) as a buyer we are responsible as a buyer it is their responsibility the buyer is the responsible one buyers are responsible buyers are to be the ones responsible

910012. Frequently on tour, a band called the Chieftains revered internationally as spirited performers of traditional Irish music.

9045

9105

9. The same analysts who once favored public-opinion polls now see them as hampering representative government.
9050

9055

(A) The same analysts who once favored public-opinion polls now see them (B) The same analysts which were once in favor of public-opinion polls now have come to see them (C) Public-opinion polls, once favored by analysts, but now seen by these same analysts (D) As for favoring public-opinion polls, the same analysts now see them (E) Analysts once were in favor of public-opinion polls, which these same analysts now see

9110

(A) revered internationally as spirited performers (B) revered internationally and they are spirited performers (C) is revered internationally for its spirited performances (D) is revered internationally as giving spirited performances (E) are revered internationally as being spirited performers

9060

9065

9070

9115

13. Psychologists advise that before making any major changes in your life, a person needs to focus on ones goals.
9120

9180

14. Some beaches are frequently contaminated by untreated sewage that flows into the ocean, which can last for several days. (A) (B) (C) (D) (E) ocean, which can last for several days ocean that can last for several days ocean, it can last for several days ocean, while contamination can last several days ocean; the contamination can last for several days

(A) (B) (C) (D) (E)

your life, a person needs to focus on ones goals their life, a person needs to focus on their goals their lives, focus on the goals ones life, you should focus on your goals their lives, people should focus on their goals

9185

9125

9130

9135

9140

9145

9150

9155

9160

9165

9170

9175

Practice Test 6 9190 SECTION 1 WRITING ESSAY

Time 25 minutes

The essay gives you an opportunity to show how effectively you can develop and express ideas. You should, therefore, take 9195care to develop your point of view, present your ideas logically and clearly, and use language precisely. Your essay must be written on the lines provided on your answer sheet you will receive no other paper on which to write. You will have enough space if you write on every line, avoid wide margins, and keep your handwriting to a reasonable size. 9200Remember that people who are not familiar with your handwriting will read what you write. Try to write or print so that what you are writing is legible to those readers. You have twenty-five minutes to write an essay on the topic assigned below. DO NOT WRITE ON ANOTHER TOPIC. 9205AN OFF-TOPIC ESSAY WILL RECEIVE A SCORE OF ZERO. Think carefully about the issue presented in the following quotations and the assignment below.
9210

Nowadays nothing is private: our culture has become too confessional and self-expressive. People think that to hide one s thoughts or feelings is to pretend not to have those thoughts or feelings. They assume that honesty requires one to express every inclination and impulse.
9215

Adapted from J. David Velleman, The Genesis of Shame

Assignment:
9220

Should people make more of an effort to keep some things private? Plan and write an essay in which you develop your point of view on this issue. Support your position with reasoning and examples taken from your reading, studies, experience, or observations.

250

Practice Test 6 9225 SECTION 2 CRITICAL READING


24 Questions
Directions: For each question in this section, select the best answer from among the choices given and fill in the corresponding circle on the answer sheet.
9230 9260

Time 25 minutes

Each sentence below has one or two blanks, each blank indicating that something has been omitted. Beneath the sentence are five words or sets of words labeled A through E. Choose the word or set of words that, when inserted in the sentence, best fits the meaning of the sentence as a whole. Example: Hoping to ------- the dispute, negotiators proposed a compromise that they felt would be ------- to both labor and management. (A) (B) (C) (D) (E) enforce .. useful end .. divisive overcome .. unattractive extend .. satisfactory resolve .. acceptable

4. Folk painter Grandma Moses has become such an enduring icon that many consider her ------of America.
9265

(A) an innovator (B) an emblem (C) a successor (D) a detractor (E) a lobbyist

5. Whether substances are medicines or poisons often depends on dosage, for substances that are ------- in 9270 small doses can be ------- in large. (A) (B) (C) (D) (E) useless .. effective mild .. benign curative .. toxic harmful .. fatal beneficial .. miraculous

9275

6. Critics dismissed the engineers seemingly creative design as being -------, that is, underdeveloped and lacking in sophistication.
9280

92351. Years of ------- lifting of heavy furniture had left him too ------- to be able to stand erect for long periods of time.

(A) defunct (B) unorthodox (C) simplistic (D) erroneous (E) ambiguous

9240

(A) (B) (C) (D) (E)

profitable .. dumbfounded generous .. distracted onerous .. hesitant strenuous .. debilitated unstinting .. eminent

7. The professor commented to other faculty members that Sheila seemed temperamentally suited to the study of 9285 logic, given her ------- for ------- intricate arguments. (A) (B) (C) (D) (E) sympathy .. influencing penchant .. evading disregard .. unhinging contempt .. following bent .. analyzing

9245

2. Canadian Lynn Johnston was named Cartoonist of the Year in 1985, the first woman to be so -------. (A) inspired (B) entrusted (D) employed (E) refined (C) honored

9290

8. While traveling near the Sun, the comet Hale-Bopp produced a ------- amount of dust, much more than the comets Halley or Hyakutake.
9295

9250

3. Because the photographer believed that wild animals should be ------- only in their various natural surroundings, she ------- often in her career. (A) (B) (C) (D) (E) depicted .. traveled displayed .. spoke captured .. protested domesticated .. roamed represented .. publicized

(A) voracious (D) superficial

(B) disposable (E) prodigious

(C) redundant

9255

The passages below are followed by questions based on their content; questions following a pair of related passages may also be based on the relationship between the paired passages. Answer the questions on the basis of what is stated or implied in the passages and in any introductory material that may be provided.
9300

Questions 9-10 are based on the following passage. Newspaper editor and political commentator Henry Louis Mencken was a force of nature, brushing aside all objects animal and mineral in his headlong rush to the publicity that surely awaited him. He seized 9305each day, shook it to within an inch of its life, and then gaily went on to the next. No matter where his writing appeared, it was quoted widely, his pungently outspoken opinions debated hotly. Nobody else could make so many people so angry, or make so many others 9310laugh so hard. 9. In lines 4-5, the words seized and shook help establish which aspect of Menckens personality? (A) (B) (C) (D) (E) His code of honor His sense of humor His vindictiveness His intensity His petulance

Questions 11-12 are based on the following passage. The ability to see the situation as your opponents see it, as difficult as it may be, is one of the most important skills that you can possess as a negotiator. You must know more than simply that they see things differently. It is not 9315enough to study them like beetles under a microscope; you need to know what it feels like to be a beetle. To accomplish this you should be prepared to withhold judgment as you "try on" their views. Your opponents may well believe that their views are right as strongly 9320as you believe yours are. 11. The reference to beetles in lines 15-16 serves to suggest that (A) people need to be more attuned to their surroundings (B) effective negotiation is more of a science than an art (C) people can be made to do what they would prefer not to do (D) effective negotiation requires identifying with a different viewpoint (E) people feel uncomfortable when their actions are under scrutiny 12. The primary purpose of the passage is to (A) persuade people to defend their positions on critical issues (B) indicate a specific ability that is useful in negotiation (C) encourage people to be more accepting of others (D) argue that few people are fit for the demands of negotiation (E) suggest that negotiators should always seek consensus

10. The public response described in lines 6-8 most strongly suggests that Menckens writings were (A) (B) (C) (D) (E) authoritative controversial arrogant informal frivolous

255
Questions 13-24 are based on the following passages. Passage 1 is from a 2003 book that examines the famous 1 Have a Dream speech delivered by Martin Luther King, Jr. at the historic March on Washington in August 1963. Passage 2 is from a 2000 biography of Martin Luther King, Jr. written by an African American scholar. Passage 1 The ability of the I Have a Dream speech to highlight Kings early career at the expense of his later career accounts for the tone of impatience and betrayal that often appears when modern-day supporters of Kings agenda talk 9325about the speech. Former Georgia state legislator Julian Bond said in 1986 that commemorations of King seemed to focus almost entirely on Martin Luther King the dreamer, not on Martin King the antiwar activist, not on Martin King the challenger of the economic order, not on Martin King 9330the opponent of apartheid, not on the complete Martin Luther King. One King scholar has proposed a ten-year moratorium on reading or listening to the I Have a Dream speech, in the hopes that America will then discover the rest of Kings legacy. 9335 This proposal effectively concedes that Kings magnificent address cannot be recovered from the misuse and overquotation it has suffered since his death. But it is not clear that this is so. Even now, upon hearing the speech, one is struck by the many forms of Kings genius. 9340Many people can still remember the first time they heard I Have a Dream, and they tend to speak of that memory with the reverence reserved for a religious experience. At the very least, reflecting on the I Have a Dream speech should be an opportunity to be grateful for the astonishing 9345transformation of America that the freedom movement wrought. In just under a decade, the civil rights movement brought down a system of segregation that stood essentially unaltered since Reconstruction. Kings dreams of an America free from racial discrimination are still some 9350distance away, but it is astounding how far the nation has come since that hot August day in 1963. Segregation in the South has been dismantled; there are no longer Whites Only signs; segregationist governors do not try to prevent Black children from entering public schools. 9355Toward the end of his life, King preached a sermon entitled Ingratitude, in which he called ingratitude one of the greatest of all sins, because the sinner fail[sl to realize his dependence on others. The annual Martin Luther King holiday is properly a day of national thanksgiving, a time 9360for the nation to recognize the immense debt it owes to King and the thousands of heroes of the civil rights movement for saving the soul of America. Passage 2 Martin Luther King was at his best when he was willing to reshape the wisdom of many of his intellec9365tual predecessors. He ingeniously harnessed their ideas to his views to advocate sweeping social change. He believed that his early views on race failed to challenge America fundamentally. He later confessed that he had underestimated how deeply entrenched racism was in 9370America. If Black Americans could not depend on goodwill to create social change, they had to provoke social change through bigger efforts at nonviolent direct action. This meant that Blacks and their allies had to obtain political power. They also had to try to restructure 9375American society, solving the riddles of poverty and economic inequality. This is not the image of King that is celebrated on Martin Luther King Day. Many of Kings admirers are uncomfortable with a focus on his mature beliefs. They 9380seek to deflect unfair attacks on Kings legacy by shrouding him in the cloth of superhuman heroism. In truth, this shroud is little more than romantic tissue. Kings image has often suffered a sad fate. His strengths have been needlessly exaggerated, his weaknesses wildly over9385played. Kings true legacy has been lost to cultural amnesia. As a nation, we have emphasized Kings aspiration to save America through inspiring words and sacrificial deeds. Time and again we replay the powerful image of King standing on a national stage 9390in the shadow of the Lincoln Memorial mouthing perhaps the most famous four words ever uttered by a Black American: I have a dream. For most Americans, those words capture King unique genius. They express his immortal longing for freedom, a longing that is familiar 9395to every person who dares imagine a future beyond unjust laws and unfair customs. The edifying universality of those four words-who hasnt dreamed, and who cannot identify with people whose dreams of a better world are punished with violence? helps to explain their durability. But those 9400words survive, too, because they comfort folk who would rather entertain the dreams of unfree people than confront their rage and despair.

13. The authors of both passages agree that Kings I Have a Dream speech (A) (B) (C) (D) (E) had significant global as well as national influence has been imitated by many of Kings followers had a profound impact on many Americans was typical of Kings thought as a whole questioned the ethical beliefs of many Americans

940514. It can be inferred that, for Julian Bond, a portrait of the complete Martin Luther King (lines 10-11) would

946519. The author of Passage 2 would most likely characterize the view of King expressed in lines 38-42 of Passage 1

9410

9415

(A) celebrate Kings influence both within and outside the United States (B) acknowledge the logical lapses in some of Kings later work (C) compare King with other significant figures of his era (D) achieve a balance between Kings earlier concerns and his later ones (E) reveal information about Kings personal as well as his public life 15. The author of Passage 2 would most likely view Julian Bonds statement in lines 7-11 of Passage 1 with (A) outright disapproval (B) considerable surprise (C) cynical mistrust (D) cautious optimism (E) complete agreement 16. In line 17, suffered most nearly means (A) (B) (C) (D) (E) endured felt prolonged tolerated lamented

9470

(The annual ... America) as (A) contradictory (B) insightful (C) atypical (D) simplistic (E) arrogant 20. Lines 57-58 (This is ... Da) mark a transition within Passage 2 from a (A) consideration of Kings views to a critique of peoples understanding of them (B) challenge to Kings beliefs to an acceptance of their cultural resonance (C) discussion of Kings intellectual predecessors to an analysis of his legacy (D) celebration of Kings strengths to an examination of his weaknesses (E) defense of Kings aspirations to an attack on those who fail to support them 21. Lines 76-79 in Passage 2 (The edifying ... durability) are best described as

9475

9420

9480

9425

9485

9430

9490

9435

17. Lines 31-34 ("Segregation in ... schools") serve primarily to (A) express ambitious hopes for the future (B) challenge the accuracy of historical accounts (C) provide a contrast with other cultures (D) illustrate a point with particular examples (E) defend a series of unusual occurrences 18. The author of Passage 1 mentions the sermon (line 35) primarily in order to
9495

9440

9500

(A) contesting the notion of Kings historical importance that is advanced by the author of Passage 1 (B) providing an explanation for the view of Kings speech that is expressed by the author of Passage 1 (C) challenging the portrait of the civil rights movement that is presented by the author of Passage 1 (D) offering a humorous anecdote that supports a statement made by the author of Passage 1 (E) dismissing a perspective that is similarly rejected by the author of Passage 1 22. Unlike the author of Passage 2, the author of Passage 1 develops his or her argument by

9445

(A) (B) (C) (D) (E)

show Kings effectiveness as a public speaker demonstrate the broad range of Kings interests illustrate an important trait that King possessed question Kings ability to empathize with others remind readers of a significant obligation to King

9505

(A) (B) (C) (D)

9450

9510

citing an authority with whom he or she disagrees referring to a famous speech delivered by King discussing the universal human trait of dreaming dismissing those who fail to understand the subtlety of Kings thought (E) assuming that his or her readers are completely unfamiliar with Kings ideas

9455

9460

9515

23. The author of Passage 2 would most likely argue that commemorations focus on Martin Luther King the dreamer (line 7 of Passage 1) because people find this aspect of King to be (A) courageous (B) unpretentious (C) reassuring (D) provocative (E) unexpected

24. Which best characterizes the overall relationship between the two passages?
9580

9520

9585

9525 9590 9530 9595 9535 9600 9540

(A) Passage 2 rejects the political goals that are described in Passage 1. (B) Passage 2 helps account for the responses to a speech discussed in Passage 1. (C) Passage 2 romanticizes a person who is objectively depicted in Passage 1. (D) Passage 2 recounts the history of a national holiday that is celebrated in Passage 1. (E) Passage 2 reflects on a figure who is denounced in Passage I.

9545

9550

9555

9560

9565

9570

9575

260

SECTION 4
9605

CRITICAL READING
24 Questions

Time 25 minutes

Directions: For each question in this section, select the best answer from among the choices given and fill in the corresponding circle on the answer sheet.

9610 9640

Each sentence below has one or two blanks, each blank indicating that something has been omitted. Beneath the sentence are five words or sets of words labeled A through E. Choose the word or set of words that, when inserted in the sentence, best fits the meaning of the sentence as a whole. Example: Hoping to ------- the dispute, negotiators proposed a compromise that they felt would be ------- to both labor and management. (A) (B) (C) (D) (E) enforce .. useful end .. divisive overcome .. unattractive extend .. satisfactory resolve .. acceptable

3. Night jars possess a camouflage perhaps unparalleled in the bird world: by day they roost hidden in shady woods, so ------- with their surroundings that they are 9645 nearly impossible to -------. (A) vexed .. dislodge (B) blended .. discern (C) harmonized .. interrupt (D) impatient .. distinguish (E) integrated .. classify 4. Many economists believe that since resources are scarce and since human desires cannot all be -------, a method of ------- is needed.
9655

9650

(A) (B) (C) (D) (E)

indulged .. apportionment verified .. distribution usurped .. expropriation expressed .. reparation anticipated .. advertising

96151. Many private universities depend heavily on -------, the wealthy individuals who support them with gifts and bequests.

9660

9620

(A) instructors (B) administrators (C) monitors (D) accountants (E) benefactors 2. One of the characters in Milton Murayamas novel is considered ------- because he deliberately defies an oppressive hierarchical society.

5. The range of colors that homeowners could use on the exterior of their houses was ------- by the communitys stringent rules regarding upkeep of property. (A) (C) (E) (B) (D) circumscribed embellished cultivated bolstered insinuated

9665

9625

(A) rebellious (B) impulsive (C) artistic (D) industrious (E) tyrannical

9670

9630

9635

The passages below are followed by questions based on their content; questions following a pair of related passages may also be based on the relationship between the paired passages. Answer the questions on the basis of what is stated or implied in the passages and in any introductory material that may be provided.

9675

Questions 6-9 are based on the following passages. Passage 1 I know what your e-mail in-box looks like, and it isnt pretty: a babble of come-ons and lies from hucksters and con artists. To find your real e-mail, you must wade through the torrent of fraud and obscenity known politely 9680as unsolicited bulk e-mail and colloquially as spam. In a perverse tribute to the power of the online revolution, we are all suddenly getting the same mail: easy weight loss, get-rich-quick schemes, etc. The crush of these messages is now numbered in billions per day. Its becoming 9685a major systems and engineering and network problem, says one e-mail expert. Spammers are gaining control of the Internet. Passage 2 Many people who hate spam assume that it is protected as free speech. Not necessarily so. The United States 9690Supreme Court has previously ruled that individuals may preserve a threshold of privacy. Nothing in the Constitution compels us to listen to or view any unwanted communication, whatever its merit, wrote Chief Justice Warren Burger in a 1970 decision. We therefore categori9695cally reject the argument that a vendor has a right to send unwanted material into the home of another. With regard to a seemingly similar problem, the Telephone Consumer Protection Act of 1991 made it illegal in the United States to send unsolicited faxes; why not extend the act to include 9700unsolicited bulk e-mail?

6. The primary purpose of Passage1 is to (A) (B) (C) (D) (E) make a comparison dispute a hypothesis settle a controversy justify a distinction highlight a concern

7. The primary purpose of Passage 2 is to (A) (B) (C) (D) (E) confirm a widely held belief discuss the inadequacies of a ruling defend a controversial technology analyze a widespread social problem lay the foundation for a course of action

8. What would be the most likely reaction by the author of Passage 1 to the argument cited in lines 16-21 of Passage 2 (Nothing ... another) ? (A) Surprise at the assumption that freedom of speech is indispensable to democracy (B) Dismay at the Supreme Courts vigorous defense of vendors rights (C) Hope that the same reasoning would be applied to all unsolicited e-mail (D) Concern for the plight of mass marketers facing substantial economic losses (E) Appreciation for the political complexity of the debate about spam 9. Unlike the author of Passage 1, the author of Passage 2 (A) (B) (C) (D) (E) criticizes a practice offers an example proposes a solution states an opinion quotes an expert

265
Questions 10-16 are based on the following passage. The following passage is adapted from a novel set in the early twentieth century. Mr. Beebe, a clergyman, is speaking with Cecil Vyse about a mutual acquaintance, Lucy Honeychurch. Miss Honeychurch has recently returned from a journey with her older cousin and chaperone, Miss Bartlett. Lucy Honeychurch has no faults said Cecil, with grave sincerity. I quite agree. At present she has none. At present? 9705 Im not cynical. Im only thinking of my pet theory about Miss Honeychurch. Does it seem reasonable that she should play piano so wonderfully, and live so quietly? I suspect that someday she shall be wonderful in both. The water-tight compartments in her will break down, 9710and music and life will mingle. Then we shall have her heroically good, heroically bad-too heroic, perhaps, to be good or bad. Cecil found his companion interesting. And at present you think her not wonderful as far 9715as life goes? Well, I must say Ive only seen her at Tunbridge Wells, where she was not wonderful, and at Florence. She wasnt wonderful in Florence either, but I kept on expecting that she would be. 9720 In what way? Conversation had become agreeable to them, and they were pacing up and down the terrace. I could as easily tell you what tune shell play next. There was simply the sense that she found wings and 9725meant to use them. I can show you a beautiful picture in my diary. Miss Honeychurch as a kite, Miss Bartlett holding the string. Picture number two: the string breaks. The sketch was in his diary, but it had been made afterwards, when he viewed things artistically. At the time he 9730had given surreptitious tugs to the string himself. But the string never broke? No. I mightnt have seen Miss Honeychurch rise, but I should certainly have heard Miss Bartlett fall. It has broken now, said the young man in low, 9735vibrating tones. Immediately he realized that of all the conceited, ludicrous, contemptible ways of announcing an engagement this was the worst. He cursed his love of metaphor; had he suggested that he was a star and that Lucy was 9740soaring up to reach him? Broken? What do you mean? I meant, Cecil said stift1y, that she is going to marry me. The clergyman was conscious of some bitter 9745disappointment which he could not keep out of his voice. I am sorry; I must apologize. I had no idea you were intimate with her, or I should never have talked in this flippant, superficial way. You ought to have 9750stopped me. And down in the garden he saw Lucy herself; yes, he was disappointed. Cecil, who naturally preferred congratulations to apologies, drew down the corner of his mouth. Was this the reaction his action would get from the whole 9755world? Of course, he despised the world as a whole; every thoughtful man should; it is almost a test of refinement. Im sorry I have given you a shock, he said dryly. I fear that Lucys choice does not meet with 9760your approval. 10. Cecils remark in line 1 (Lucy ... faults) is made in a tone of (A) (B) (C) (D) (E) great conviction studied neutrality playful irony genuine surprise weary cynicism

11. Mr. Beebe asks the question in lines 6-7 (Does ... quietly) primarily in order to (A) (B) (C) (D) (E) raise an urgent concern anticipate a possible objection challenge a widely accepted theory note an apparent inconsistency criticize a popular pastime

12. Mr. Beebes statement, The water-tight ... bad (lines 9-11), suggests that Lucy will (A) ultimately become a famous and respected musician (B) eventually play music in a less disciplined fashion (C) one day begin to live with great passion (D) soon regret an impetuous decision (E) someday marry a man who will be the cause of her undoing

13. In line 24, sense most nearly means


9765

(A) (B) (C) (D) (E)

definition intelligence plausibility consensus impression

9825

15. Ultimately, Cecil views his remark in line 34 (It ... now) as (A) (B) (C) (D) (E) singularly poetic particularly memorable embarrassingly inapt excessively critical regrettably underhanded

9830 977014. For Mr. Beebe, Picture number two (line 27) represents

9775

(A) (B) (C) (D) (E)

a misleading occurrence a dangerous gamble an unlikely development an anticipated outcome an avoidable difficulty

16. The question in lines 39-40 (had ... him) suggests that Cecil fears that Mr. Beebe will
9835

(A) (B) (C) (D) (E)

detect the lack of originality in his thinking consider him to be vain tell Lucy of his inappropriate remark distrust him as a confidant attempt to block his engagement to Lucy

9840 9780

9785

9790

9795

9800

9805

9810

9815

9820

Questions 17-24 are based on the following passage. The following passage is adapted from a book published in 1999. Calling it a cover-up would be far too dramatic. But for more than half a century-even in the midst of some of the greatest scientific achievements in history-physicists 9845have been quietly aware of a dark cloud looming on a distant horizon. The problem is this: There are two foundational pillars upon which modern physics rests. One is general relativity, which provides a theoretical framework for understanding the universe on the largest 9850of scales: stars, galaxies, clusters of galaxies, and beyond to the immense expanse of the universe itself. The other is quantum mechanics, which provides a theoretical framework for understanding the universe on the smallest of scales: molecules, atoms, and all the way down to 9855subatomic particles like electrons and quarks. Through years of research, physicists have experimentally confirmed to almost unimaginable accuracy virtually all predictions made by each of these theories. But these same theoretical tools inexorably lead to another disturbing conclusion: 9860As they are currently formulated, general relativity and quantum mechanics cannot both be right. The two theories underlying the tremendous progress of physics during the last hundred years-progress that has explained the expansion of the heavens and the fundamental structure 9865of matter are mutually incompatible. If you have not heard previously about this ferocious antagonism, you may be wondering why. The answer is not hard to come by. In all but the most extreme situations, physicists study things that are either small and light (like 9870atoms and their constituents) or things that are huge and heavy (like stars and galaxies), but not both. This means that they need use only quantum mechanics or only general relativity and can, with a furtive glance, shrug off the barking admonition of the other. For 50 years this approach 9875has not been quite as blissful as ignorance, but it has been pretty close. But the universe can be extreme. In the central depths of a black hole, an enormous mass is crushed to a minuscule size. According to the big bang theory, the whole of the 9880universe erupted from a microscopic nugget whose size makes a grain of sand look colossal. These are realms that are tiny and yet incredibly massive, therefore requiring that both quantum mechanics and general relativity simultaneously be brought to bear. The equations of general 9885relativity and quantum mechanics, when combined, begin to shake, rattle, and gush with steam like a decrepit automobile. Put less figuratively, well-posed physical questions elicit nonsensical answers from the unhappy amalgam of

these two theories. Even if you are willing to keep the 9890deep interior of a black hole and the beginning of the universe shrouded in mystery, you cant help feeling that the hostility between quantum mechanics and general relativity cries out for a deeper level of understanding. Can it really be that the universe at its most fundamental 9895level is divided, requiring one set of laws when things are large and a different, incompatible set when things are small? Superstring theory, a young upstart compared with the venerable edifices of quantum mechanics and general 9900relativity, answers with a resounding no. Intense research over the past decade by physicists and mathematicians around the world has revealed that this new approach to describing matter at its most fundamental level resolves the tension between general relativity and quantum 9905mechanics. In fact, superstring theory shows more: within this new framework, general relativity and quantum mechanics require one another for the theory to make sense. According to superstring theory, the marriage of the laws of the large and the small is not 9910only happy but inevitable. Superstring theory has the potential to show that all of the wondrous happenings in the universe-from the frantic dance of subatomic quarks to the stately waltz of orbiting binary stars-are reflections of one grand physical principle, one master 9915equation. 17. The dark cloud mentioned in line 4 refers to an (A) atypical diagnosis (B) unsupported hypothesis (C) unknown threat (D) evil influence (E) important contradiction 18. Which pairing best represents the different models of the universe presented in lines 7-14 ? (A) (B) (C) (D) (E) Big and little Old and new Complex and simple Verified and undocumented Theoretical and practical

19. The authors use of italics in line 20 serves primarily to (A) (B) (C) (D) (E) draw attention to a commonly known hypothesis stress a speculative aspect of two theories support a difficult claim underscore a surprising point emphasize an area of agreement

270

20. The author uses the automobile (lines 45-46) to represent equations that
9920

9980

(A) (B) (C) (D) (E)

demand a professionals attention are intrinsically unreliable do not work together effectively can be easily adjusted if necessary are based on dated mathematics

23. Those who hold the conclusion referred to in line 18 would most likely believe that the marriage (line 68) was an (A) (B) (C) (D) (E) inevitable result of their research unjustifiable elevation of their hypotheses inadvisable use of research funds unfortunate consequence impossible outcome

9985

9925

21. Which of the following, if available, would best refute the authors assertion about the young upstart (line 57) ? (A) Evidence that certain kinds of particles in nature exceed the speed of light (B) Confirmation of conditions that existed in the earliest stages of the big bang (C) Speculation that the deep interior of a black hole is not as dense as scientists have believed (D) Mathematical formulas that link general relativity and quantum mechanics in the same realm (E) Proof that the laws governing the universe depend on the size of the system being studied
9990

24. The author uses dance imagery in lines 71-72 in order to (A) suggest a similarity between the study of science and the study of dance (B) highlight the extremes found in the physical world (C) emphasize the different ways that binary stars move (D) illustrate the intricacy of the subatomic world of quarks (E) suggest the cohesive nature of both science and dance

9930

9995

9935

994022. The primary reason described for the usefulness of the theory mentioned in line 57 is its ability to

10000

9945

(A) (B) (C) (D) (E)

explain new phenomena replace the theory of general relativity reinforce the predictions of quantum mechanics indicate where other theories are inapplicable reconcile two seemingly contradictory theories

9950

9955

9960

9965

9970

9975

SECTION 6

WRITING

Time - 25 minutes 35 Questions


10005 Directions: For each question in this section, select the best answer from among the choices given and fill in the corresponding circle on the answer sheet.

The following sentences test correctness and effectiveness of expression. Part of each sentence or the entire sentence is underlined; beneath each sentence are five ways of phrasing the underlined material. Choice A repeats the original phrasing; the other four choices are different. If you think the original phrasing produces a better sentence than any of the alternatives, select choice A; if not, select one of the other choices. In making your selection, follow the requirements of standard written English; that is, pay attention to grammar, choice of words, sentence construction, and punctuation. Your selection should result in the most effective sentence clear and precise, without awkwardness or ambiguity. EXAMPLE: Laura Ingalls Wilder published her first book and she was sixty-five years old then. (A) and she was sixty-five years old then (B) when she was sixty-five (C) at age sixty-five years old (D) upon the reaching of sixty-five years (E) at the time when she was sixty-five

10030

2. To help freshmen and sophomores in selecting their courses, candid reviews of courses and instructors compiled by juniors and seniors. (A) candid reviews of courses and instructors compiled by juniors and seniors (B) candid reviews of courses and instructors being compiled by juniors and seniors (C) and to compile candid reviews of courses and instructors by juniors and seniors (D) juniors and seniors have compiled candid reviews of courses and instructors (E) with juniors and seniors compiling candid reviews of courses and instructors

10035

10040

3. The landscape artist who designed New York Citys Central Park believed that providing scenic settings 10045 accessible to all would not only benefit the publics physical and mental health and also foster a sense of democracy. (A) (B) (C) (D) (E) and also foster a sense of democracy as it also fosters a sense of democracy and would foster a sense of democracy also but also foster a sense of democracy and foster a sense of democracy also

10050

10055

10010
1. Since last September Patricia has been working at the convenience store down the road. (A) (B) (C) (D) (E) has been working works is working will be working worked
10060

4. In areas where deer roam freely, residents must dress to protect themselves against deer ticks that might transmit diseases. (A) (B) (C) (D) (E) areas where deer roam freely areas roamed by deer freely areas, freely roamed by deer areas, in which there are deer that roam freely areas which deer roam free

10015

10020

10025

275
10065

5. Given the cost of a hardcover book, the price of it typically hovers around $25, many consumers ask their book dealers, "When will the paperback be out?" (A) (B) (C) (D) (E) the price of it typically hovers and typically it hovers at a price which typically hovers in that it typically hovers they typically hover

10125

9. At the Constitutional Convention of 1787, the proposal to replace the existing Articles of Confederation with a federal constitution were met with fierce opposition. (A) (B) (C) (D) (E) were met with having been met with it met met with met their

10070

10130

100756. The article featured the Sea Islands because many were known there to live much as their ancestors of a century ago had lived.

10080

(A) (B) (C) (D) (E)

many were known there to live they were known there for living many of the people there were known to live of the many people, they were there living of knowing that many people lived there

10135 When for the first time the United States imported 10. more oil than it exported, Americans should have realized that an energy crisis was imminent and could happen in the future. 10140

(A) (B) (C) (D) (E)

was imminent and could happen in the future could happen imminently in the future will be imminent and happening soon is an imminent thing might be imminent

10085

7. A poetic form congenial to Robert Browning was the dramatic monologue, it let him explore a characters mind without the simplifications demanded by stage productions. (A) (B) (C) (D) (E) monologue, it let him explore monologue, which let him explore monologue that lets him explore monologue; letting him explore monologue by letting him do exploration of

10145 Intimacy, love, and marriage are three different, if 11. interrelated, subjects.

10090

10150

(A) (B) (C) (D) (E)

different, if interrelated, subjects interrelated subjects, being, however, different different subjects, whereas they are interrelated different subjects when interrelated subjects that are different although being interrelated

10095

8. Many eighteenth- and nineteenth-century Romantic poets were believers in rebellion against social conventions, express strong emotion, and the power of imagination. (A) were believers in rebellion against soci~l conventions, express strong emotion (B) are believers in rebelling against social conventions, strong emotions being expressed (C) who believed in rebellion against social conventions, express strong emotion (D) believed in rebellion against social conventions, to express strong emotions (E) believed in rebellion against social conventions, the expression of strong emotions

10155

10100

10105

10110

10115

10120

10190

15. Storing bread in the refrigerator delays drying and the The following sentences test your ability to recognize grammar and usage errors. Each sentence contains either a single error or no error at all. No sentence contains more than one error. The error, if there is one, is underlined and lettered. If the sentence contains an error, select the one underlined part that must be changed to make the sentence correct. If the sentence is correct, select choice E. In choosing answers, follow the requirements of standard written English. EXAMPLE: The other delegates and him immediately A B C accepted the resolution drafted by the D neutral states. No error E A growth of mold but increase the rate at which the
10195

B bread loses flavor. No error D E

16. According to last weeks survey, most voters


10200

were disappointed by legislators inability working A D B E C together on key issues. No error

1020517. When Marie Curie shared the 1903 Nobel Prize for

A Physics with two other scientists her husband


1016012. Americas first roller coaster ride, which opened in

B Pierre Curie and Henri Becquerel she had been


10210

A 1884 at Coney Island, Brooklyn, and capable of B a top speed of only six miles per hour. No error
10165

C the first woman to win the prize. No error D E

E
10215

18. Question 18 did not count toward your score. 19. Those investors who sold stocks just before the A B C
10220

13. The inflation rate in that country is so high that A even with adjusted wages, most workers can barely
10170

B E

stock market crashed in 1929 were either wise or exceptional lucky. No error D E

pay for food and shelter. No error

14. Over the past two years, apparel manufacturers have


10175

A worked to meeting the revised federal standards C for the design of uniforms. No error D E

10180

10185

10225

20. Most of the sediment and nutrients of the


10275

25. In order for the audience to believe in and A be engaged by a Shakespearean character, B they have to come across as a real person C
10280

Mississippi River no longer reach the coastal


10230

A wetlands, a phenomenon that has adversely B affected the regions ecological balance. C D

D E

on the stage. No error

10235

No error E 21. Most major air pollutants cannot be seen, although


10285

26. Most of the hypotheses that Kepler developed A B C to explain physical forces were later rejected as inconsistent to Newtonian theory. No error D E

10240

large amounts of them concentrated in cities A are visible as smog. No error C D E B

1029027. Lynn Marguliss theory that evolution is a process

A
1024522. The light emitted by high-intensity-discharge

involving interdependency rather than competition B among organisms differs dramatically from
10295

car headlights are very effective in activating A


10250

B C

C most biologists. No error D E

the reflective paints of road markers, thereby making driving at night safer. No error D E
10300

28. The Empire State Building, the Sears Tower, the Canadian National Tower each of these structures A was the tallest in the world at the time they were B
10305

23. During the nineteenth century, Greek mythology


10255

A acquired renewed significance when both poets and B painters turned to the ancient myths for subject C D C E

built. No error

10260

matter. No error E 24. The museum is submitting proposals to several A B D E D


10310

29. The cost of safely disposing of the toxic chemicals A is approximately five times what the company paid B to purchase it. No error E C

10265

foundations in the hope to gain funds to build C a tropical butterfly conservatory. No error

10270

280

10365 10315 Directions: The following passage is an early draft of an essay. Some parts of the passage need to be rewritten.

30. Which is the best version of the underlined part of sentence 2 (reproduced below) ? Acclaimed as an artist in the United States and Europe 10370at the turn of the century, Tanner was called the dean of art by W. E. B. Du Bois. (A) (as it is now) (B) century; Tanner was called the dean of art by W. E. B. Du Bois (C) century, Tanner, who was called dean of art by W. E. B. Du Bois (D) century, W. E. B. Du Bois calling Tanner the dean of art (E) century, it was W. E. B. Du Bois who called Tanner the dean of art 31. Which is the best version of the underlined portion of sentence 4 (reproduced below) ?
10385

Read the passage and select the best answers for the questions that follow. Some questions are about particular sentences or parts of sentences and ask you to improve sentence structure or word choice. Other questions ask you to consider organization and development. In choosing answers, follow the requirements of standard written English.

10375

Questions 30-35 refer to the following passage. (1) On September 10, 1973, the United States Postal Service issued a stamp honoring Henry Ossawa Tanner (1859-1937), one of four stamps in the American Arts series. (2) Acclaimed as an artist in the United States and Europe at the turn of the century, Tanner was called the 10325dean of art by W. E. B. Du Bois. (3) But after his death, Tanners work was largely forgotten. (4) And so it remained, and even later, in 1969, the donation of one of his paintings to the Smithsonian Institution aroused new interest in the art of this American master. (5) Now his 10330works are on exhibit again. (6) You can even buy posters of his paintings! (7) One of his most famous works is a realistic painting by the name of The Banjo Lesson. (8) It was inspired by a poem of Paul Laurence Dunbar. (9) The painting isnt 10335like a photograph. (10) The magnificence of his work can be seen with each subtle brush stroke, each carefully crafted detail. (11) The effect is truly beautiful. (12) If I were to try to identify the dominant theme of the painting, I would have to say that it is family cohesiveness because the 10340entire scene seems to emphasize the bond between the boy and his grandfather.
10320 10380

And so it remained. and even later. in 1969, the donation of one of his paintings to the Smithsonian Institution aroused new interest in the art of this American master. (A) (B) (C) (D) (E) (as it is now) remained, and even after that, in 1969, remained, but even then, in 1969, remained until 1969, when remained when in 1969

10390

10395

32. In context, which is the best revision of sentence 6 (reproduced below) ? You can even buy posters of his paintings! (A) It is amazing, you can buy posters of his paintings. (B) Even ordinary people like us can buy posters of his paintings. (C) Posters of his paintings had been sold. (D) People can even buy his paintings as a poster. (E) One can even buy posters of his paintings.

10400

10345 10405

10350

10355

10360

33. In context, which is the best way to revise sentence 7 (reproduced below) ?
10410

10470

35. Which is best to add to the beginning of sentence 9 ? (A) (B) (C) (D) (E) Although it is realistic, You can almost hear the music, but Photographs have a beauty of their own, but As a lifelike work, Some people just copy what they see;

One of his most famous works is a realistic painting by the name of The Banjo Lesson. (A) Add In contrast, to the beginning of the sentence. (B) Change a realistic painting by the name of to the realistic painting. (C) Delete the words of his most famous works. (D) Change is to had been. (E) Delete most famous.
10475

10415

10480

1042034. Which sentence is best inserted after sentence 7 ?

10425

(A) The painting shows a man teaching his grandson how to play the banjo. (B) He finished The Banjo Lesson in 1893. (C) In the painting, a bright light sets off the man and boy. (D) Banjos came to the United States from West Africa. (E) Portraits by Tanner show a psychological depth and compassion.

10430

10435

10440

10445

10450

10455

10460

10465

285 SECTION 8 CRITICAL READING


19 Questions
10485 Directions: For each question in this section, select the best answer from among the choices given and fill in the corresponding circle on the answer sheet.

Time 20 minutes

Each sentence below has one or two blanks, each blank indicating that something has been omitted. Beneath the sentence are five words or sets of words labeled A through E. Choose the word or set of words that, when inserted in the sentence, best fits the meaning of the sentence as a whole. Example: Hoping to ------- the dispute, negotiators proposed a compromise that they felt would be ------- to both labor and management. (A) (B) (C) (D) (E)
10490

105203. Geysers vary widely: some may discharge -------, whereas others may have only a brief explosive eruption and then remain ------- for hours or days.

10525

(A) (B) (C) (D) (E)

violently .. dangerous continuously .. quiescent spontaneously .. unpredictable regularly .. active faintly .. imperceptible

10530

enforce .. useful end .. divisive overcome .. unattractive extend .. satisfactory resolve .. acceptable

4. Although the administration repeatedly threatened to use its authority in order to ------- the student protestors into submission, they refused to be intimidated. (A) ease (D) bully (B) delude (E) nudge (C) cajole

1. The writer came to be labeled ------- because she isolated herself in her apartment, shunning outside contact.
10495(A) a loner (B) a miser (C) a connoisseur (D) a conspirator (E) an ingnue

105355. Only after the campaign volunteers became aware of their candidates questionable motives could they recognize the ------- statements made in his seemingly ------- speeches. 10540

(A) (B) (C) (D) (E)

insightful .. astute partisan .. callous cordial .. hostile duplicitous .. candid cunning .. surreptitious

2. Some Tibetan nomads used yak butter as a -------, one that often took the place of money in commercial 10500 transactions. (A) promotion (B) commodity (C) formula (D) refund (E) register

105456. No longer narrowly preoccupied with their own national pasts, historians are increasingly ------- in that they often take a transnational perspective.

(A) conciliatory (B) bombastic (D) cosmopolitan (E) jocular


10550

(C) mendacious

10505

10510

10515

The passage below is followed by questions based on its content. Answer the questions on the basis of what is stated or implied in the passage and in any introductory material that may be provided.

Questions 7-19 are based on tbe following passage. In the introduction to one of her dramas, a well-known playwright and actor discusses some of her ideas about acting. Words have always held a particular power for me. I remember leafing through a book of Native American poems one morning while I was waiting for my Shakespeare class to begin and being struck by a phrase from the preface, "The word, the word above all, is truly magical, not only by 10560its meaning, but by its artful manipulation." This quote, which I added to my journal, reminded me of something my grandfather had told me when I was a girl: "If you say a word often enough it becomes your own." I added that phrase to my journal next to the quote 10565about the magic of words. When I traveled home to Baltimore for my grandfathers funeral a year after my journal entry, I mentioned my grandfathers words to my father. He corrected me. He told me that my grandfather had actually said, "If you say a word often enough, it 10570becomes you." I was still a student at the time, but I knew even then, even before I had made a conscious decision to teach as well as act, that my grandfathers words would be important. Actors are very impressionable people, or some would 10575say, suggestible people. We are trained to develop aspects of our memories that are more emotional and sensory than intellectual. The general public often wonders how actors remember their lines. Whats more remarkable to me is how actors remember, recall, and reiterate feelings and 10580sensations. The body has a memory just as the mind does. The heart has a memory, just as the mind does. The act of speech is a physical act. It is powerful enough that it can create, with the rest of the body, a kind of cooperative dance. That dance is a sketch of something that is inside a 10585person, and not fully revealed by the words alone. I came to realize that if I were able to record part of the dance that is, the spoken part-and reenact it, the rest of the body would follow. I could then create the illusion of being another person by reenacting something she had said as she 10590had said it. My grandfathers idea led me to consider that the reenactment, or the reiteration, of a persons words would also teach me about that person. I had been trained in the tradition of acting called "psychological realism." A basic tenet of psychological 10595realism is that characters live inside of you and that you create a lifelike portrayal of the character through a process of realizing your own similarity to the character. When I
10555

later became a teacher of acting, I began to become more and more troubled by the self-oriented method. I began to 10600look for ways to engage my students in putting themselves in other peoples shoes. This went against the grain of the psychological realism tradition, which was to get the character to walk in the actors shoes. It became less and less interesting intellectually to bring the dramatic literature of 10605the world into a classroom of people in their late teens and twenties, and to explore it within the framework of their real lives. Aesthetically it seemed limited, because most of the time the characters all sounded the same. Most characters spoke somewhere inside the rhythmic range of the 10610students. More troubling was that this method left an important bridge out of acting. The spirit of acting is the travel from the self to the other. This "self-based" method seemed to come to a spiritual halt. It saw the self as the ultimate home of the character. To me, the search for char10615acter is constantly in motion. It is a quest that moves back and forth between the self and the other. I needed evidence that you could find a characters psychological reality by "inhabiting" that characters words. I needed evidence of the limitations of basing a character on 10620a series of metaphors from an actors real life. I wanted to develop an alternative to the self-based technique, a technique that would begin with the other and come to the self, a technique that would empower the other to find the actor rather than the other way around. 7. The primary purpose of the first three paragraphs (lines 1-38) is to (A) (B) (C) (D) describe the actors process of developing a role trace the beginnings of a personal philosophy analyze the grandfathers insights into acting investigate the effect of words on interpersonal relationships (E) explore a viewpoint that the author is forced to reverse 8. The author of the passage uses the quotation in lines 5-6 primarily as a (A) vivid expression of how she views words (B) powerful example of what she sought in Shakespeare (C) scholarly citation linking her to poetic words (D) comical introduction to a problem encountered by every dramatic performer (E) pragmatic assessment of the power of words for beginning drama students

10625

9. By presenting both versions of the grandfathers words 1068515. The authors explanation in the fourth paragraph suggests (lines 9-10 and lines 15-16), the author primarily conveys the that the "self-oriented method" (line 45) rests on the assumption that (A) grandfathers attempts to play with language (B) grandfathers enthusiasm in spite of her reaction (A) audience members appreciate complex nuances of 10630 (C) fathers intervention in a private moment character (D) ambivalence she feels toward her grandfather 10690(B) the playwrights biography provides the main evidence (E) significance of the grandfathers message for interpreting character (C) actors have already felt the full range of human 10. The comparisons in lines 26-27 serve primarily to emotions (D) actors are extremely independent and self-serving 10635 (A) show the similarities that exist between dancing 10695people and acting (E) actors lives become fulfilled through their dramatic (B) celebrate the broad range of memories that actors portrayals learn to draw on (C) justify the authors adherence to conventional 16. Which statement best captures the authors point in 10640 acting theory 10700lines 54-56 ("Most characters ... students") ? (D) explain why actors have difficulty interpreting character (A) The characters spoke through the students own (E) enhance the authors credibility as a technically rich cadences. trained actor (B) Young drama students have an uncanny knack for conveying character. 10645 11. In lines 29-34 (a kind ... follow), the author uses the 10705(C) Most students found class to be repetitious. idea of a dance to (D) Characterizations were confined by what the students knew. (A) supply an image for the awkwardness some actors (E) The spontaneity that the students had hoped for experience had not been achieved. 10650 (B) illustrate a process that words can set in motion 10710 (C) portray the enactment of a character as an exhilarating 17. In line 60, the phrase "home of the character" most experience nearly means (D) argue that acting requires physical agility (E) show how a word can evoke multiple meanings (A) way of understanding eccentricities (B) social context surrounding a character 10655 12. In line 34, follow most nearly means 10715(C) environment for practicing acting (D) forum in which the self is presented publicly (A) pursue (E) source of a roles psychological truth (B) result (C) surpass 18. In lines 63-64, "psychological reality" describes which 10660 (D) join in 10720quality? (E) listen carefully (A) The versatility of a performer 13. In lines 39-62, the author reveals herself to be someone (B) The physical gestures of a character who believes that (C) The essence of an identity (D) The accuracy of an audiences expectations 10665 (A) teachers and students should examine controversial 10725(E) The logical consistency of certain actions issues together (B) playwrights especially benefit from experience on stage 19. The metaphors in line 66 are best described as (C) conventional approaches should be open to questioning and reevaluation (A) private misgivings 10670 (D) traditional methods often reflect the accumulated (B) objective observations insight of generations 10730(C) abstract equations (E) standard practices are the most suitable to teach to (D) memorable phrases beginners (E) personal comparisons
10675 Lines 39-70 present the authors argument primarily by 14.

10680

(A) celebrating the appeal of a discredited tradition (B) exploring the impact of her early experiences on her acting (C) explaining her reasons for rejecting a technique (D) describing challenges commonly met by professional actors . (E) analyzing insights gained from debates with other drama professors

290

10735 SECTION 10 WRITING

Time 10 minutes 14 Questions


Directions: For each question in this section, select the best answer from among the choices given and fill in the corresponding circle on the answer sheet.

10740

The following sentences test correctness and effectiveness of expression. Part of each sentence or the entire sentence is underlined; beneath each sentence are five ways of phrasing the underlined material. Choice A repeats the original phrasing; the other four choices are different. If you think the original phrasing produces a better sentence than any of the alternatives, select choice A; if not, select one of the other choices. In making your selection, follow the requirements of standard written English; that is, pay attention to grammar, choice of words, sentence construction, and punctuation. Your selection should result in the most effective sentence clear and precise, without awkwardness or ambiguity. EXAMPLE: Laura Ingalls Wilder published her first book and she was sixty-five years old then. (A) and she was sixty-five years old then (B) when she was sixty-five (C) at age sixty-five years old (D) upon the reaching of sixty-five years (E) at the time when she was sixty-five
10745

10765

2. Since scientific advances are central to progress, basic research deserving continuing support. (A) basic research deserving continuing support (B) basic research being what deserves continuing support (C) basic research deserves continuing support (D) continuing support is deserved by basic research (E) continuing support is what they deserve in basic research 3. With Americans consuming sugar in record amounts, nutritionists are urging the public to reduce its consumption of sodas, which have largely replaced other, more healthful, beverages. (A) nutritionists are urging the public to reduce its consumption of sodas. which (B) nutritionists have been urging that the public reduces its consumption of sodas; those (C) the public ought to reduce its consuming of sodas, as urged by nutritionists. because they (D) nutritionists urge about reducing public soda consumption, which (E) less soda should be consumed by the public, urge nutritionists. which 4. Experts disagree about what is the definition of intelligence and how to measure it. (A) what is the definition of intelligence and how to measure it (B) how to define intelligence. and also its measurement (C) how to define and measure intelligence (D) defining intelligence as well as measurement (E) the definition of intelligence and measuring it

10770

10775

10780

10785

10790

1. In everything from finding comets to spotting supernovae, amateur astronomers have become so accomplished, and professional astronomers sometimes 10750 seek their help. (A) (B) (C) (D) (E) accomplished, and accomplished, also accomplished that accomplished therefore accomplished when

10795

10800

10755

10760

10860

5. The charges against the organization are being investigated by a committee. it includes several 10805 senators. (A) (B) (C) (D) (E) committee, it includes committee; it including committee, and it will include committee, they include committee that includes

10. James Barrie, the author of Peter Pan and other plays, is noted for portraying adulthood 10865 as unpleasant and childhood is glorified. (A) (B) (C) (D) (E) childhood is glorified childhood as being glorious childhood as glorious childhood glorified glorified childhood

10810

10870

6. Travel writing often describes a journey of exploration and endurance, a trip that is risky either because of natural hazards but also because of political unrest.
10815

(A) (B) (C) (D) (E)

but also because of but also due to or because there was or because of or the cause is

11. Medical insurance coverage that requires high monthly premiums and that is beyond the financial means of many people.
10875

(A) (B) (C) (D)

10820

7. Though they had earlier indicated otherwise, it was eventually decided upon by the legislators to have the bill passed. (A) it was eventually decided upon by the legislators to have the bill passed (B) it was eventually decided upon by the legislators to pass the bill (C) the eventual decision of the legislators was for passage of the bill (D) the legislators eventual decision was passing the bill (E) the legislators eventually decided to pass the bill 8. Spread by rat fleas, millions of people in medieval Europe were killed by bubonic plague. (A) millions of people in medieval Europe were killed by bubonic plague (B) and millions of medieval Europeans killed by bubonic plague (C) this led to the killing of millions of medieval Europeans by bubonic plague (D) bubonic plague in medieval Europe was why millions of people were killed (E) bubonic plague killed millions of people in medieval Europe 9. Traditional Jamaican music. enriched with rock, jazz, and other modern rhythms from America, were the basis for reggae.

10880

that requires high monthly premiums and that is that requires high monthly premiums and it is requiring high monthly premiums are with the requirements of high monthly premiums are (E) that requires high monthly premiums is 12. Among the most flavorful cuisines in the United States, New Orleans has also become one of the most popular.

10825

10885

(A) (B) (C) (D) (E)

New Orleans has also become New Orleans has also become famous as the cuisine of New Orleans is also cuisines in New Orleans also have become also the cuisine of New Orleans is

10830

10890

13. Meals prepared by the Algonquin Indians, who were farmers as well as hunters, included more maize and pumpkin than other Indian tribes. (A) pumpkin than other Indian tribes (B) pumpkin than did those prepared by other Indian tribes (C) pumpkin than that which other Indian tribes did (D) pumpkin, and this was not the same as other Indian tribes (E) pumpkin; and other Indian tribes did not prepare meals in this way

10835

10895

10840

10900

10845

14. Born of Ihuza parents in Nigeria, novelist Buchi Emecheta moved to England in 1962, since which she 10905 has lived in North London. (A) (B) (C) (D) (E) 1962, since which she has lived in North London 1962 and has lived since then in North London 1962, since then she has lived in North London 1962 and lived since then in North London 1962, and living in North London since that lime

10850

(A) (B) (C) (D) (E)

were the basis for have been a basis for become the basis of was the basis for being the basis of

10910

10855

You might also like